Download as pdf or txt
Download as pdf or txt
You are on page 1of 124

www.sakshieducation.

com
Quantitative Aptitude

1. NUMBER SYSTEM
In Indian system, numbers are expressed by means of symbols 0,1,2,3, 4, 5, 6, 7, 8, 9, called digits.
Here, 0 is called insignificant digit whereas 1 ,2 ,3 ,4 ,5 ,6,7 , 8,9 are called significant digits. We can
express a number in two ways.
Notation: Representing a number in figures is known as notation as 350.
Numeration: Representing a number in words is known as numeration as ‘Five hundred and forty
five.
Face Value and Place Value of a Digit:
Face Value: It is the value of the digit itself.
e.g. in 3452, face value of 4 is ‘four’, face value of 2 is ‘two’.
Place Value: It is the face value of the digit multiplied by the place value at which it is situated
e.g. in 2586, place value of 5 is 5×102 = 500.
Number Categories -
Natural Numbers (N): If N is the set of natural numbers, then we write N = {1 ,2 ,3 ,4 , 5 6,...}
The smallest natural number is 1.
Whole Numbers (W): If W is the set of whole numbers, then we write W = {0, 1,2,3,4, 5,...}
The smallest whole number is 0.
Integers (I): If I is the set of integers, then we write I = {- 3, -2, - 1, 0 , 1, 2, 3,...}
Rational Numbers: Any number which can be expressed in the form of p/q, where p and q are both
integers and q # 0 are called rational numbers.
 
E.g. : , , , 
 
There exists infinite number of rational numbers between any two rational numbers.
Irrational Numbers Non-recurring and non-terminating decimals are called irrational numbers.

These numbers cannot be expressed in the form of .


E.g. √ , √,√ …
Real Numbers: Real numbers include both rational and irrational numbers.
Basic Rules on Natural Numbers
1. One digit numbers are from 1 to 9. There are 9 one digit numbers. i.e. °
2. Two digit numbers are from 10 to 99. There, are 90 two digit numbers, i.e.

3. Three digit numbers are from 100 to 999. There are 900 three digit numbers i.e.

        

4. Sum of the first n natural numbers i.e. 1+ 2 + 3+ 4 + ...+n =

  
5. Sum of the squares of the first n natural numbers i.e. 12+ 22+ 32 + 42 + ...+ n2 =


3
6. Sum of the cubes of the first n natural number i.e. 1 + 2 + 3 + ...+n = 3 3 3
!2

Example: What is the value of 51 + 52 + 53 + ...+ 100 ?
Solution: 51 + 52 + 33 + ... + 100 = (1 + 3 + ...+ 100) — (1 + 2 + 3 + ... + 50)
  
"  "    # " ##
 
Different Types of Numbers:
Even Numbers: Numbers which are exactly divisible by 2 are called even numbers,
e.g., - 4 , - 2 , 0,2, 4...
Sum of first n even numbers = n (n + 1)

www.sakshieducation.com
www.sakshieducation.com
Quantitative Aptitude

Odd Numbers: Numbers which are not exactly divisible by 2 are called odd numbers.
e.g., -5 ,-3 ,-1 ,0 , 1,3,5...
Sum of first n odd numbers = n2
Prime Numbers: Numbers which are divisible by one and itself only are called prime numbers.
E.g. 2,3,5,7,11...
• 2 is the only even prime number.
• 1 is not a prime number because it has two equal factors.
• Every prime number greater than 3 can be written in the form of (6K+ l) or (6K -1) where K is an
intE.g.er. There are 15 prime numbers between 1 and 50 and 10 prime numbers between 50 and 100.
Relative Prime Numbers: Two numbers are said to be relatively prime if they do not have any
common factor other than 1.
e.g. (3,5),(4,7),(11,15),(15,4)...
Twin Primes: Two prime numbers which differ by 2 are called twin primes.
e.g., (3,5),(5,7),(11,13),...
Composite Numbers: Numbers which are not prime are called composite numbers.
e.g., 4,6,9,15,...
1 is neither prime nor composite.
Perfect Number: A number is said to be a perfect number, if the sum of all its factors excluding
itself is equal to the number itself, e.g. Factors of 6 are 1, 2,3 and 6.
Sum of factors excluding 6 = 1 + 2 + 3= 6.
6 is a perfect number.
Other examples of perfect numbers are 28,496, 8128 etc.
Rules for Divisibility:
Divisibility by 2: A number is divisible by 2 when the digit at ones place is 0,2 ,4 ,6 or 8.
e.g., 3582,460,28,352,..........
Divisibility by 3: A number is divisible by 3 when sum of all digits of a number is divisible by 3.
e.g., 453 = 4 + 5 + 3= 12.
12 is divisible by 3 so, 453 is also divisible by 3.
Divisibility by 4: A number is divisible by 4, if the number formed with its last two digits is divisible
by 4. e.g., if we take the number 45024, the last two digits form 24. Since, the number 24 is divisible
by 4, the number 45024 is also divisible by 4.
Divisibility by 5: A number is divisible by 5 if its last digit is 0 or 5.
e.g., 10,25,60
Divisibility by 6: A number is divisible by 6, if it is divisible by both 2 and 3.
e.g., 48,24,108
Divisibility by 7: A number is divisible by 7 when the difference between twice the digit at ones
place and the number formed by other digits is either zero or divisible by 7.
e.g. 658
6 5 - 2 × 8 = 65 -16 = 49
As 49 is divisible by 7 the number 658 is also divisible by 7.
Divisibility by 8: A number is divisible by 8, if the number formed by the last 3 digits of the number
is divisible by 8.
e.g. if we take the number 57832, the last three digits form 832. Since, the number 832 is divisible by
8, the number 57832 is also divisible by 8.
Divisibility by 9: A number is divisible by 9, if the sum of all the digits of a number is divisible by 9.
www.sakshieducation.com
www.sakshieducation.com
Quantitative Aptitude

e.g. 684 = 6 + 8 + 4=18.


18 is divisible by 9 so, 684 is also divisible by 9.
Divisibility by 10: A number is divisible by 10, if its last digit is 0. e.g. 20,, 180,350,....
Divisibility by 11: A number is divisible by 11 When the difference between the sum of its digits in
odd places and in even places is either 0 or divisible by 11.
e.g. $%
& $ & % "
& "
 "
As the difference is divisible by 11 the number 30426 is also divisible by 11.
Division on Numbers
In a sum of division, we have four quantities.
- They are (i) Dividend, (ii) Divisor, (iii) Quotient and (iv) Remainder. These quantities are
connected by a relation.
(a) Dividend = Divisor × Quotient + Remainder.
(b) Divisor = (Dividend-Remainder) ÷Quotient.
(c) Quotient = (Dividend-Remainder)÷Divisor.
Example 2: In a sum of division, the quotient is 110, the remainder is 250, the divisor is equal to the
sum of the quotient and remainder. What is the dividend ?
Solution: Divisor =(110+ 250) = 360
Dividend =(360 ×110)+250 = 39850
Hence, the dividend is 39850.
Example 3: Find the number of numbers up to 600 which are divisible by 13.
Solution: Divide 600 by 13, the quotient obtained is 46. Thus, there are 46 numbers less than 600
which are divisible by 13.
Factors and Multiples
Factor: A number which divides a given number exactly is called a factor of the given number.
e.g. $ " $,  , ', $ %
Thus, 1,2,3,4, 6, 8,12 and 24 are factors of 24.
• 1 is a factor of every number
• A number is a factor of itself
• The smallest factor of a given number is 1 and the greatest factor is the number itself.
• If a number is divided by any of its factors, the remainder is always zero.
• Number of factors of a number are finite.
Number of Factors of a Number: If N is a composite number such that ( " ) * + . ..
where a, b, c ... are prime factors of N and m, n, o ... are positive integers, then the number of factors
of N is given by the expression (m + 1) (n + 1)(o+ 1)
Example 4: Find the number of factors that 224 has.
Solution: 224 = 25 × 71
Hence, 224 has (5 + 1) (1 + I) = 6 x 2 = 12 factors.
Multiple: A multiple of a number is a number obtained by multiplying it by a natural number e.g.
Multiples of 5 are 5,10,15,20.
Multiples of 12 are 12,24,36,48
• Every number is a multiple of 1.
• The smallest multiple of a number is the number itself.
www.sakshieducation.com
www.sakshieducation.com
Quantitative Aptitude

• We cannot find the greatest multiple of a number.


• Number of multiples of a number are infinite.
EXERCISE
(c) 1 (d) 4
||||  10. What is the unit digit in (476 × 198 x 359
1. Evaluate:
  
×242)?
(a) 9/10 (b) -8/17
(a) 8 (b) 6
(c) -16/19 (d) 4/7
(c) 4 (d) 2
2. Find the product of place value and face
11. Find the least value of k so that 39 k20 is
value of 5 in 65231.
divisible by 3.
(a) 28000 (b) 25000
(a) 1 (b) 3
(c) 27000 (d) 26000
(c) 5 (d) 2
3. The sum of three consecutive natural
12. Find the least value of K so that 36 K36 is
numbers each divisible by 3 is 72. What is
divisible by 6 .
the largest among them?
(a) 1 (b) 6
(a) 25 (b) 26
(c) 2 (d) 0
(c) 27 (d) 30
13. Find the number of divisors of 10800.
4. 55% of a number is more than one-third
(a) 57 (b) 60
of that number by 52. What is two-fifth of
(c) 72 (d) 62
that number?
14. N = n! where n is natural number. The
(a) 96 (b) 240
unit’s digit of N can’t be
(c) 144 (d) 142
(a) 2 (b) 6
5. A number gets reduced to its one-third
(c) 5 (d) 0
when 48 subtracted from it. What is two –
15. Find the number of prime factors of
third of that number ?
 # 
(a) 22 (b) 76
(a) 22 (b)21
(c) 36 (d) 48
(c) 6 (d) 18
6. The digits of a two-digit number are in the
16. Which is smallest prime number?
ratio of 2 : 3 and the number obtained by
(a) 0 (b) 1
interchanging the digits is bigger than the
(c) 2 (d) 3
original number by 27. What is the
17. Which of the following is a prime
original number?
number?
(a) 63 (b) 48
(a) 149 (b) 437
(c) 96 (d) 69
(c) 319 (d) 567
7. What least number would be subtracted
18. The rational number. Which equals the
from 427398 so that the remaining
number . ,,,,,,
# with recurring decimal is:
number is divisible by 15?  
(a) 13 (b) 3 (a) (b)
 
(c) 16 (d) 11 
(c) (d) none of these

8. Find the number which when multiplied
19. Find the sum of the first 50 natural
by 13 is increased by 180:
numbers.
(a) 20 (b) 15
(a) 1275 (b)1025
(c) 124 (d) 5
(c) 1235 (d) 1205
9. Find unit digit in (515) + (515)90:
31

(a) 0 (b) 5

www.sakshieducation.com
www.sakshieducation.com
Quantitative Aptitude

20. Find the sum of all natural numbers from 30. For any natural number n, what is the
100 to 175. value of
(a) 10456 (b )10452
& & & -& ?
(c) 10450 (d) 10455 .  . .$ ./. & 0
21. Find the sum of all natural-numbers (a) 1 (b) >1 (c) <1
between 100 and 175. (d) Can’t be determined
(a) 10450 (b)10175  
31. Arrangement of – , ).2  3. the
  
(c) 10170 (d) 10435
ascending order is:
22. What least number must be subtracted    
from 3475 to make it divisible by 50 ? (a)  ,  , (b) ,  , 
     
   
(a) 75 (b )100 (c)  ,  , (d)  , , 
     
(c) 25 (d) 50 32. Which one of the following numbers will
23. How many numbers up to 800 are completely divide
divisible by 24? 4 &  &  &  &  5?
(a) 30 (b)29
(a) 7 (b) 11 (c) 9 (d) 13
(c) 33 (d) 26
33. The sum of the numerator and
24. How many numbers up to 700 are
denominator of a fraction is equal to 5.
divisible by both 3 and 5?
Five times the numerator is 4 more than
(a) 42 (b) 46
twice the denominator. The fraction is:
(c) 39 (d) 52    
25. Find the number nearest to 2559 which is (a) (b) (c) (d)
   
exactly divisible by 35. 34. How many integers from 1 to 100 exist
(a) 2535 (b) 2555 such that each is divisible by 5 and also
(c) 2540 (d) 2560 has 5 as a digit ?
26. The prime numbers dividing 109 and (a) 10 (b) 11
leaving a remainder of 4 in each case are: (c) 12 (d) 20
(a) 5 and 7 (b) 2 and 11 35. In the array 48392874362754869364, the
(c) 3 and 8 (d) 11 and 12 number of instances where an even
27. Sum of two numbers is 60 and their number is followed by two odd numbers
difference is 12. Find their product. is
(a) 864 (b) 852 (a) 1 (b) 2
(c) 824 (d) 836 (c) 3 (d) 4
28. Out of four consecutive prime numbers, 36. Ashok had to do a multiplication. Instead
the product of first three is 385 and the of taking 35 as one of the multipliers, he
product of the last three is 1001. Find the took 53. As a result, the product went up
last number? by 540. What is the new product?
(a) 7 (b) 11 (a) 1050 (b) 1590
(c) 13 (d) 17 (c) 1440 (d) None of these
29. If a piece of road is 3000 m and we have 37. The number zero (0) is surrounded by the
to supply some lamp posts. One post is at same two digit number on both (left and
each end and distance between two right) sides: for example 25025, 67067
consecutive lamp posts is etc. The largest number that always
75 m. Find the number of posts required. divides such a number is
(a) 41 (b) 39 (a) 7 (b) 11
(c) 40 (d) 36 (c) 13 (d) 1001
www.sakshieducation.com
www.sakshieducation.com
Quantitative Aptitude

38. A number when divided by 119 leaves the    


(a) , (b) ,
   
remainder 19. If the same number is    
divided by 17, the remainder will be (c) , (d) ,
   
(a) 19 (b) 10 42. ( " /) * + 20, where a, b, c, d
(c) 7 (d) 2 are distinct integers lying between -7 and
39. A worker was engaged for a certain 12, both inclusive. The minimum value of
number of days and was promised to be N is
paid Rs. 1755. He remained absent for (a)-9240 (b)-840
some days and was paid 1365 only. What (c) -2520 (d) None of these
were his daily wages? 43. Find the number of digits that are to be
(a) Rs. 182 (b) Rs. 195 used in numbering a book of 500 pages.
(c) Rs. 185 (d) Rs. 192 (a) 1392 (b) 1346
2 2 (c) 1325 (d) 1352
40. (999 - 998 ) is equal to
(a) 1 (b) 999
(c) 1997 (d) 998
41. The rational numbers lying between

).2 )67:
 
ANSWER KEY
1 c 11 a 21 b 31 a 42 a
2 b 12 d 22 c 32 b 43 a
3 c 13 b 23 c 33 b
4 a 14 c 24 b 34 b
5 d 15 a 25 b 35 c
7 b 16 c 26 a 36 b
6 d 17 a 27 a 37 d
8 b 18 c 28 c 38 d
9 a 19 a 29 a 39 b
10 c 20 c 30 c 40 c

SOLUTIONS

|||| 
1.
  
     
=
  
 
= =-
   
2. Place value = 5× 1000 = 5000
Face value = 5
Product = 5000× 5 = 25000
3. 9 & / 9 & 0 & / 9 & %0 " #
9 & " # : 9 " # 

Or 9 " "#

;The largest of them is 27.
www.sakshieducation.com
www.sakshieducation.com
Quantitative Aptitude

4. Let the number be x.



9" 9 & 

9 "  : 9 " $

 
; 9" $ " %
 
5. Let the number be x. Then
9  $' : 9 : 9  9 " $'

: 9 " $'

6. Let the number be 10x+ y


9 < = "  < … … . /30
/ = & 90 — / 9 & =0 " # . . . /330
: = 9 " # : = 9 " := "9&
Putting this value of y, in (i)
 
" : 9 "6
 
;y=9
Hence the number is 69.
7. Apply the divisibility rules of 3 and 5.
8. M-I: 9 " ' &9
9  9 " ' or 9 " '
9" 
M-II: OTP
9. Since the unit digit of any number which is ending in 5 to the power of any number will be 5
only. Thus Units place of (515)31 as well as (515)90 is 5. Since the sum of Units place of total
expression is 5 + 5 = 10, thus Zero is the units place of the given expression.
10. Unit digit in 476 × 198×359 × 242
= unit digit in 6 × 8 × 9× 2 = 4
11. 3 + 9 + K + 2 + 0 = 14 + K
Least value of K is 1.
12. Given number is divisible by 6, if it is divisible by both 2 and 3.
;The least value of K is 0.
13. 10800 = 24 x 52 x 33
Number of divisors
= (4+1) (2+1) (3 + 1) = 60.
14. Since, n ! is always even for n > 1. Therefore, 5 cannot be the unit digit.
15. Number of prime factors
" &  & % " 
16. 2 is the smallest prime number.
17. We know that  @ 149
Prime numbers less than14 are 2, 3, 5, 7, 11

www.sakshieducation.com
www.sakshieducation.com
Quantitative Aptitude

Clearly none of the numbers divide 149.


Therefore, 149 is a prime number.
# "  & . ,,,,,,
18. . ,,,,,, #
  
"& " "
  
19. Sum of the first 50 natural numbers
 / & 0
"

 
" " #

20. From 100 to 175 mean including both 100 and 175.
Sum of natural numbers up to 99
" " $ 

Sum of natural numbers up to 175
# #%
" " $

; Sum of all natural numbers from 100 to # " $ $  " $
21. Between 100 and 175 means excluding 100 and 175.
Sum of natural numbers up to 100
"  

Sum of natural numbers upto 174
#$ #
" " 

∴ Required sum
"     " #
22. Remainder when 3475 is divided by 50 is 25.
; Least number that should be subtracted
" 
23. Quotient when 800 is divided by 24 is 33.There are 33 numbers up to 800
divisible by 24.
24. Quotient when 700 is divided by the LCM of 3 and 5 i.e., 15 is 46.
;There are 46 numbers upto 700.
25. M-I: 35)2559(73
245
109
105
4
The two numbers nearest to 2559 divisible by 35 are
(a)   $ " 
(b)  & /   $ 0 " 
;The required number is 2555.
M-II: OTP

www.sakshieducation.com
www.sakshieducation.com
Quantitative Aptitude

26. Remainder left in each case is 4.


So 109-4 =105
105 is the number which is exactly divisible by prime numbers.
The two prime numbers which divide 105 are 5 and 7
27. Let the two numbers be x and y.
Then 9 & = " % , and 9  = " 
Solving, 9 " $, = " %
Product = $ % " '%$
28. . Let the four numbers be p, q, r and s.
DE6 " ', E6F "
E6F
" "
DE6 ' 
F
"
D 
So, p = 5 and s = 13.
Required number is 13.
29. Number of lamp posts required
& "$ & "$
#
30. Given expression
& & &-
. . .$ ./. & 0
=G  H & G  H & G  H & - & G  H =1 - " I1
       
(; Denominator is greater than numerator.)

31.  "  . $'


" . %%%#

 " . $

So, the ascending order is given by
-0.4285 < -0.3334 < 0.6667
32. 4 &  &  &  &  5


"  (1+5+25+125+625)
"  (781)
Since 781 is divisible by 11,  ×781 is also divisible by 11.

33. M-I: Let the fraction be .

9 & = "  and 9  = " $
Solving, 9 "  ).2 = "

∴ The fraction is

M-II: OTP
34. These numbers are following 5,15,25, 35, 45, 50, 55, 65, 75, 85, 95
So, total 11 such type of numbers.
35. .In the array 392874362754869364; 8, 2 and 6 i.e., three number of required instances.

www.sakshieducation.com
www.sakshieducation.com
Quantitative Aptitude

36. Suppose, one of the multiplier is 9, then


 9  9 " $
'9 " $
9 "
New multiplication"  " 
37. The largest such number is 1001.
38. M-I: The number is = 119n+ 19 = 138,257
When these numbers are divided by 17, then remainder is 2.
M-II: The remainder when 19 is divided by 17 is 2
39. The daily wages of the worker is the HCF of 1755 and 1365. HCF of 1755 and 1365 is 195.
The daily wages of the worker is '195’.
40. 9992 -9982
= (999 + 998) (999 -998) = (1997)(1)= 1997

41. " . $  " . #
 
 
" .  .# #
 
  
∴ , lies between and
   
42. Take,—7,12,11 and 10 because these will give minimum value.
#  "  $
43. One digit numbers from 1 to 9 = 9
Two digit numbers from 10 to 99 = 90
Three digit numbers from 100 to 500= 401
; Total number of digits that are to be used " / 0 & / 0 & /$ 0
" & ' & 
" 

www.sakshieducation.com
www.sakshieducation.com
Quantitative Aptitude

2. FRACTIONS

A fraction is a part of the whole (object, thing, region). It forms the part of basic aptitude of a
person to have an idea of the parts of a population, group or territory. Aspirants must have a feel of

fractional’ thinking, e.g. here ‘12’ is the number of equal part into which the whole has been

divided, is called denominator and ‘5’ is the number of equal parts which have been taken out, is
called numerator.
 
Example l: Name the numerator of and denominator of .
 

Solution: Numerator of is 3.


Denominator of is 13.

Lowest Term of a Fraction:
Dividing the numerator and denominator by the highest common element (or number) in them, we
get the fraction in its lowest form.

e.g. To find the fraction in lowest form Since ‘2’ is highest common element in numerator 6 and

 
denominator 14 so dividing them by 2, we get Which is the lowest form of .
 
Equivalent Fractions
If numerator and denominator of any fraction are multiplied by the same number then all resulting
fractions are called equivalent fractions.
  
e.g. , , , all are equivalent fractions but is the lowest form.
    

Example 2: Find the equivalent fractions of having numerator 6.

Solution: We know that 2×3=6. This means we need to multiply both the numerator and
denominator by 3 to get the equivalent fraction.
 
Hence, required equivalent fraction " =
 
Addition and Subtraction of Fractions
Here two cases arise as denominators of the fraction are same or not.
Case I: When denominators of the two fractions are same then we write denominator once and add
(or subtract) the numerators.
  
E.g.: & "
  
Case II: If denominators are different, we need to find a common denominator that both
denominators will divide into.

E.g.: &
 
  
We can write, & " "
   

Example 3: Calculate 
 
 
Solution :  " "
   

Multiplication and Division of Fractions

www.sakshieducation.com
www.sakshieducation.com
Quantitative Aptitude

To multiply fractions, the numerators are multiplied together and denominators are multiplied
together.
  
E.g.: " = "
    
In division of fraction, the numerator of first fraction is multiplied by the denominator of second
fraction and gives the numerator. Also denominator of first fraction is multiplied by the numerator of
second fraction and gives the denominator.
   
J *7+KL7F " "
     
Proper and Improper Fractions
The fractions, in which the number in numerator is less than that of denominator, arc
called proper fractions. Also if the number in numerator is greater than that of denominator, then the
fractions are called improper fractions.
 
E.g.: is an improper fraction while is a proper fraction.
 
Mixed Numbers
A mixed number is that, which contains both a whole number and a fraction.
 
E.g. $ , , % are mixed numbers.
  
Example 4: Which of the following are proper and improper fractions?
   
(a) (b) (c) (d)
   
Solution: (a) and (d) are proper fractions as numerator is less than denominator.
Also, (b) and (c) are improper fractions as numerator is greater than denominator.
 
Example 5: Are and $ mixed numbers?
 
 
Solution: is only a proper fraction as it does not contain any whole number, while $ is a mixed
 

number as it contains ‘4’ as a whole number and   M.

Decimal Fractions
The fractions in which denominators has the power of 10 are called decimal fractions.

E.g.: 0.25= = =one quarter
 
0.1 = nought point one= "   

For converting a decimal fraction into simple fraction, we write the numerator without point and in
the denominator, we write ‘ 1 ’ and put the number of zeros as many times as number of digits after
the point in the given decimal fraction.
 
E.g. 0.037= , . # "
 
Example 6: Convert each of the following decimal fractions into simple fractions.
(a) 5.76 (b) 0.023 (c) 257.5
Solution:
  
(a) 5.76= " "
  

(b) 0.023=

  
(c) 257.5= =515=
 

www.sakshieducation.com
www.sakshieducation.com
Quantitative Aptitude

Addition or Subtraction of decimal Fractions: In the addition or subtraction of decimal fractions,


we write the decimal fractions in such a way that all the decimal points are in the same straight line
then these numbers can be add or subtract in simple manner.
Example 7: Solve 0.68 + 0.062 + 0.20
0.680
Solution: +0.062
+0.200
0.942
Multiplication of Decimal Fractions
To multiply by multiplies (powers) of 10 the decimal point is moved to the right by the, respective
number of zero.
Example 8: 0.75 × 10 = ?
Solution: 0.75 × 10 = 7.5 (The decimal point is shifted to right by one place)
To multiply decimals by number other than 10. We ignore the decimal point and multiply them in
simple manner and at last put the points after the number of digits (from right) corresponding to the
given problem.
Example 9: Multiply 8 and 10.24
Solution: First we multiply 8 and 1024
8 × 1024 =8192
Now, 8 × 10.24 =81.92
(We put decimal points after two digits from right as in given question)
Example 10: 12.4 ×1.62
Solution: We know that
124× 162 =20088
12.4× 1.62 =20.088
Division of Decimal Fractions
Division of decimal numbers is the reverse of the multiplication case ie, we move the decimal point
to the left while dividing by multiplies of 10.
Example 11: 25.75 ÷10 = ?
Solution: 25.75 ÷10 = 2.575
When a decimal number is divided by an intE.g.er, then at first divide the number ignoring the
decimal point and at last put the decimal after the number of digits (from right) according to the
given problem.
Example 12: Divide 0.0221 by 17
Solution: First we divide 221 by 17 i.e. without decimal 221 ÷17 = 13
Now, we put decimal according as in the given problem.
0.0221 ÷17 =0.0013
If divisor and dividend both are decimal numbers then first we convert them in simple fraction by
putting number of zeros in the denominator of both. Then divide in the above manner.
Example 13: Divide by 0.0256 by 0.016
.    .
Solution: " " " " .%
.     
Example 14: Divide 70.5 by 0.25
.   
Solution: " " " '
.   
To Find HCF and LCM of Decimal Fractions
www.sakshieducation.com
www.sakshieducation.com
Quantitative Aptitude

First we make the decimal digits of the given decimal numbers, same by putting some number of
zeros if necessary. Then find HCF or LCM ignoring decimals. And at last put the decimal according
as the given numbers.
Example 15: Determine HCF and LCM of 0.27, 1.8 and 0.036.
Solution: Given numbers are 0.27, 1.8 and 0.036.
Or 0.270,1.800 and 0.036.
These numbers without decimals are 270, 1800 and 36.
Now, HCF of 270, 1800 and 36 =18
HCF of 0.270,1.800 and 0.036 = 0.018
; LCM of 270, 1800 and 36 = 18 ×5 ×2×3×10 = 5400
18 270 1800
36
5 15 100 2
2 3 20 2
3 10 1
; LCM of 0.270, 1.800 and 0.36 = 5.400 or 5.4
Terminating and Non-Terminating Recurring Decimals
If decimal expression of any fraction be terminated then fraction is called terminating.

as = 0.3125

But if we take example 33÷ 26, then
26) 33 (1.2692307
26
70
52
180 —> A
156
240
234
60
52
80
78
200
182
180 —> B
In this division, we see that remainder at the stages A and B are same. In the continued process of
division by 26, the digits 6,9 ,2,3 ,0,7 in the quotient will repeat onwards.
; " . %  #%  #. ..
%
This process of division is non terminating. Therefore, such decimal expressions are called non
terminating repeating (recurring) decimals.
In repeating digit, we put (-) bar.

ie, = . ,,,,,,,,,,,,,
%  #


www.sakshieducation.com
www.sakshieducation.com
Quantitative Aptitude

Example 16: Write the following fractions in decimal form and test that these are terminating or non
terminating recurring.
   
(a) (b) (c) (d)
  
Solution:

N non terminating recurring
(a) = 0.6666.... = . %


(b) = 0.8 terminating


(c) = 0.272727... = . ,,,,
# non terminating recurring

(d) = 0.188 8 = . ,,,,
' non terminating recurring

Non-Terminating, Non-Recurring Decimals
Every fraction can be put in the form of terminating or non-terminating recurring decimals ie, these

decimal numbers can be put in the form of . These are called rational numbers. But some decimals


numbers are there that can’t be put in the form of , these are non-terminating, non-recurring

decimals. Also these are called irrational numbers,
E.g.. 0.101001000100001...
To convert non-terminating recurring decimals into simple fraction
First write the non-terminating recurring decimal in bar notation. Then write the digit 9 in the
denominators as many times as number of digits recurring in the numerator. Also don’t put decimal
in the numerator.
Example 7: Convert the following in simple fraction
(i)0.33333….. (ii) 0.181818….
  
(iii) 0.33333…." . N " " (iv) 0.181818…" . ,,,, '" "
  
Mixed Recurring Decimals
A decimal fraction in which some digits are not repeated and some are repeated, called mixed
recurring decimal.
How to Convert Mixed Recurring Decimal into a Simple Fraction?
First we subtract non repeated part from the number (without decimal) and put number 9 as
many times as number of recurring digits and also put the number ‘01 as many times odd number of
non-recurring digits.
Example 18: Convert the following in simple fraction
(i) . ,,,,
' (ii) . # ,,,,
Solution:, (i) 0.,,,,' = 0.1888……………. (i)
Multiply both side by 10
10 × 0.18 = 1.888……………(ii)
Multiply both side by 100
,,,,
100 × . ' = 18.888 ………..(iii)
Subtract (ii) from (iii) we have
90 × . ,,,,' = 18-1
#
; . ,,,,
'"
  
(ii) . ,,,, = 3+ 0.00#
# ,,,, = 3 + =3+ =3
  

www.sakshieducation.com
www.sakshieducation.com
Quantitative Aptitude

   
Example 19: Arrange , , and in ascending order.
   

Solution: Here =0.67


= 0.86


=0.80


= 0.94

Here, it is clear that 0.67 < 0.80 < 0.86 < 0.94
 $ 
O I I I
  %
   
Example20: Arrange , , ).2 in descending order,
  
  
Solution: Here = 0.77, = 0.72, =0.38
 

=0.28

Here it is clear that 0.77 > 0.72 > 0.38 > 0.28
# '  
O @ @ @
#
Note: We see other methods of solving this kind of problems in the upcoming chapters.
EXERCISE
. . .  . . .
1. =? (a) (b)
......  
(a) 0.09 (b) 0.9 (c) (d)
 
(c) 0.009 (d) 0.001
8. Which one of the following is least?
2. Which of the following is proper  
fraction? (a) (b)
 
  
(a) 1 (b) (c) (d)
  
  9. Simplify:
(c) (d)
 

3. 75.83 ÷ 1000 is equal to $J# 
P & Q & J 
(a) 7.583 (b) 0.7583  ' $ # $ #
(c) 0.07583 (d) 0.007583 %
  (a) 2/9 (b) 56/77
4. 1 ×1 is equal to 
  (c) 50/73 (d) 3
  
(a) (b) 2
  10. The rational number, which equals the
 
(c) 1

(d) 3

number 2.,,,,,,
# with recurring decimal is:
5. Solve 0.635 + 1.87 + 2.9+ 18.358 + (a) 235/101 (b) 2355/999
0.02345 (c) 2335/1001 (d) 2379/997

(a) 23.78645 (b) 22.87654 11. Sarita bought m of ribbon and Lalita

(c) 23.88456 (d) 24.78645 
m of ribbon.What is the total
6. What will be the approximate value of 

779.5 ×15 -46.5×19-9? length of the ribbon they bought?



(a) 10800 (b) 18008 (a) 1m (b) 1 m

(c) 10080 (d) 10008 
(c) 2m (d) 2 m
  
7.  is equal to
 

www.sakshieducation.com
www.sakshieducation.com
Quantitative Aptitude

12. What approximate value should come in   


17. Ascending order of , ,  is
   
place of z in the following question?
242 + 2.42 + 0.242 + 0.0242 = z      
(a) 670 (b) 575 (a) , , , (b) , , ,
       
     
(c) 580 (d) 680 (c) , , , (d) , , ,
       
13. If √  = 45, then the value of
√ .   + √ .   &
√  +√ .  =
(a) 49.95 (b) 49.5495
(c) 4.9995 (d) 499.95
14. If 4137÷ 1.75 = 2364, then 41.37
÷17.5=?
(a) 23.64 (b) 2.364
(c) 0.2364 (d) 236.4
15. .The simplification of . % N & .#
N &
N & . ,,,, yields:
.'
(a) . ,,,,
$% (b) . ,,,,
%
(c) . ,,,,
$% ,,,,
(d) . $%
16. The numerator of a non-zero rational
number is five less than the denominator.
If the denominator is increased by eight
and the numerator is doubled, then again
we get the same rational number. The
required rational number is:
(a) 1/8 (b) 4/9
(c) 2/8 (d) 3/8

ANSWER KEY
1 a 5 a 9 d 13 b 17 b
2 c 6 a 10 b 14 b
3 c 7 a 11 b 15 a
4 b 8 a 12 c 16 d

SOLUTIONS

2. is proper fraction as 2<3.
.   .  
1. 3. 75.83 ÷ 1000 = 0.07583
. ૛ .. . ૛
    
If 0.05=a, 0.04=b 4. 1 " " "
     
૜ ૜
Then , = a+b 5.
૛ ૛
= 0.05 + 0.04 = 0.09 0.63500

www.sakshieducation.com
www.sakshieducation.com
Quantitative Aptitude

1.87000 = 581.818176 ≃ 580


2.90000 13. √  = 45
18.35800 √ .   + √ .   +
0.02345
√  + √ . 
23.78645
= 0.0045 + 0.045 +45 +4.5
= 49.5495
6. ## .    $%.    . .  .  
14. " "
= 11692.5-883.5-9 . . . 
  
= 11692.5 - 892.5=10800 " " =2.364
.  
૜ ૠ ૜ൈ૝ ૠ ૚૛ ૠ ૞ ૚
7.        N +0.#
15. 0.% N+0.,,,,
N+0.'
૞ ૛૙ ૞ൈ૝ ૛૙ ૛૙ ૛૙ ૛૙ ૝
  
" . $$, % # ' $$
8.

= 0.285, =0.375,
 " & & & " " . ,,,,
$%

" . % 16. M-I:Let the denominator be x, then


% by the given condition
Clearly 0.285 is least among them. 
 the rational number is .
Thus is the least number. 
   

૜ Also, = or
     
9. ૝
૞ J G & H& J KR 
     
૟ " K6 9 " 9 & ' K6 9 " '
Using BODMAS rule, 9' 9

૚૚ The rational number is .
    
" ૝
૚૚ J JG H
    M-II: OTP

       
"G J H &G J H 17. " . #%, " . # , " . ', "
        
% # #  ' . 
"P J Q &P Q   
$ '  # ∴ ascending order is , , ,
   
 # 
P Q&
# 

"

10. 2.,,,,,,
# =2 + 0.,,,,,,
# =2+

 & #  
" "
   
11. Total length of ribbon = & =
  
 
" L
 
12. 242 + 2.42 + 0.242 + 0.0242
= 576 + 5.76 + 0.0576 + 0.000576

www.sakshieducation.com
www.sakshieducation.com
Quantitative Aptitude

3. SQUARE ROOT & CUBE ROOT

Square Root
The square root of a number is that number the product of which itself gives the given number,
i.e. the square root of 400 is 20, the square root of 625 is 25.
The process of finding the square root is called evaluation. The square root of a number is
denoted by the symbol “√”called the radical sign. The expression “√ ” is read as “root nine”,
“radical nine” or “the square root of nine”.
How to Find the Square Root of an Integer?
(i) By the method of Prime Factors: When a given number is a perfect square, we resolve it
into prime factors and take the product of prime factors, choosing one out of every two.
Example 1: Find the square root of 4356.
Solution:
2 4356
2 2178
3 1089
4 363
11 121
11
4356 = 2× 2 × 3 ×3 ×11 × 11=22 × 32 × l l2
√$ % = 2 ×3× 11 = 66
Thus from the above example it is clear that in order to find the complete square root of a given
number every prime factor of that number should be repeated twice. Thus, we can make a number
which is not a perfect square, a perfect square by multiplying or dividing the number by those factors
of it which are not contained in pairs.
Example 2: Find the least number by which 1800 be multiplied or divided to make it a perfect
square.
Solution: 1800 = 2×2× 2×3×3×5×5
The least number by which the given number be multiplied or divided is 2.
To Find the Square Root of a Decimal
 √  
Example 3: √ . ' = S = = = 1.35
 √  
To Find the Square Root of a Fraction

Example 4: Find the square root of .

  √ 
Solution: S =S = = =
  √  
• The square of a number other than unity is either a multiple of 4 or exceeds a multiple of 4 by 1.
• A perfect square can never end with (a) an odd number of zeroes and (b) 2, 3, 7 and 8.
• The square root of an-intE.g.er is not always an integer i.e., √ , √, √ are not integers.
• √)* = √) ×√*
 √
•S =
 √

• √)+√* T √) & *

www.sakshieducation.com
www.sakshieducation.com
Quantitative Aptitude

• √)-√* T √)  *
Example 5: Find the square root of 0.0016.
 √  
Solution: √ . %=S = = = 0.04
 √  

.
Example 6: Find the value of x if S = 8.


. .
Solution: Given S =8: = 64
 
: 64x = 25.6 : x=0.4
Example 7: If √ = 2.236 , find the value of √$ - √' -√ correct to three places of decimal.

√ 
Solution: √$ -  √' -√ = √$ –

– √$ 
√
= 7√- -2√ = 7√ -2√ - 2√

= 3√ = 3×2.236 = 6.708
Example 8: Find the smallest number that must be added to 2400 to make it a perfect square.
Solution:
48² = 2304
49² = 2401
Number to be added = (49)2-2400 = 2401 - 2400 =1
Cube Root
The cube root of a number is that number the cube of which itself gives the given number i.e. the
૜ ૜
cube root of 64 is 4. The cube root of a number is denoted by the symbol √ . The expression √' is
read as “cube eight” or the “cube root of eight”.
To Find the Cube Root of an IntE.g.er:
(i) By the method of prime factors: When a given number is a perfect cube, we resolve it into
prime factors and take the product of prime factors, choosing one out of every three.
Example 9: Find the cube root of 74088.
Solution:
2 74088
2 37044
2 18522
7 9261
7 189
3 27
3 9
3 3
1

74088 = 23 × 73× 33

√#$ '' = 2 ×7 ×3 = 42

To Find the Cube Root of a Decimal:


Example 10: Find the cube root of 19.683.

www.sakshieducation.com
www.sakshieducation.com
Quantitative Aptitude

૜ ૜
૜ ૜  √  ૢ ૜ 
Solution: √ . %' = S = ૜ =૜ = = = 2.7
 √   ૜  
3
19683
3
6561
3
2187
7293
2433
81 3
27 3
9 3
3 3
1
19683 = 39
To Find the Cube Root of a Fraction:

Example 11: Find the cube root of 1

૜ ૜
૜  ૜  √  ૜
Solution: S =S = ૜ =૜ =1
  √ ૜ 

૜ ૜ ૜ ૜  √
(1) √)* = √) × √* (2) S = ૜
 √
૜ ૜ ૜ ૜ ૜ ૜
(3) √) + √* T √) & * (4) √) - √* T √)  *
Example 12: Find the smallest number by which 2400 be divided to make it a perfect cube.
Solution: 2400 = 25 ×52×3
To make it a perfect cube it must be divided by 22×52 × 3 = 300

EXERCISE
1. Square root of 2025 is (c) 2.28 (d) 2.22
(a) 35 (b) 45 6. If √ . #% = 0.0024, then the
(c) 55 (d) 65 square root of 57,60,000 is
2. Find the square root of 104976 (a) (b) 2400

(a) 322 (b) 324
(c) 240 (d) 24
(c) 326 (d) 328
3. Find the value of 7. Find the square root of (8 + 2√#)(8-2√#) .
(a) 6 (b) 4
U # & S & V  & √' (c) 8 (d) 9

8. S# = x. Find x.
(a) 11 (b) 5 

(c) 7 (d) 9 (a) 7.15 (b) 8.25


(c) 5.25 (d) 6.15
4. V ' & √% =?
(a) 19 (b) 15 9. If √ = 1.732, then the value of approx
√
(c) 17 (d) 13 is.
5. Find the value of (a) 0.577 (b) 0.477
√ . +√ . ' +√ . $$+√ . $ (c) .0.512 (d) 0.417
(a) 2.42 (b) 2.24

www.sakshieducation.com
www.sakshieducation.com
Quantitative Aptitude

10. Find the least six-digit perfect square 13. Find the least number by which 234375
number. be divided to make it a perfect cube.
(a) 100009 (b) 100289 (a) 20 (b) 8
(c) 100441 (d) 100489 (c) 15 (d) 10
11. What is the smallest number by which 14. Find the least perfect square number
1400 be divided to make it a perfect divisible by 2, 3, 4, 5 and 6.
cube? (a) 900 (b) 1600
(a) 130 (b) 145 (c) 2700 (d) 400
(c) 160 (d) 175

12. Find the cube root of 4 .

 
(a) 1 (b) 1
 

(c) (d) None of these

Answer Key
1 b 5 a 9 a 13 c
2 b 6 b 10 d 14 d
3 b 7 a 11 d
4 d 8 c 12 a

SOLUTIONS
1.   = 5 × 5 × 9 × 9      
" & & & " &
     
√  = 45 $
2. (b)From the options we can say the " " . $
square root of 104976 is either 324 or 6. √#% =√ . #% 
326, and by inspection method we can ૚૛
√.  √   . 
say √ $ #% = 324  ૟

" $
3. Given U # & S & V  & √'
7. S/' & √#0/'  √#0

= S # & V & √  & # = √%$  ' =√ % = 6.


  √
8. Given S# "S "
= S # & V & √ %   √ 

= V # & √ & " " . 
$
= V # & √%$ 9.
√
"
√
"
.
" . ##
√ √  
= √ # & ' = √ = 5 10. Least number of 6 digits is 100000.
4. √% = 51 3 100000 316
V ' & √% = √ '& = 9
√ % = 13 100
5. Given expression 61
    626 3900
=S +S ÷S +S 3756
   

www.sakshieducation.com
www.sakshieducation.com
Quantitative Aptitude

144 Now 60 = 2 × 2 × 3 × 5.
/ % I 100000 I / #0
0 To make it a perfect square it must be
Hence required number = / #0  = 100489 multiplied by 3× 5
11. $ " ³ ² # So required number "  
 "
To make it a perfect cube, it must be
divided by 52 × 7 = 175
૜ ૜
૜  ૜   √  ૜ 
12. S$ =S =૜ =૜ =
  √  ૜ 

=1

7
13. 234375 = 5 × 3
To make it a perfect cube it must be
divided by 5×3 = 15.
14. L.C.M. of 2, 3 ,4 ,5 and 6 = 60.

www.sakshieducation.com
www.sakshieducation.com
Quantitative Aptitude

4. SIMPLIFICATIONS
In simplification of an expression there are certain laws which should be strictly adhered to.
These laws are as follows:
‘VBODMAS’ Rule
This rule gives the correct sequence in which the mathematical operations are to be performed so as
to find out the value of a given expression.
Here, ‘V’ stands for Vinculum (or Bar), ‘B’ stands for ‘Bracket’, ‘O’ stands for ‘Of’, ‘D’ stands
for ‘Division’, ‘M’ stands for ‘Multiplication’, ‘A’ stands for ‘Addition’ and ‘S’ stand for
‘Subtraction’. (a) Here, "VBODMAS’ gives the order of simplification. Thus, the order of
performing the mathematical operations in a given expression are
First: Vinculum or line bracket or bar
Second: Bracket
Third: Of
Fourth: Division
Fifth: Multiplication
Sixth: Addition &
Seventh: Subtraction
The above order should strictly be followed.
(b) There are four types of brackets.
(i) Square brackets [ ]
(ii) Curly brackets { }
(iii) Circular brackets ()
(iv) Bar or Vinculum -
Thus, in simplifying an expression all the brackets must be removed in the order ‘-‘, ‘( )’, ‘{ }’ and ‘[ ]’.
Modulus of a Real Number
The modulus of a real number x is defined as
9 3R 9 @ 0 \
|9| = [
9 3R 9 I 0
Basic Formulae:
(i) (a+b)2 = a2+2ab+b2
(ii) (a-b)2 = a2-2ab+b2
(iii) (a+b)2 -(a-b)2 =4ab
(iv) (a+b)2+(a-b)2 = 2(a2+b2)
(v) (a-b)2 = (a+b)(a-b)
(vi) (a+b+c)2 = a2 + b2 + c2 + 2(ab+ bc +ca)
(vii) (a3+b3) = (a+b)(a2 - ab+b2)
(viii) (a3-b3) = (a-b)(a2+ab+b2)
(ix) (a3+b3+c3-3abc) = (a+b+c)(a2+b2+c2-ab-bc-ca)
(x) (a3+b3+c3) = 3abc , if a+b+c=0

Example 1: Simplify 1005 + 500 - 10 - 80.


Solution: 1005 + 500- 10-80 = 1005 + 490 – 80 = 1495 - 80 = 1415
Example 2: If a* b = 2 (a + b), then what is the value of 5 * 2?
Solution: 5 * 2 = 2(5 + 2) = 2 * 7 = 14

www.sakshieducation.com
www.sakshieducation.com
Quantitative Aptitude

Example 3: 3/5 part of the students in a class are the girls and remaining are the boys. If 2/9 part of
the girls and 1/4 part of the boys are absent, then what part of total students is
present.
Solution: Let the total number of students be 9

Number of girls " 9


Number of boys " 9

    
Number of students absent = G KR 9H & G KR 9H = 9 & 9 " 9
      
 
No. of students Present " /  09 " 9
 
10 9 8 7
Example 4: Simplify (2 - 2 ) (2 - 2 ).
Solution : (210 - 29) (28 - 27) = 29(2-1). 27(2-1) = 29+7 = 216
  ૛ ૛
Example 5: If = find the value of .
  ૛ ૛
࢞૛

૛ ૛ ࢟૛
Solution: Method-I: = ૛ (dividing both numerator and denominator by y2)
૛ ૛ ࢞

࢟૛
࢞૛ ૜૛ ૢ
! ૛ " ! ૛ " 
࢟ ૛ ૝  
= ࢞૛
= ૜૛
= ૢ = =2
! ૛ " ! ૛ "   
࢟ ૛ ૝

Method-II: Replace x and y with 3 & 2 respectively .


......
Example 6: Simplify
.
Solution: 0.3× 0.3 + 0.03× 0.03 - 0.6 ×0.03 = 0.3 × 0.3 + 0.03× 0.03-2×0.3×0.03 is of the form
(a-b)2 = a2-2ab+b2
a=0.3 and b=0.03=(0.3-0.03)2 = (0.27)2
..
Given expression = 0.135
.

EXERCISE
(a) 18 (b) 21
1. What is th
of 3? (c) 16 (d) 24

6. Simplify 1÷ [1+1÷{1+1÷(1÷1)}].
(a) 6 (b) 3 (c) 1/2 (d) 1/3  
2. Divide 0.045 by 100. (a) (b)
 
(a) 0.0045 (b) 0.00045  
(c) (d)
 
(c) 0.000045 (d) 0.45  
7. . Find the value of
3. Simplify 15.876 - (2.49 + 4.056) ÷ . 
 
(a) 2.784 (b) 3.052 (a) (b)
 

(c) 2.984 (d) 3.152 (c) (d)

.. .
4. Simplify
.
8. Simplify [1 - 2 (3- 4 ) - l ]-1
(a) 100 (b) 110 (a) (b)
 
(c) 90 (d) 105
(c) (d)
 
૜ ૚ ૜
  9. (16÷ 4) ÷4 is equal to
5. Simplify ૜૛ ૚૛ ૛
૜ ÷ (a) 4 (b) 1
 # 
૛ ૛ ૛

www.sakshieducation.com
www.sakshieducation.com
Quantitative Aptitude

(c) (d) 16 16. The sum of first 50 positive integers is



1275. What is the sum of the integers from
10. 16 ÷(4 ÷ 4) is equal to
51 to 100?
(a) 4 (b) 1
(a) 2525 (b) 2550
(c) (d) 16 (c) 3250 (d) 3775

11. Simplify 17. 2 tables and 3 chairs cost Rs.3500 while 3
P & QP  QP & QP  Q tables and 2 chairs cost Rs.4000. The cost
$ $   of a table (in rupees) is
P & QP  Q (a) 500 (b) 1000
% % (c) 1200 (d) 1500
 
(a) (b) 18. If x + y - z = m and if x - y + z = n, then x
 
 is equal to
(c) (d)
 
 (a) (m + n)/2 (b) m — n
12. Find P in the expression, if ૚ =1.

૚శ
࢖ (c) 2m + n (d) m + n
૚ష࢖
19. If the numerator of a fraction is double and
(a) 2 (b) the denominator is increased by 3, the new

(c) 1 (d) fraction is 3/5. What is the original

fraction, if its denominator is more than
13. If x # y = x + y, then find the value of (3 #
twice the numerator by 1?
4) # 3.
(a) 3/7 (b) 6/13
(a) 6 (b) 7
(c) 1/3 (d) 5/11
(c) 10 (d) 8
20. It is required to change a rupee coin into 2
14. If 1 + 2 + 3 + ... + 93 = 2025, then find
3 3 3
paisa and 5 paisa coins with the total
the value of (0.1)3 + (0.2)3 + ... + (0.9)3
number of coins equal to 26. Find the
(a) 2.025 (b) 202.5
number of each type of coins.
(c) 20.25 (d) 0.2025
(a) 10 and 16 (b) 12 and 14
15. What least fraction should be added to
(c) 10 and20 (d) 10 and 14
+ + +…..+ , so that the
     21. For what value of x, 8 + (x - 3)2 have the
result is unity. least value?
 
(a) (b) ( a ) -3 (b) 0
  (c) 3 (d) 5
(c) (d)

ANSWER KEY
1 c 6 c 11 c 16 d 21 c
2 b 7 a 12 c 17 b
3 a 8 b 13 c 18
4 a 9 b 14 a 19 a
5 a 10 d 15 b 20 a

www.sakshieducation.com
www.sakshieducation.com
Quantitative Aptitude

SOLUTIONS
1. th of 3 = ×3 = = (0.1)3 (2025) = 0.001 x 2025 = 2.025
  
. 15. Given expression is
2. " . $
 & & & -..&
    
3. The given expression is
= G  H +G  H + G  H +….& G 
15.876-(2.49 + 4.056)÷       
  
= 15.876 -(6.546) ÷ 
H "    "  "


= 15.876-(6.546 ×2) Thus, least fraction to be added = 1- =
= 15.876- 13.092 = 2.784 

4. . $' J .  & . $   


. $' 16. Sum of 1 to 100 =

= 5050
" & . $ 
.  ; Sum of 51 to 100 = sum of 1 to 100 -
=4+1=5 sum of 1 to 50 = 5050- 1275 = 3775

Given expression = = 100 17. 2t + 3c =3500 ……….(i)
.
૜ ૚ ૜
 
૜ ૛ ૜
 
3t+ 2c =4000 ……….(ii)
5. Given expression ૜૛ ૚૛ ૛
૜ ÷ = ૛૜ ૚ ૜૛ J On solving t = Rs.1000
 #   
૛ ૛ ૛ ૛ ૝
ૢ ૢ 18. 9 & =  ] " L  9  = & ] " .
 
= ૛
૜ ૝ J = ૛
÷ = × = 18 Adding the above two we have,
    
૛ ૜
9 " L & . K6 9 " /L & .0/.
6. Given expression 
19. M-I: Let the fraction be
=l÷[l+l÷{l+l÷(1÷l)}] 
 
=1÷[1 + 1÷ {1 + 1÷1}] Then, = : 10x = 6x + 12
  
= 1÷ [1 + 1÷{1 + 1}] : 9  %9 "  : 9 "
  
= 1÷[1 +1÷2] = 1÷[1+ ] = 1÷ = So, the fraction is
   
  
7. Given expression = = = M-II: 0TP
  
8. Given expression =[1 - 2 (3- 4 ) - l -1
] 20. M-I: Let x be the number of 5 paisa coins,
-1 -1 -1 -1 then (26 - x) will be the number of 2 paisa
= [1-2(-1) ] = [1+2] = 3 =
 coins.
9. (16 ÷ 4 ) ÷ 4 = 4 ÷ 4= 1 So, 9 & /%  90  "
10. 16 ÷ (4 ÷ 4 ) = 16 ÷ 1 = 16 9 &   9 "
11. Given expression is 9 " $' "@ 9 " %
G & HG  H G & HG  H G & H Number of 5 paisa coins = 16
    
       Number of 2 paisa coins = 26 — 16 = 10
G  H " "
        M-II: 0TP
 
12. Given, ૚ =1: ૚ =1 21. Clearly for 9 " , given expression has least
 ሺ૚ష࢖ሻశ࢖  ૚
૚ష࢖ ૚ష࢖ value.
 
: =1 : =1
  
: p=2-p or p=1
13. Given x # y = x + y
Then, (3 # 4) # 3 = (3 + 4) # 3 = 7 # 3= 10
14. Given l3 + 23 + 33 + ... + 93 = 2.025
Now, (0.1)3 + (0.2)3 + ... + (0.9)3
= (0.1)3 [l3 + 23 + ... + 93]

www.sakshieducation.com
www.sakshieducation.com
Quantitative Aptitude

5. HCF & LCM


Highest Common Factor (HCF):
HCF of two or more numbers is the greatest number (divisor) that divides all the given
numbers exactly. So, HCF is also called the GREATEST COMMON DIVISOR (GCD).
HCF of two or more than two numbers is the product of the least powers of all the prime
factors that occur in the numbers.
Lowest Common Multiple (LCM):
LCM is the least dividend which is exactly divisible by the given numbers.
LCM of two or more than two numbers is the product of the highest powers of all the prime factors
that occur in the numbers.
LCM of a number will always be divisible by HCF
Product of two numbers:
If ‘A’ and ‘B’ are two numbers and their HCF and LCM are ‘C’ and ‘D’ respectively. So, the
product of those two numbers is the product of HCF and LCM.
HCF of numbers × LCM of numbers = Product of numbers
S. No Type of problem Approach to the problem
1. Find the GREATEST NUMBER that Required number = HCF of given numbers
will exactly divide given numbers. (greatest divisor)
2. Find the GREATEST NUMBER that Required number (greatest divisor)
will exactly divide x, y and z leaving = HCF{(x-a), (y-b) and (z-c)}
remainders a, b and c respectively.
3. Find the LEAST NUMBER which is Required number = LCM of x, y and z (least
exactly divisible by x, y and z. dividend)
4. Find the LEAST NUMBER which when It is always observed that (x-a) = (y-b) = (z-c)
divided by x, y and z leaves the = k (say)
remainders a, b and c respectively. ; Required number=(LCM of x, y and z) - k
5. Find the LEAST NUMBER which when Required number
divided by x, y and z leaves the same = (LCM of x, y and z) + r
remainder ‘r’ in each case.
6. Find the GREATEST NUMBER that Required number
will exactly divide x, y and z leaves same = HCF of (x-r), (y-r) and (z-r)
remainder ‘r’ in each case.
7. HCF of fractions $%& # '()*)+
HCF of fractions =
,%- # .( / *)+
8. LCM of fractions ,%- # '()*)+
LCM of fractions =
$%& # .( / *)+
9. HCF of decimal numbers Step 1. Find HCF of given numbers without
decimals
Step 2. In the HCF make decimal point from
right to left according to the maximum
decimal places among the given numbers.
10. LCM of decimal numbers Step 1. Find LCM of given numbers without
decimals
Step 2. In the LCM make decimal point from
right to left according to the maximum
decimal places among the given numbers.

www.sakshieducation.com
www.sakshieducation.com
Quantitative Aptitude

EXERCISE
3 4 2 2 3
1. Find the LCM of 2 ×3 × 5 , 2 × 3 × 7, (a) 3 (b) 7
52 ×72. (c) 1 (d) 0
(a) 23×34 ×52×72 (b) 23×52 ×72 13. Find the greatest number which will divide
(c) 34 × 52 ×72 (d) 23 ×72×5 321,428 and 535 exactly.
2. Find the HCF of 22×33×52 , 23×32×5, 52×7. (a) 105 (b) 107
(a) 5 (b)23×33×5 (c) 109 (d) 102
(c) 23 ×33×7 (d) 32 ×52×7 14. Find the greatest number that will divide
3. Find the LCM and HCF of 0.25, 0.5, 0.75. 640,710 and 1526 so as to leave 11, 7 and
(a) 1.5, 0.25 (b) 2, 1 9 as remainders respectively.
(c) 1.5, 0.5 (d) 3, 1 (a) 36 (b) 37
4. Find the HCF of 120, 150,180. (c) 42 (d) 29
(a) 30 (b) 60 15. Find the least number which when divided
(c) 50 (d) 10 by 16,18 and 20 leaves a remainder 4 in
   each case, but is completely divisible by 7.
5. Find the LCM of , ,
  
(a) 465 (b) 3234
(a) 20 (b) 24 (c) (d) (c) 2884 (d) 3234
 

6. Find the HCF of and 16. The least number which when divided by
 
(a) 3/2 (b) 1/2 4, 6, 8, 12 and 16 leaves a remainder of 2
(c) 1 (d) 3 in each case is:
7. Find out the LCM of 45, 4-81, 412 and 47. (a) 20 (b) 43
(a) 45 (b) 4-81 (c) 50 (d) 59
(c) 412 (d) 47 17. Find the least number which when
8. Find the LCM of 5/2,8/9,11/14. increased by 3 is exactly divisible by
(a) 290 (b) 380 10,12,14 and 16.
(c) 420 (d) 440 (a) 1680 (b) 1677
9. The HCF and LCM of two numbers are 18 (c) 1697 (d) 1670
and 3780 respectively. If one of them is 18. Find the least number which when
540, then the second one is: decreased by 4 is exactly divisible by
(a) 146 (b) 126 9,12,15 and 18.
(c) 118 (d) 117 (a) 188 (b) 182
10. The HCF of two numbers is 8. Which one (c) 186 (d) 184
of the following can never be their LCM? 19. Find the smallest four digit number that is
(a) 32 (b) 24 exactly divisible by 8,10 and 12.
(c) 48 (d) 60 (a) 1080 (b) 1100
11. LCM of two numbers is 12 times of their (c) 1050 (d) 1120
HCF. Sum of LCM and HCF is 195. If one 20. Five bells bE.g.in to toll together and toll
of them is 60. Find the other. at intervals of 24, 40, 64, 72 and 120 s.
(a) 48 (b) 45 After what interval of time will they toll
(c) 52 (d) 36 again together ?
12. The HCF and LCM of a pair of numbers (a) 42min (b) 36min
are (c) 48min (d) 54min
12 and 926 respectively. How many such 21. Find the least number which is exactly
distinct pairs are possible? divisible by 12,15,20 and 27.
(a) 650 (b) 520
www.sakshieducation.com
www.sakshieducation.com
Quantitative Aptitude

(c) 600 (d) 540 (c) 5 (d) 4


22. A number when divided by 225 gives a 28. The LCM of two numbers is 280 and the
remainder of 32. What will be the ratio of the numbers is 7 : 8. Find the
remainder when the same number is numbers.
divided by 15 ? (a) 70 and 48 (b) 42 and 48
(a) 4 (b) 2 (c) 35 and 40 (d) 28 and 32
(c) 3 (d) 1 29. A florist has 200 roses and 180 jasmines
23. Five bells bE.g.in to toil together and toll with him. He was asked to make garlands
respectively at intervals of 6, 7, 8, 9 and 12 of flowers with only roses or only jasmines
sec. How many times they will toll each containing the same number of
together in one hour, excluding the one at flowers. What will be the largest number
the start? of flowers, he can join together without
(a) 3 (b) 5 leaving a single flower?
(c) 7 (d) 9 (a) 16 (b) 17
24. What is the smallest whole number that is (c) 20 (d) 19
  30. What is the largest number which when
exactly divisible by 1 ,2 and 3 ?
 
divides 1475, 3155 and 5255 leaves the
(a) 264 (b) 130
same remainder in each case?
(c) 138 (d) 124
(a) 220 (b) 420
25. The least positive intE.g.er which leaves a
(c) 350 (d) 540
remainder 2, when divided by each of the
31. Find the side of the largest possible square
numbers 4,6,8,12 and 16.
slabs which can be paved on the floor of a
(a) 46 (b) 48
room 2m 50 cm long and 1 m 50 cm
(c) 50 (d) 52
broad. Also find the number of such slabs
26. HCF of two numbers is 12 and their
to pave the floor.
product is 3600. How many such pairs of
(a) 40,18 (b) 30,15
numbers can be formed?
(c) 50,15 (d) 20,25
(a) 0 (b) 1
32. How many numbers less than 10,000 are
(c) 2 (d) 4
there which are divisible by 21,35 and 63?
27. The sum of two numbers is 135 and their
(a) 33 (b) 32
HCF is 9. How many such pairs of
(c) 38 (d) 31
numbers can be formed?
(a) 6 (b) 2

Answer Key
1 a 5 b 9 b 13 b 17 b 21 d 25 c 29 c
2 a 6 b 10 d 14 b 18 d 22 b 26 b 30 b
3 a 7 c 11 b 15 c 19 a 23 c 27 d 31 c
4 a 8 d 12 d 16 c 20 c 24 a 28 c 32 d

www.sakshieducation.com
www.sakshieducation.com
Quantitative Aptitude

SOLUTIONS
1. LCM of given numbers = 23×34×52 ×72 7. Clearly, LCM = 412
(Take the greatest power of each term) ,%- # ,, 
8. = = 440
$%& # ,, 
2. HCF = 5 (Take the least powers of
9. Sol: (b) We know that,
common terms).
_`a b`c " 9 =
3. (a) LCM of 0.25, 0.5 and 0.75
' #' " $ =
The given numbers can be written as of 
0.25, 0.50 and 0.75 "=

Now ignoring the decimals we find LCM = " %
of 25, 50, and 75 Another number is 126.
5 25, 50, 75 10. Since HCF of any two numbers is also a
5 5, 10, 15 factor of their LCM, 60 can never be their
1, 2, 3 LCM
∴ LCM = 1 × 2 × 3 × 5 × 5 since HCF 8 is not a factor of 60.
= 25 × 6 = 150 11. Let the HCF be x
LCM = 150. Similarly we will find out LCM = 9
the HCF for 25, 50 and 75. 9 & 9 " 
Since given numbers are not high, we can 9 " 
follow Prime factorization method to get the 9" 
HCF easily ; HCF = 15 and LCM = (12)(15) = 180
25 = 52  
Other number = " $

50 = 2 × 52
12. HCF = 12 and LCM = 926. If the numbers
75 = 3 × 52.
be of the form 12a and 12b; then LCM =
As discussed before HCF = 52 = 25.
12ab, i.e., LCM is always divisible by
Now we got LCM = 150 and HCF = 25,
HCF Clearly in this question 926 is not
now after putting decimal places as per
divisible by 12, so no such pair exists.
given in the question the LCM = 1.5, and
13. Required number
HCF = 0.25.
= HCF of 321, 428 and 535 = 107
4. (a) since given numbers are not high, we
14. HCF [(640 – 11), (710 - 7), (1526 - 9)] =
can follow Prime factorization
HCF [629, 703, 1517] = 37.
method to get the HCF easily
3 15. LCM (16, 18, 20) = 720
120 = 2 x 3 x 5
The number required is off the 720 k + 4,
150 = 2 x 3 x 52
where k is a natural number. In order to
180 = 22 x 32 x 5
make it divisible by 7,we put k = 4. Hence
In Prime factorization method after
the number is
converting the numbers into product of
# $ & $ " ''$.
prime factors, take the common factors
16. The number = LCM (4, 6, 8, 12,16) + 2 =
from all the numbers of the least powers.
50
Therefore HCF is   "
,%- # 0'()*)+
17. Required number
5. LCM of fraction = " /def  , , $, %0 
$%& # 1( / *)+

=
,%-2,,4
=24 " %'  " %##
$%&2,,4
18. Required number
 $%& #  . 
6. HCF of , =
  ,%- #   . 
=
 " /def  , , , '0 & $

www.sakshieducation.com
www.sakshieducation.com
Quantitative Aptitude

" ' & $ " '$ 28. Let the two numbers be #9 and '9 and
19. The smallest four digit number exactly LCM is %9.
divisible by 8, 10 and 12 should also be It is given that LCM = 280
divisible by the LCM of 8,10 and 12. ie, %9 " ' and 9 " 
LCM of 8, 10 and 12= 120 ie, numbers are 35 and 40.
So, the required number = 1080 29. HCF (200, 180) = 20
20. Required time interval 30. Take the relative differences of the given
= LCM of 24, 40, 64, 72 and 120 s numbers.
= 2880 seconds (3155-1475), (5255 - 3155), (5255 - 1475)
= 48 min = 1680,2100,3780
21. Required number HCF (1680,2100, 3780) = 420
= LCM of 12, 15, 20 and 27 = 540 31. HCF (250, 150) = 50 cm
22. M-I: Let the number be 225 + 32 = 257,  
; The number of slabs = = 15

and after dividing 257 with 15 we will get
32. LCM (21, 35, 63) = 315
the remainder as 2.
The numbers less than 10,000 which are
M-II: The remainder when 32 is divided ,
by 15 is 2 divisible by 315 are given by ! i.e. the
 
23. The bells would toll together at LCM 6, 7, integral part when 10,000 is divided by 315
8, 9, 12, which is 504. is 31.
Number of times, they toll together The required answer is 31

" " #

  
24. Required number = LCM[ , , ]=264
  
25. The LCM of 4, 6, 8, 12 and 16 is 48.
So, required number = 48 + 2 = 50
26. Let the two numbers be 9 and =.
9 = " %
9= " 
Possible values of x and y are (1,25) (5,5).
But (5,5) are not co primes.
; Only one pair of numbers can be
formed.

27. The two numbers are always the multiples


of the HCF
∴ Let the two numbers be 9x and 9y.
9 & = "  : 9 & = " 
Now, the possible values of 9 and = are
(1,14), (2,13) (3,12), (4,11), (5,10), (6,9),
(7,8).
Now, consider only the co prime pairs. These
are (1 ,14), (2,13), (4,11), (7,8).
∴ 4 pairs of numbers can be formed whose
sum is 135 and HCF is 9
www.sakshieducation.com
www.sakshieducation.com
Quantitative Aptitude

6. UNITARY METHOD
Unitary method is the basic area of arithmetic.
Direct Proportion
Two quantities are said to be directly proportional, if on the increase in one the other increases
proportionally or on the decrease in one the other decreases proportionally,
e.g., More the numbers of articles, More is the cost.
More the number of workers, More is the work done.
Less the number of articles, Less is the cost.
Less the number of workers, Less is the work done.
Indirect Proportion
Two quantities are said to be indirectly proportional, if on the increase in one the other decreases
proportionally or on the decrease in one the other increases proportionally.
e.g., More the number of workers, less is the number of days required to finish a work. More the
speed, less is the time taken to cover a certain distance.
Less the number of workers, more is the number of days required to finish a work. Less the speed,
more is the time taken to cover a certain distance.
Chain Rule
When a series of variables are connected with one another, that we know how much of the first kind
is equivalent to a given quantity of second, how much of the second is equivalent to a given quantity
of the third and so on. The rule by which we can find how much of the last kind is equivalent to a
given quantity of the first kind is called the Chain Rule.
Example 1: If 12 apples cost Rs.216, what is the cost of 3 dozen apples ?
Solution: Let the required cost be Rs. 9. Also, 3 dozen apples = 36 apples
more apples- more cost (Direct Proportion);

Apples Cost
12 216
36 X
 
12×9 = 36×216: 9 = = Rs.648


Example 2: A man completes of a job in 18 days. At this rate, how many more days will it take

him to finish the job?

Solution: Let the number of days still required to finish the job be x days. Remaining Work = (1- )


=

Less work- Less hours (Direct Proportion).
Work Time in days

18


x

   
O x × = ×18 O x = = 12 days
  
Example 3: A wheel that has 5 cogs is meshed with a larger wheel of 15 cogs. When the smaller
wheel has made 27 revolutions, find the number of revolution made by the larger wheel.
Solution:: Let the required number of revolutions made by larger wheel be x.
More cogs-Less revolutions (Indirect Proportion)
Cogs Revolutions made
5 27
15 x

O 15×9 = 5×27 O x= = 9 revolutions


www.sakshieducation.com
www.sakshieducation.com
Quantitative Aptitude

Example 4: If the wages for 8 men for 24 days be Rs.3040, then find the wages of 6 men for 18
days.
Solution: . Let the required wages be Rs. x.
Less men, Less wages (Direct Proportion)
Less days, Less wages (Direct Proportion)
Men Days Wages in Rs.
8 24 3040
6 18 x
 
O x = × ×3040 = Rs.1710
 
Example 5: If a man walks a certain distance in 20 days when he rests 10 h each day, how long will
it take him to walk three times as far, if he walks twice as fast and rests 12 h each day?
Solution: In the first case the man walks (24- 10)hr = 14h each day. Let the distance travelled be d
and the speed be s.
In the second case the man walks (24-12) hr = 12 hr each day. The distance travelled is 3d and the
speed is 2s. Let the required number of days be x.
More distance-More days (Direct Proportion)
More speed-Less days (Indirect Proportion)
Less hours-More days (Indirect Proportion)
Distance Speed Hours Days
d s 14 20
3d 2s 12 x
. + 
X=  = 35 days
. + 
Example 6: A contractor undertakes to complete a road 420 m long in 140 days and employ 28 men
for the work. After 70 days he finds that only 140 m of the road is made. How many men should the
contractor employ more so that the work may be completed in time?
Solution: In the second case in (140-70) days i.e., 70 days the length of the road to be construct
(420-140) m i.e., 280 m.
Let the number of men employed be x.
More length of road-More men (Direct Proportion)
Days Length of road in meters Men
70 140 28
70 280 x

X= ' = 56 men

Additional men required= 56 - 28 = 28 men

EXERCISE
1. Cost of 24 pens is Rs. 96. Find the cost of ૙
(a) 24 C (b) 220C
16 such pens, 

(a) Rs. 66 (b) Rs. 64 (c) 22 C (d) 240C

(c) Rs. 62 (d) Rs. 68 4. Cost of 8 dozen bananas is Rs.180. How
2. A bus travels 240 km in 3 h. How long many bananas can be purchased for Rs.
will it take to travel 360 km? 30?
(a) 5 h (b) 4 h (a) 16 bananas (b) 24 bananas
(c) 4 g (d) 5 g (c) 14 bananas (d) 22 bananas
 
3. The temperature dropped 18°C in the last 5. 20 men can reap a field in 20 days. When
24 days. If the rate of temperature drop should 5 men leave the work, if the whole
remains constant, then how many degrees field is to be reaped in 24 days after they
will the temperature drop in the next 32 leave the work?
days? (a) 2 days (b) 4 days

www.sakshieducation.com
www.sakshieducation.com
Quantitative Aptitude

(c) 3 days (d) 5 days 14. The work done by (m + 3) men in (m - 2)


6. A rope makes 125 rounds of a cylinder days and work done by (m - 2) men in (m
with base radius 15 cm. How many times + 5) days are in the ratio 4 :5 . Find the
can it go round a cylinder with base radius value of m?
25 cm? (a) 5 (b) 6 (c) 4 (d)8
(a) 100 (b) 75 (c) 80 (d) 65 15. If 36 men take 9 days to earn rs 9000. How
7. 6 men finish one-fourth work in 2 days. many men will earn rs 6000 in 6 days?
The number of additional men required for (a) 42 (b) 36 (c) 45 (d) 39
finishing the same work in 2 days is 16. A contractor undertakes to complete the
(a) 18 men (b) 24 men repairing of a rail track 480 m long in 60
(c) 28 men (d) 14 men days and employs 60 men for the work.
8. A certain number of men complete a piece After 30 days he finds that only 160 m of
of work in 45 days. If there were 5 men the rail track is repaired. How many more
more, the work could be finished in 9 men Should he employ to complete the
days less. How many men were originally work in time?
there? (a) 60 men (b) 64 men
(a) 30 (b) 15 (c) 68 men (d) 62 men
(c) 25 (d) 20 17. If 8 men or 12 women can do a piece of
9. 10 workers can make 15 boxes in 6 days, work in 52 days. In how many days can 12
how many boxes will 12 workers make in men and 8 women do the same piece of
3 days. work?
(a) 10 (b) 9 (a) 28 days (b) 24 days
(c) 6 (d) 8 (c) 25 days (d) 30 days
10. If 25 binders bind 25 books in 25 days. 18. A camp of 3000 soldiers has provision for
How many binders can bind 10 books in 60 days. After 15 days how many soldiers
10 days? must leave so that the provision may last
(a) 25 (b) 10 for 75 days?
(c) 15 (d) 20 (a) 1200 (b) 1400
11. If 8 men working 9 h a day can reap a field (c) 1300 (d) 1500
in 24 days, in how many days will 12 men
reap the field, working 6 h a day?
(a) 24 days (b)20 days
(c) 28 days (d) 16 days
12. If 5 men take 21 days of 8 h each to do a ANSWER KEY
piece of work. How many days of 6 h 1 b 5 a 9 b 13 a 17 b
each would 14 women take, if 2 women 2 c 6 b 10 a 14 a 18 a
do as much work as a man? 3 d 7 a 11 a 15 b
(a) 20 days (b) 16 days 4 a 8 d 12 a 16 a
(c) 18 days (d) 22 days
13. If the cost of ‘m’ articles is - n the cost of
P articles is
 
(a) Rs. (b) Rs.


(c) Rs. DL. (d) Rs.

SOLUTIONS
1. Let the required cost be Rs. x. Then, 
9 " % " hF. %$

Less pens, Less cost (Direct
2. Let the required number of hours be x.
Proportion)
More distance, More hours (Direct
Pens Cost in Rs.
Proportion)
24 96
Distance (in km) Hours
16 9
240 3
www.sakshieducation.com
www.sakshieducation.com
Quantitative Aptitude

360 9 Then, /9 & 0 men can finish the work in


  (45 - 9) days ie, 36 days
: 9" " " $ g
  
More days, Less men (Inverse Proportion)
3. Let the required drop in temperature be x.
Days Men
More days, More drop in temperature
36 /9 & 0
(Direct Proportion)
45 9
Days Drop in temperature  
24 18 O 9" O 9 "  L7.

32 9 9. Let the number of boxes made be x.
 More workers, More boxes (Direct
O 9" ' " $°`
$ Proportion)
4. 8 dozens = 8×12 = 96 bananas Less days, Less boxes (Direct Proportion)
Let the required number of bananas that Workers Days Boxes
can be bought be x. 10 6 15
Less price, Less bananas (Direct 12 3 9
proportion)  
O 9 "  " *K97F
Cost in Rs. Bananas  
180 96 10. Let the number of binders required be x.
30 9 Less books, Less binders (Direct
Variation)
O 9" % " % *).).)F Less days, More binders (Indirect
'
5. 20 men can reap a field in 20 days. Variation)
⇒ 1 man can reap that field in (20 × 20) Books Days Binders
days = 400 days 25 25 25
Let 5 men leave the field after x days, so that 10 10 9
 
the remaining 15 men can complete the work O 9 "  "
 
field  *3.276F
in 24 days. 11. Let the required number of days be x.
209+ 15×24 = 400 More men, Less days (Indirect Proportion)
O 9 "  2)=F Less working hours, More days (Indirect
; 5 men must leave the work after 2 days. Proportion)
6. Let the required number of rounds be x. Men Hours Days
More radius, Less rounds(Inverse 8 9 24
Proportion) 12 6 x
Radius in cm Round  
O 9" $ " $2)=F
15 125  

25 9 12. 2 women = 1 man


  14 women = 7 men
: 9" " # 6Ki.2F More men, Less days (Indirect
 Proportion)
7. Time taken by 6 men to finish the whole
work = 8 days Less working hours, More days (Indirect
Let the number of men required be x. Proportion)
Less days, More men (Inverse Proportion) Men Hours Days
Days Men 5 8 21
8 6 7 6 9
 
2 9 O 9"  "  2)=F
 

O 9 " " $ L7. 13. Let the required cost be rs x.

The additional men required = 24-6 men= More articles, More cost (Direct
18 men Proportion)
8. Let there be originally x men. Articles Cost in Rs.
m n
p x
www.sakshieducation.com
www.sakshieducation.com
Quantitative Aptitude


:x=

14. Work done by (m+2) men in (m-2) days
= (m+3)(m-2)
Work done by (m-2) men in (m+5)
days
= (m- 2)(m+5)
5 5 
O = => m =5
5  5 
15. Let the required number of men be x.
Less days, More men (Indirect Proportion)
Less earnings, Less men (Direct
Proportion)
Days Earnings in Rs. Men
9 9000 36
6 6000 9
 
O 9 " % " % L7.
 
16. In the second case in (60 - 30) days ie, 30
days the length of the rail track to be
repaired (480 - 160) m, i.e. 320 m. More
length of rail track, More men (Direct
Proportion)
Men Days Length of rail track
in meter
60 30 160
9 30 320

O 9 " % "  L7.

Additional men required= (120 - 60) men
= 60 men
6
17. 8men = 12womenO 1 man =

6
12 men + 8 women = 12 ( 0+8w = 26women

Let the required number of days be x.
More women, Less days (indirect Proportion)
Women Days
12 52
26 9

O 9 " " $ 2)=F

18. After 15 days, 3000 soldiers have
provision for 45 days.
Let the number of soldiers be x.
More days, Less soldiers (Indirect
Proportion)
Days Soldiers
45 3000
75 9

O 9 " " ' FKj2376F

Required Soldiers = 3000- 1800 = 1200

www.sakshieducation.com
www.sakshieducation.com
Quantitative Aptitude

7. Problems on Ages

Ages Introduction:
• If the present age of A is x years, then his/her age m years ago was (x-m) years and his/her age
after n years will be (x + n) years.
• If the age of a person n1 years ago was x years, then after n2 years his age will be (x+n1+n2)
years.
• If the age of a person after n1 years will be x years, then his age n2 years ago was (x-n1-n2)
years.
• If the sum of the ages of n persons is S years, then the sum of their ages m years ago was (S-mn)
years
• If the sum of the ages of n persons is S years, then the sum of their ages after m years will be (S
+ mn) years
EXERCISE
1. Hitesh is 40 years old and Ronnie is 60 7. The ratio of the ages of a man and his wife
years old. How many years ago was the is 4 : 3. After 4 years, this ratio will be 9
ratio of their ages 3:5? :7. If at the time of marriage, the ratio was
(a) 2 years (b) 10 years 5:3, then how many years ago were they
(c) 30 years (d) 47 years married?
2. The ratio of the father’s age to his son’s (a) 10 years (b) 25 years
age is 7 : 3.The product of their ages is (c) 12 years (d) 18 years
756. The ratio of their ages after 6 years 8. The ratio between the school ages of
will be: Neelam and Shaan is 5 : 6 respectively. If
(a) 6 : 7 (b) 2 : 1 the ratio between the one-third age of
(c) 10:9 (d) 3 : 2 Neelam and half of Shaan’s age is 5 : 9,
3. The present ages of three persons are in then what is the school age of Shaan?
proportions 4 :7 : 9. Eight years ago, the (a) 28 years (b) 23 years
sum of their ages was 56. Find their (c) 39 years (d) cannot be
present ages (in years). determined
(a) 10, 12, 24 (b) 16, 28, 36 9. The ratio between the present ages of A
(c) 20, 35, 45 (d) 20, 25, 35 and B is 5:3 respectively. The ratio
4. The ratio of the present ages of two between A’s age 4 years ago and B’s age 4
brothers is 1 :2 and 5 years back, the ratio years hence is 1 : 1. What is the ratio
was l : 3. What will be the ratio of their between A’s age 4 years hence and B's age
ages after 5 years? 4 years ago?
(a) 3 : 4 (b) 3 : 2 (a) 3 : 2 (b) 5 : 2
(c) 3 : 5 (d) 5 : 6 (c) 3 : 1 (d) 6: 1
5. The total of the ages of Jayant, Prem and 10. Ten years ago, A was half of B in age. If
Saransh is 93 years. Ten years ago, the the ratio of their present ages is 3 : 4, what
ratio of their ages was 2 : 3 :4. What is the will be the total of their present ages?
present age of Saransh? (a) 25 years (b) 32 years
(a) 44 years (b) 36 years (c) 45 years (d) None of these
(c) 33 years (d) 38 years 11. A is two years older than B who is twice as
6. Six years ago, the ratio of the ages of old as C. If the total of the ages of A, B and
Kunal and Sagar was 6 : 5. Four years C be 27, then how old is B?
hence, the ratio of their ages will be 11 : (a) 6 years (b) 9 years
10. What is Sagar’s age at present? (c) 13 years (d) 10 years
(a) 16 years (b) 19 years 12. A man is 24 years older than his son. In
(c) 22 years (d) 25 years two years, his age will be twice the age of
his son. The present age of the son is:
www.sakshieducation.com
www.sakshieducation.com
Quantitative Aptitude

(a) 24 years (b) 26 years (c) 11:2 (d) 13 :4


(c) 28 years (d) 22 years 17. Four years ago, the father’s age was three
13. Eighteen years ago, a father was three times the age of his son. The total of the
times as old as his son. Now the father is ages of the father and the son after four
only twice as old as his son. Then the sum years, will be 64 years. What is the
of the present ages of the son and the father’s age at present?
father is: (a) 35 years (b) 40 years
(a) 102 years (b) 76 years (c) 46 years (d) Data inadequate
(c) 105 years (d) 108 years 18. One year ago, Promila was four times as
14. A person’s present age is two-fifth of the old as her daughter Sakshi. Six years
age of his mother. After 8 years, he will be hence, Promila’s age will exceed her
one-half of the age of his mother. How old daughter’s age by 9 years. The ratio of the
is the mother at present? present ages of Promila and her daughter
(a) 44 years (b) 42 years is:
(c) 40 years (d) 55 years (a) 8:3 (b) 11:5
15. Tanya’s grandfather was 8 times older to (c) 12:5 (d) 13:4
her 16 years ago. He would be 3 times of 19. The sum of the present ages of a father and
her age 8 years from now. Eight years ago, his son is 60 years. Six years ago, father’s
what was the ratio of Tanya’s age to that age was five times the age of the son.
of her grandfather? After 6 years, son’s age will be:
(a) 6:1 (b) 1:5 (a) 15 years (b) 16 years
(c) 7:3 (d) 11:53 (c) 19 years (d) 20 years
16. The age of father 10 years ago was thrice 20. The total age of A and B is 12 years more
the age of his son. Ten years hence, than the total age of B and C. C is how
father’s age will be twice that of his son. many years younger than A?
The ratio of their present ages is: (a) 12 (b) 26
(a) 8 : 3 (b) 7 : 3 (c) C is elder than A (d)None of these

ANSWER KEY
1 b 6 a 11 d 16 b
2 b 7 c 12 d 17 b
3 b 8 d 13 d 18 d
4 c 9 c 14 c 19 d
5 d 10 d 15 d 20 a

SOLUTIONS
1. Suppose, the ratio was 3 :5, 9 years ago.
 
Then, =
 
k /$  90 " /%  90 l 9 "  k 9 " =7)6F
2. Let the present ages of the father and son be 79 and 39 years respectively.
Then, 79 39 = 756
l 219 = 756l 9 = 36 l 9 = 6.
Required ratio = (79 + 6): (39 + 6)
= 48 : 24 = 2 : 1.
3. Let their present ages be 49, 79 and 99 years respectively.
Then, (49 - 8) + (79 - 8) + (99 - 8) = 56 k  9 " ' k 9 " $.
Their present ages are 16 years, 28 years and 36 years respectively.
4. Let the present ages of the two brothers be x years and 2x years respectively.

Then, =
 

www.sakshieducation.com
www.sakshieducation.com
Quantitative Aptitude

k3 (9 - 5) = (9 - 5) k x = 10.
Required ratio = (9 + 5): (29 + 5) = 15 : 25
= 3 : 5.
5. Let the ages of Jayant, Prem and Saransh 10 years ago be 2 x, 3 x and 4x years respectively.
Then, (2x + 10) + (3x + 10) + (4x + 10) = 93 l 9x = 63=> x = 7.
Saransh’s present age" /$9 & 0 " ' y
6. Let the ages of Kunal and Sagar 6 years ago be %9 and 9 years respectively.
 
Then, =
  
= 10(%9 & 0= 11(9 +10)
= 9 = 10 l 9 " 
; Sagar’s present age = (9 + 6) = 16 y
7. Let the present ages of the man and his wife be 49 and 39 years respectively.
 
Then, =
 
k #/$9 & $0 " / 9 & $0
k 9 " '.
So, their present ages are 32 years and 24 years respectively.
Suppose they were married z years ago.
7 
Then, =
7 
k /   ]0 " /$  ]0
k ] " $ k ] "  =7)6F
8. Let the school ages of Neelam and Shaan be 59 and 69 years respectively.

 
Then, ૜
૚ = kG 9H
  


" P %9Q l  " .

Thus, Shaan’s age cannot be determined.
9. Let the present ages of A and B be 59 and 39 years respectively.

Then, =

k 9  $ " 9 & $
l 9 " ' k 9 " $ .
Required ratio = (59 + 4): (39 - 4)
= 24 : 8 = 3: 1.
10. Let the ages of A and B 10 years ago be 9 and 29 years respectively.
  
Then =
  
k $/9 & 0 " /9 & 0
l 9 " k9".
Sum of their present ages
= (9 + 10) + (29 +10) = (39 +20) = 35 years.
11. Let C’ s age be x years. Then, B’s age = 29 years. A’s age = (29 + 2) years.
(29 + 2) + 29 + x = 27 k 59 = 25 k x = 5.
Hence, B’s age = 29 =10 years.
12. Let the son’s present age be 9 years. Then, man’s present age = (9 + 24) years.
(9 + 24) + 2 = 2 (9 + 2)
k 9 & % " 9 & $ k 9 " .
13. Let the present ages of the father and son be 29 and 9 years respectively.
Then, (29 - 18) = 3 (9 - 18) k 9 = 36.
Required sum = (29 + 9)= 39 = 108 years.

14. Let the mother’s present age be 9 years. Then, the person’s present age = 9 years.


www.sakshieducation.com
www.sakshieducation.com
Quantitative Aptitude


P 9 & 'Q " /9 & '0 k /9 & $ 0 " /9 & '0 k 9 " $ .
 
15. 16 years ago, let T = 9 years and G = 89 years.
After 8 years from now, T = (9 + 16 + 8) years and G = (89 + 16 + 8) years.
89 + 24 = 3 (9 + 24) k 5 9 = 48.
8 years ago,
૝ૡ
8   
" " ૞
૝ૡ " " .
9     

16. Let the ages of father and son 10 years ago be 39 and 9 years respectively.
Then, (39 + 10) + 10 = 2 [(9 + 10) + 10] l 3 9 + 20 = 2 9 + 40
k 9 "  .
Required ratio
= (39 + 10) : (9 + 10) = 70 : 30 = 7 : 3.
17. Let the ages of father and son 4 years ago be 39 and 9 years respectively.
Then, [(39 + 4) + 4] + [(9 +4)+ 4]= 64
k 49 = 48 k 9 = 12.
Father’s present age = 9 & $
"  & $ " $ m
18. Let the ages of Promila and Sakshi 1 year ago be 49 and 9 years respectively.
Then, [(49 + 1) + 6] - [(9 + 1) + 6] =
9 k 39 = 9k 9 = 3.
Required ratio = (49 + 1): (9+ 1) = 13 :4.
19. Let the present ages of son and father be 9 and (60 - 9) years respectively.
Then, (60 - 9) - 6 = 5 (9 - 6) k 54- 9 = 59- 30 k 69= 84 k 9 = 14.
Son’s age after 6 years = (9 + 6)=20 years.
20. (A + B ) - (B + C) = 12 k A - C = 12.

www.sakshieducation.com
www.sakshieducation.com
Quantitative Aptitude

8.AVERAGES
Average
The average of a given number of quantities of the same kind is expressed as
:' # *;( ' */*/(+
Average =
<=5>?@ AB C=D EFEF?G
Average is also called the Arithmetic Mean.
Sum of the quantities = Average x Number of quantities
:' # *;( ' */*/(+
Number of quantities =
HI()J(
• If all the given quantities have the same value, then the number itself is the average.
• If all the given quantities are not the same, then the average of the given quantities is always
greater than the smallest number and less than the largest number
• If each of the given quantities is increased by a constant p, then their average is also increased by P.
• If each of the given quantities is decreased by a constant p, then their average is also decreased by
P.
• If each of the given quantities is multiplied by a constant p, then their average is also multiplied by
P.
• Whenever the given quantities form an arithmetic sequence and if the given quantities have odd
terms, then the average is the middle term in the sequence and if the given quantities have even
terms, then the average of the sequence is the average of the middle two terms.
•In order to calculate the weighted average of a set of numbers, multiply each number in the set
by the number of times it appears, add all the products and divide by the total number of
numbers in the set.
• If the speed of an object from A to B is x km/h and from B to A is y km/h, then the average speed

during the whole journey is km/h

• If the average of N1 quantities is 9 and N2 quantities is y then the average of total (N1 + N2)
0 0 
quantities is given by ૚ ૛
0૚ 0૛
Example 1: What is the average of first five even numbers
Solution :. The first five numbers are 2, 4, 6, 8, 10
  
Average = = =6
 
Example 2: The average of five consecutive even numbers is 50. What is the largest of these
numbers?
Solution:: Let the numbers be x-4, x -2, x, x + 2, x + 4.
:' # *;( ' */*/(+
Average =
<=5>?@ AB C=D EFEF?G

= = 50


= 50

9 " 
∴Largest Number = 9 & $ "  & $ " $
Example 3: Average weight of 32 students of a class is 30.5 kg. If weight of a teacher is also
included then average weight is increased by 500 g. What is the weight of the teacher?
Solution : Total weight of 32 students = 30.5×32 = 976 kg
Average weight of (32 students + 1 teacher) = (30.5 + 0.5) = 31 kg
:. Total weight of (32 students + 1 teacher) = 31 × 33 = 1023 kg
Weight of teacher = (1023 - 976) kg = 47 kg
Example 4: The average salary per head of all the employees of an institution is Rs.60. The average
salary per head of 12 officers is Rs.400 and average salary per head of the rest is Rs.56. Find the
total number of employees in the institution.
Solution: Let the total number of employees be x.

www.sakshieducation.com
www.sakshieducation.com
Quantitative Aptitude

8*K +K) # KK (K((+


Then 60 =

      
60 =
L
O % 9 "  $ & /9  0 % " $' & %9  %#
O % 9 – %9 " $'  %#
O $9 " $ ' O 9 " 
Hence, the total number of employees is 1032.
Example 5: If the average of p and q is 58 and the average of q and s is 64, what is the value of
s-p?

solution : Given = 58 O p+q = 116 …(i)

+
Also = 64 O q+s = 128 …(ii)

Subtracting Eq. (i) from (ii), we get
(q + s) - (p + q) =128 - 116 => s - p = 12
Example 6: 12 men went to a restaurant. 11 of them spent Rs.5 each and the 12th person spent
Rs.11 more than the average expenditure of all. Find the total money spent by them?
Solution: Let the average money spent by the 12 men = Rs. x
Money spent by the 12th man = Rs. (x + 11)
Money spent by the other 11 men = Rs. (11×5) = Rs. 55
Total money spent by 12 men = Rs. (55+x + 11) = Rs. (x + 66)

;x= : 129 = 9+66 O 119 = 66

O 9 =Rs.6
Total money spent by 12 men =6×12 = Rs. 72.
EXERCISE
1. Find the average of the first 100 natural (c) 38 (d) 27
numbers 6. The average marks obtained by 80
(a) 50.5 (b) 52 students in a certain examination is 28. If
(c) 51.5 (d) 53 the average marks of the students who
2. Find the mean of the first 25 multiples of have passed is 40 and that of those who
10. have failed is 25, how many students
(a) 150 (b) 125 failed the examination ?
(c) 120 (d) 130 (a) 64 (b) 24
3. Find the average weight of 8 students 49 (c) 16 (d) 38
kg,47 kg, 46 kg, 42 kg, 39 kg, 48 kg, 50 kg 7. The average of 100 numbers is 50. It is
and 43 kg. found that while calculating the average ,
(a) 44 kg (b) 42.5 kg two numbers, namely 81 and 66 were
(c) 47 kg (d) 45.5 kg wrongly read as 18 and 6. The correct
4. The average weight of 20 students in a average is
class is decreased by 500 g, when one boy (a) 51.23 (b) 41.45
whose weight is 45 kg is replaced by a (c) 36.42 (d) 52.46
new boy. What is the weight of the new 8. The average age of 25 students in a class is
boy ? 10 yr. If the teacher’s age is included, the
(a) 35 kg (b) 32 kg average increase by 1 yr. The age of the
(c) 36 kg (d) 34 kg teacher is
5. A batsman has a certain average of runs (a) 33 yr (b) 42 yr
for (c) 36 yr (d) 45 yr
16 innings. In the 17th innings, he makes a 9. The average monthly salary of 11 workers
score of 85 runs , thereby increasing his and one officer in an organization is 600.
average by 3 . What is the average after When the officer whose salary was rs 1600
the 17th innings? per month retired, a new officer was
(a) 60 (b) 37 appointed and the average salary of the 12

www.sakshieducation.com
www.sakshieducation.com
Quantitative Aptitude

employees is Rs. 570 per month. The 17. Jittu scores 60, 80, 40, 50 and 90 percent
salary of the new officer is? marks in the subjects A, B, C, D and E
(a) Rs. 1260 (b) Rs. 1240 respectively. However, the weights
(c) Rs. 1220 (d) Rs. 1280 attached to those subjects are 5, 4, 3, 2
10. A person travels 120 km in 6 h, 130 km in and 1 respectively, Which of the
5 h and 200 km in 4 h. Find his average following represents the weighted average
speed during the whole journey of marks scored by Jittu in these five
(a) 36 km/h (b) 35 km/h subjects
(c) 30 km/h (d) 32 km/h (a) 75 (b) 78
11. The average age of students in section A of (c) 62 (d) 79
40 students is 10 yr and the average age of 18. The average of 10 numbers is 7. If each
students in section B of 30 students is 12 number is multiplied by 12, then the
yr. Find the average age of students in both average of new set of numbers is
sections taken together. (a) 72 (b) 86
(a) 10 yr (b) 11 yr (c) 79 (d) 84
(c) 10.86 yr (d) 11.32 yr 19. In Arun’s opinion his weight is greater
12. If average of a, b, c is m and )* & *+ & than 65 kg but less than 72 kg. His brother
+) " , then average of a2, b2, c2 is does not agree with Arun and he thinks
(a) m2 (b) 3 m2 that Arun’s weight is greater than 60 kg
2
(c) 6m (d) 9 m2 but less than 70 kg. His mother’s view is
13. The average weight of 8 persons increases that his weight cannot be greater than 68
by2.5 kg when a new person comes in kg. If all of them are correct in their
place of one of them weighing 65 kg. estimation, what is the average of
What might be the weight of the new different probable weights of Arun?
person? (a) 71 kg (b) 67 kg
(a) 82 kg (b) 85 kg (c) 73 kg (d) 58 kg
(c) 76.5 kg (d) 80 kg 20. Find the average weight of four containers,
14. The average marks fetched by Mohan in if it is known that the weight of the first
History, Geography, Science and container is 100 kg and the total of the
Mathematics is 10 more than the marks second, third and fourth containers’ weight
fetched in Mathematics . If he has got 110 is defined by R/90 " 9²  /$ /9²0! ,
marks aggregate in History and where 9 " .
Geography, what will be the aggregate (a) 650 kg (b) 880 kg
marks fetched in Science and (c) 760 kg (d) 460 kg
Mathematics? 21. 19 persons went to a hotel for a combined
(a) 90 (b) 70 dinner party. 13 of them spent rs.79 each
(c) 75 (d) can’t be determined on their dinner and the rest spent Rs4 more
15. In a journey of 160 km, a train covers the than the average expenditure of all the 19.
distance 120 km at a speed of 80 km/h What was the total money spent by them?
and the remaining distance at 40 km/h. (a) rs.1628.4 (b) rs.1536
The average speed of the train for the (c) rs.1492 (d) rs.1632
whole journey is: 22. One-half of a certain distance is covered at
(a) 50 km/h (b) 64 km/h 40 km/h, one-third of it at 80 km/h and the
(c) 68 km/h (d) 74 km/h rest at 120 km/h. Find the average speed
16. A man travels from destination A to B by for the whole journey.
car at an average speed of 48 km/h and 
(a) 51 km/h (b) 55 km/h
 
returns on his bike with an average speed  
of 16 km/h. Find his average speed for the (c) 52 km/h (d) 56 km/h
 
entire journey 23. What is the average of all number from 1to
(a) 36 km/h (b) 24 km/h 100 that end in 3?
(c) 32 km/h (d) 21 km/h (a) 49 (b) 46

www.sakshieducation.com
www.sakshieducation.com
Quantitative Aptitude

(c) 48 (d) 47 the youngest child, if they differ in age by


24. Average of two numbers is 14.5 and 2 yr ?
square root of their product is 10. What are (a) 12 yr (b) 9yr
the (c) 11 yr (d) 7 yr
numbers? 27. A Student on his birthday distributed on an
(a) 16 and 9 (b) 25 and 4 average 5 chocolates per student. If on the
(c) 4 and 16 (d) 25 and 9 arrival of the teacher and the headmaster to
25. The mean yearly salary of an employee of
whom the student gives 10 and 15
company was Rs. 20000. The mean yearly
salaries of male and female employees chocolates respectively, the average
were Rs. 20800 and Rs. 16800 chocolate distributed per head increase to
respectively. Find the ratio of males to 5.5, what is the strength of the class?
females employed by the company?
(a) 3 :2 (b) 4:1 (a) 28 (b) 30
(c) 2 :1 (d) 5 :3 (c) 32 (d) None of these
26. Twelve years ago, the average age of a
husband and a wife was 20 yr. The average
remains the same today, when they have
two children . What is the present age of

ANSWER KEY
1 a 7 a 13 b 19 b 25 b
2 d 8 c 14 d 20 a 26 d
3 d 9 b 15 b 21 b 27 a
4 a 10 c 16 b 22 b
5 b 11 c 17 c 23 c
6 a 12 b 18 d 24 b

SOLUTIONS

1. Sum of the first n natural numbers 5. Let the average score be x runs, then by
 given condition total score of 16 innings
"
 = 16x.
Sum of the first 100 natural numbers = 
  = (x + 3) => x = 34
"   
 ; Average after 17th innings
Average of first 100 natural numbers
= 34 + 3 = 37 .
 
" "  . 6. Let the number of students who passed
"9
2. Sum of the first 25 multiples of 10
Then$ 9 & /'  90  " ' 
= 10+ 20+ 30+ ...250
O $ 9  9 &  ' " ' '
= 10 [1 + 2 + 3 + .... +25]  
   O 9" " " %
= = =3250  
 
 Hence the number of students who failed
Required average = " " /'  %0 " %$

3. Average weight 7. Sum of the numbers
 "/   '  % & ' & %%0
"

 "  
= = 45.5 kg  
 :Correct average = "  .

4. Total decrease = (20× ) kg = 10kg 8. Sum of the ages of 25 students

Weight of new man = (45-10) kg = 35kg "  "  =7)6F
www.sakshieducation.com
www.sakshieducation.com Quantitative Aptitude

Let the age of the teacher = x yrs % % 


 " " " %
Then, (10+1) =  $ .  
 &
' $
:  & 9 "/ & 0/ & 0 " %$ nL/g
" % 
16. Average speed = ( 0km/hr
:9" %   
 
" % =7)6F =( 0km/hr
 
9. Let the average monthly salary of each = 24 km/hr
worker = Rs. x 17. Weighted average =
Total salary of 11 workers and 1 officer         
" 9& % 
   
Then, 600 = = = 62 marks
 
: 9 " %   % 18. Required Answer =  # " '$
" % 19. According to Arun, his weigh x kg is
Let the salary of the new officer be Rs. y. such that
 % I s I 72
Then, 570 =
 According to his brother,
: 9 & = " #  % I s I 70
: = " #   9 According to his mother, x v 68
: = " #   % " $ From (i), (ii) and (iii), we get
∴ Hence the salary of the new officer % I s v 68 i.e.,
= Rs. 1240. 9 " %%, %# K6 %'

10. Total distance travelled = 120 + 130 + Average = = 67 kg

200= 450 km 
20. Sum = 100+(100)3- (100)2 =
Total time of travel = 6 + 5 + 4 = 15 h 

Average speed =

= 30 km/ph 100+(100)2[1- ]


   = 100+10000× = 2600
11. Required average = = 10.86 
 
years Avg. = =650 kg

12. 21. Let the average expenditure of all be Rs.
(a + b + c)2 =a2 + b2 + c2 + 2 (ab + bc + ca) x, then by given condition
= a2 +b2 + c2 9 " 9 # & /9 & $0%
M 
But = m : a+b+c = 3m O 9 "  O 9 "
 
²૛ M૛ M ૛ 
; = = m2= 3m2 = 80.846
  
; Total money spent by them
13. By shortcut: The weight of the new
" ' . '$%
person
" hF.  % /)DD6K90
" ' .  & % " ' no
14. By given condition, 22. Let total journey = x km
$/+*)  9(J);  :M/( M(  -*;+ Total, time taken
  
 =( J & 0h
   
=10 + Maths
: (History + Geography) + Science - 3    
Maths = 40 =( & & )h = ( ) h
   
:110 + Science — 3 Maths = 40 Average speed
  
O 3 Maths - Science = 70 " 9 " "  km/h
  
Since there is no relation given between 23. Numbers between 1 and 100 which end
Maths and Science the question cannot in 3 are 3, 13, 23, 33, 43, 53, 63, 73, 83,
be solved. and 93.
15. Average speed = Sum of the numbers = 3 + 13 + 23 + ... +
pKq)j 23Fq).+7 q6)r7jj72
93
pKq)j q3L7 q)n7. This is an AP with a = 3, l = 93 and

www.sakshieducation.com
www.sakshieducation.com Quantitative Aptitude

d =10 : ' 9 "  =


  
S n= [ a + l] =
 
[ 3 + 93] = 480 O = O 9: = " $:


Hence, required average = = 48 Method: II

24. Let the number be a and b. Then, Options to question.
) & * 26. Sum of the ages of husband and wife 12
" $.  year ago = (2 × 20) year = 40 years

: ) &* " Present sum of the ages of husband and
wife " /$ &  ) year = 64 years
: √)* "
Let the age o f the two children be x and
: )* "
9 & 
/)  *0² " /) & *0²  $)*  
" / 0²  /$ 0 " $$ Now, 20 = =
 
: /)  *0 "  : %% & 9 " ' : 9 " $
On solving a + b = 29 and a - b = 21, we get : 9 " #
a = 25 and b = 4. Hence, the age of the younger child is 7
25. Let the number of males be 9 and the years.
number of females be y. 27. Let x be the total strength of class.
Sum of the salaries of the men Old average chocolate per student = 5,
"  ' 9 Total chocolate" 9
Sum of the salaries of the females By given condition:
 .
" %' = " . 
   
Now, 20000 =
 /9 & 0 " . 9 &
O  /9 & =0 . 9 " $
"  ' 9 & %' = 9 " '

www.sakshieducation.com
www.sakshieducation.com Quantitative Aptitude

9. PERCENTAGE

Percentage
‘Per cent’ means ‘per hundred’. It is denoted by the symbol %. Here x% means x per

hundred or .

Thus, any percentage can be converted into an equivalent fraction by dividing it by
100.
  
e.g. 20% = " ; 150% = "
   
Also, any fraction or decimal can be converted into its equivalent percentage by
multiplying with 100.
 
e.g. " "  %; " "  %.
   

Important Formulae
O M)(+(
1. Percentage increase = %
P)/J/ K QK'(
1(M)(+(
2. Percentage decrease = %
P)/J/ K QK'(
3. If the price of the commodity increases by r% then the reduction in consumption so
as not to
)
increase the expenditure is { |%
)
4. If the price of the commodity decreases by r% then the reduction in consumption so
as not to
)
increase the expenditure is { |%
)
5. If A's income is r% more than B’s income then B’s income is less than A’s income
)
by { |%
)
6. If A’s income is r% less than B's income then B’s income is more than A’s income
)
by { |%
)
7. Let the population of a town be P and it increases at the rate of r% per annum, then
)
(a) Population after- n years = }/ & 0

R
(b) Population n years ago = ࢘ ࢔

૚૙૙
8. Let the present value of the machine be P and if it depreciates at the rate of r% per
annum, then
)
(a) Value of machine after n years = }/  0

R
(b) Value of machine n years ago = ࢘ ࢔

૚૙૙
Example 1: Express 3/2 as rate per cent.
 
Solution: " G H% "  %
 
Example 2: Find 25% of 1000.

Solution: 25% of 1000 =G H " 

Example 3: What per cent of 6 is 144?

Solution: Required percentage = G H % " $ %

Example 4: What per cent of 2.5 kg is 15 g?

Solution: Required percentage - G H % " . %%
. 
Example 5: If the price of tea falls by 12%, by how much per cent must a house holder
increases its consumption, so as not to decrease its expenditure on tea?

www.sakshieducation.com
www.sakshieducation.com Quantitative Aptitude

) 
Solution: Increase % in consumption = ~ % " ~ %
)  
  
=~ % " %" %

Example 6: The value of a machine depreciates at the rate of 10% per annum. If its
present value is Rs.162000, what was the value of the machine 2 year ago?
  
Solution: Value of the machine 2 years ago = Rs. ૚૙ ૛ ] = Rs. / % 0
  
૚૙૙
= Rs. 200000
Example 7: If the price of 1 kg cornflakes is increased by 25%, the increase is Rs. 10.
Find the new price of cornflakes per kg.
1/##()( M( / )/M( 
Solution: Original Price == " " Rs.40
1/##()( M( / ()M( * 

(7€ D63+7 " $ " Rs.50

EXERCISE
1. 0.05 = ?% 10. A’s salary is 20% of B’s salary which is
(a) 5 (b) 0.05 25% of C’s salary. What percentage of
(c) 0.5 (d) 50 Cs salary is A’s salary?
2. 20 ? = 25% of 800 (a) 8% (b) 5%
(a) 10 (b) 15 (c) 6% (d) 4%
(c) 8 (d) 12 11. The population of a city increases at the
3. What is 40% of 40% equal to? rate of 5% per annum. There is additional
(a) 0.16% (b) 16% annual increase of 5% due to influx of
(c) 1.6% (d) 0.016% job seekers. The percent increase in
4. 9 : 4 = ? population after 3 yr is
(a) 125% (b) 200% (a) 33.1% (b) 33%
(c) 225% (d) 250% (c) 34% (d) 33.24%
5. A school mini bus brakes from 60 m/h to 12. In an examination, 32% students failed in
40 m/h. What is the percentage decrease Mathematics and 40% failed in English.
in speed? If 22% of the students failed both the
(a) 33.3% (b) 66.6% subjects then percentage of students who
(c) 77.7% (d) 45% passed in both the subject was
6. The price of a book is reduced by 25%, (a) 30% (b) 40%
what is the ratio of change in price to the (c) 50% (d) None
old price? 13. In an examination of n questions, a
(a) 1 : 4 (b) 4 : 1 student replied 15 out of the first 20
(c) 5 : 4 (d) 4 : 5 questions correctly. Of the remaining
7. What percent decrease in salaries would questions, he answered one -third
exactly cancel out the 25% increase? correctly. All the questions have the
(a) 25% (b) 20% same credit. If the student gets 50%
(c) 24% (d) 27% marks, the value of n is:
8. A’s income is 40% more than B’s (a) 30 (b) 67
income. How much per cent is B’s (c) 50 (d) 82
income less than A’s? 14. A’s marks in Biology are 20 less than
 25% of the total marks obtained by him
(a) ' % (b) 26 %
  in Biology, Maths and Drawing. If his
 
(c) ' % (d) 26 % marks in Drawing be 50, what are his
 
9. If A’s height is 50% less than that of B, marks in Maths?
how much percent B’s height is more (a) 60 (b) 47
than that of A? (c) 63 (d) cannot be determined
(a) 100% (b) 80% 15. In an election, a candidate who gets 76%
(c) 150% (d) 50% of the votes is elected by a majority of

www.sakshieducation.com
www.sakshieducation.com Quantitative Aptitude

468 votes. What is the total number of (c) 4.2% decrease (d) 4.5% decrease
votes polled? 23. A number is decreased by 25% and then
(a) 964 (b) 900 increased by 25%. The number so
(c) 1008 (d) 1024 obtained is 8 less than the original
16. 15% of the people eligible to vote are number. What was the original number?
between 18 and 25 yr of age. In an (a) 128 (b) 144
election, 75% of those eligible to vote, (c) 136 (d) 114
who are between 18 and 25, actually 24. The difference of two numbers is 20% of
voted. In that election, the number of the larger number. If the smaller number
persons between 18 and 25, who actually is 20, then the larger number is:
voted was, what percent of those eligible (a) 25 (b) 46
to vote? (c) 27 (d) 82
(a) 12.50% (b) 10.75% 25. Two numbers A and B are such that the
(c) 11.25% (d) 10.25% sum of 5% of A and 4% of B is two-
17. A scored 30% marks and failed by 15 third of the sum of 6% of A and 8% of
marks. B scored 40% marks and obtained B. Find the ratio of A : B.
35 marks more than those required to (a) 1: 2 (b) 3 : 1
pass. The pass percentage is: (c) 3 :4 (d) 4 : 3
(a) 33% (b) 40% 26. A student multiplied a number by 3/5
(c) 34% (d) 48% instead of 5/3. What is the percentage
18. The price of a table is Rs. 400 more than error in the calculation?
that of a chair. If 6 tables and 6 chairs (a) 36% (b) 64%
together cost Rs. 4800, by what percent (c) 55% (d) 35%
is the price of the chair less than that of 27. A tempo is insured to the extent of

the table? qg of its original value. If the premium

(a) 66.9% (b) 60%
 on it at the rate of 1.3 percent amounts to
(c) %% % (d) 44% Rs. 910, the original value of the tempo

19. A salesman is allowed 5 % discount on is:
 (a) Rs. 78,000 (b) Rs. 78,500
the total sales made by him plus a bonus
(c) Rs. 80,000 (d) Rs. 87,500
of % on the sales over Rs. 10,000. If his 28. When 15% is lost in grinding wheat, a

total earnings were Rs. 1990, then his country can export 30 lakh tons of wheat.
total sales (in Rs.) were: On the other hand, if 10% is lost in
(a) 60,000 (b) 42,000 grinding, it can export 40 lakh tons of
(c) 34,000 (d) 35,000 wheat. The production of wheat in the
20. A number 9 is short of y by 40%. By country is:
what per cent is y in excess of x ? (a) 40 lakh tons (b) 400 lakh tons
(a) 33 % (b) 66 % (c) 200 lakh tons (d) 900 lakh tons
  29. The sum of the number of boys and girls

(c) 66 % (d) 33 % in a school is 150. If the number of boys
 
21. A man spends 30% of his income on is x, then the number of girls becomes
food, 12% on house rent, 28% on x% of the total number of students. The
miscellaneous. If the savings at the end number of boys is:
of a month is Rs. 810, then the man’s (a) 51 (b) 65
total income is (c) 60 (d) 95
(a) Rs. 2100 (b) Rs. 2400 30. A reduction of 20% in the price of salt
(c) Rs. 2600 (d) Rs. 2700 enables a person to buy 2.5 kg more for
22. The price of a book is first increased by Rs. 100. What is the reduced price per
10% and then decreased by 5%, then the kg?
net change in the price will be (a) Rs. 8 (b) Rs. 10
(a) 4.2% increase (b) 4.5% increase (c) Rs. 8.50 (d) Rs. 9.50

www.sakshieducation.com
www.sakshieducation.com Quantitative Aptitude

31. The price per kg of sugar decreases by (a) 9408 (b) 9409
20%. By what percentage should the (c) 9410 (d) 9412
consumption be increased such that 34. The price of a Fan is 2000. After 2 year
expenditure remain the same? the price of fan is 500. The value of fan
(a) 18% (b) 30% depreciates, at the rate of R% find R%?
(c) 20% (d) 25% (a) 33.55 (b) 50
32. 24% of x +28% of 500 = 219, find the (c) 43.33 (d) 43.55
value of x? 35. The value of a machine depreciates at
(a) 329.17 (b) 330.17 rate of 20% per annum. If its value is Rs.
(c) 392.71 (d) 239.17 3200, what was the value of the machine
33. The value of mobile price depreciates at 2 years ago?
rate of 3%. The price of mobile in June (a) 3240 (b) 1620
2012 is 10000. What could be the rate of (c) 6800 (d) 5000
mobile in June 2014?
ANSWER KEY
1 a 8 c 15 b 22 b 29 c
2 a 9 a 16 c 23 a 30 b
3 a 10 b 17 a 24 a 31 d
4 c 11 a 18 c 25 d 32 a
5 a 12 c 19 c 26 b 33 b
6 a 13 c 20 c 27 d 34 b
7 b 14 d 21 d 28 c 35 d

SOLUTIONS
  
1. 0.05 = "G H " % Required % = G H % = 100%
  
2. Let  9 = 25% of 800 10. A = 20% of B; B=25% of C)
  
Then, 9 = / ' 0" ‚"G H %  e
   
3. 40% of 40% = " %  e
$ $ % 11. Total rate of increase in population per
" " %%
annum = 10%

4. 9 : 4 = G H % " % Initially, let the population be 1000.

 Population after 3 year
5. % change = %" . % 
 = 1000 / & 0
6. Let the old price be Rs. 100. 
M  M  =
; " " 
 M $ Increase % = G H%

7. Let the original salary be Rs. 100. " . %
New salary = Rs. 125 12. n (A) = 32, n (B) = 40, n (A ƒ B)
Decrease on Salary 125 - 100 = 25 = 22
Decrease on 100 = (

l00) % So, n(A „ B) = n(A) + n (B) - n (A ƒ B)
 = 32 + 40 - 22 = 50
= 20%
Percentage failed in either or both the
8. Let B’s income = Rs. 100
subjects = 50
Then, A’s income = Rs. 140

Hence, percentage of pass
Required % = G H% = (100 - 50) % = 50%


" ' % 13. 15 + (n -20) = 50% of n =
 

9. Let B’s height = 100 units l 90 + 2n - 40 = 3n l n = 50.
Then, A’s height = 50 units

www.sakshieducation.com
www.sakshieducation.com Quantitative Aptitude

14. Let B + M + D = x. Then, B = 25% of 9 "


- 20 l 9  " '
  l 9 " $ '
=( 9 - 20) = ( - 20) and D = 50.
 
 l9" $ .
- 20 + M + 50 = 9 or M  


20. % excess = 100%= %% %
 
=G  H. 21. Saving = [100 - (30 + 12 + 28)]% = 30%

So, marks in Maths cannotbe ; 30% of 9= 810
determined. 
: "'
15. Let the total number of votes polled be 9. 
  
Thus, votes polled by other candidate : 9" " Rs. 2700

= (100 - 76) % of 9 = 24% of 9 22. Let the original price be Rs. 100.
; 76% of 9 - 24% of 9 = 468 New final price = 95% of (110% of Rs.

: 52% of 9 = 468 : 9 100)
  
= 468 = Rs.
 
 
:9= " = Rs. 104.5

16. Let the number of persons eligible to ; Increase in price = 4.5%
vote be x. 23. Let the original number be 9.
Then, number of eligible persons Final number obtained = 125% of (75% of 9)
between 18 and 25 = 15% of x   
Number of persons between 18 and 25, = 9"
  
who Voted = 75% of 15% of 9 =9 "'

   
=G 9H " = 9 = 128
  
∴Required percentage 24. Let the larger number be x,

 Then, x - 20 = 9l9 9"
=G H %= 11.25%  
  $ 
17. Let the total marks be x. k 9 "  k 9 " [ " 
Then, (30% of 9) + 15  $

= (40% of x) -35 25. 5% of A + 4% of B = (6% of A + 8%

 
l 9& " 9  of B)
      
k ‚& †" / ‚& †)
l 9" l9"     
 
So, passing marks = (30% of 500) + 15 k ‚& †" ‚& B
   
 H  
"G  & H " %. k ‚" †k " "
   S  
 26. Let the number be x. Then, Error
Pass percentage =G H% " %   

18. Let the price of a chair be Rs. x. = 9 9" 9
  
 
Then, price of a table = Rs. (9 + 400). Error % = G H % = 64%
 
So, 6/9 + 400) + 69= 4800 27. Let the original value of the tempo be
l 9 " $ l 9 "  Rs. x.
Price of a table = Rs. 600 
Price of a chair = Rs. 200. Then, 1.3% KR of 9 = 910

 
Required percentage = l 9"
    
G H % " %% % l9"G
   
H " '# .
 

19. Let the total sales be Rs. x. 28. Let the total production be x lakh tons.
Then, 5 % of 9 & % (9  0 Then,
 
" 15% of x - 10% of x = (40 - 30) lakh tons
l 5% of x = 10 lakh tons
l 9&
  l9"P Q" ‡ .
/9  0 

www.sakshieducation.com
www.sakshieducation.com Quantitative Aptitude

29. We have : x + x% of 150 = 150 



2000 = ‫ܚ‬ ૛
l9&  "  U  V
૚૙૙

 @ 
l 9"  G  H =
  
 @
l9"/ 0"% G  H=
  
30. Let the original price be Rs. x per kg "
)
 
Reduced price = Rs. per kg R = 50% (depreciation)

35. Let the present value of a machine be P.
;  " . 
$9 9 suppose it depreciates at the rate of R%
 per annum.
  
:  " Then, Value of the machines n years ago
  
  R
: " =ˆ ‰
  ࡾ ࢔
  U V
૚૙૙
:9" "
 R%=20%; present value=3200; n=2
 
Reduced price = Rs. G H  ‡ " ' Value of a machine 2 years ago

) 
31. Increase % = ~ % = hF Š ‹
) ૛૙ ૛
 U V
=~ % ૚૙૙


 = ૡ૙ ૛
= 5000
=~  % " % U
૚૙૙
V

32. (a) 329.17
0.24×x+ (0.28×500) = 219
  .
X= = 329.17
.
33. (b) 9409
Value of mobile after 2 years


= , P G HQ

 
= G H

=9409
34. Ans: (b) 50
It depreciates at the rate of R% per
annum then value of the fan after n years =
T
}{  G H|


www.sakshieducation.com
www.sakshieducation.com Quantitative Aptitude

10. PROFIT AND LOSS

Cost Price: The price at which an article is purchased, is called the cost price or CP.
Selling Price: The price at which an article is sold is called the selling price or SP.
Formulae:
Gain or Profit = SP - CP
J/ )#/*
Gain % or Profit % = G % H K6 G %H
M M
)#/*%
SP = ( )×CP

Similarly, Loss = CP - SP
K++
Loss % = 0
M
K++%
SP= CP

• The Profit and Loss per cent is always calculated on the cost price.
• If a trader professes to sell his goods at CP but uses false weight, then Gain per cent or Profit percent
()))
=G H%
*)'( IK'(()))
Marked Price or List Price:
Price that is indicated or marked on the article is called marked price or MP.
Discount:
It is reduction given on the Marked Price or List Price of an article.
./+M' * .
Discount % = %; Selling Price = MP
-R 
If a trader gets x% profit and x% loss in selling two different articles, then in over all transaction, there
is always a loss which is given by

Loss% = ( 02

Example 1: A chair is bought for Rs.1950 and sold at Rs.2340. Find the gain per cent.
Solution: CP = Rs.. 1950 and SP = Rs. 2340
Gain = Rs. (2340 - 1950) = Rs. 390

Gain % = G %H "  %

Example 2: A radio is bought for Rs.780 and sold at Rs. 650. Find the loss per cent.
Solution: CP = rs.780 and SP = Rs. 650
Loss = CP - SP = Rs. (780 - 650) = Rs.130

Loss % = G H % " %. %#%

Example 3: A book is bought for Rs.80 and sold at the gain of 5%. Find the selling price.
Solution: CP = Rs. 80, Gain = 5%

SP = 105% of 80 = Rs.G ' H " hF '$

Example 4: If the cost price of 15 articles is equal to the selling price of 12 articles, then find the gain
percent.
Solution . Let cost price of each article = Rs. 1
Then, Cost price of 15 articles = Rs. 15
Selling price of 12 articles = Rs.15
But Cost price of 12 articles = Rs.12
Profit= Rs. (15 - 12) = Rs. 3

Profit % = =25%

Example 5: What is the equivalent discount of three consecutive discount 30%, 20% and 5%?
Solution: Let MP = Rs. 100
SP = 95% of 80% of 70% of 100 = Rs. 53.20
Required equivalent discount = Rs. (100 - 53.20) = Rs. 46.80

www.sakshieducation.com
www.sakshieducation.com Quantitative Aptitude

Example 6: By selling 66 m of cloth a person gains the cost price of 22 m. Find the gain percent.
Solution: Let CP of 1 m cloth = Rs. 1
J/ M #  MK*; 
Gain % = = ×100 = = 33 %
M M #  MK*;  
Example 7: A radio is listed at Rs. 500 with a discount of 10%. What additional discount must be
offered to the customer to bring the net price to Rs.423?
Solution . List price = Rs. 500, Discount = 10%
SP = 90% of Rs. 500 = Rs. 450
Sale price = Rs. 423

Additional discount = ( ×100) % = 6%


EXERCISE
1. If SP = Rs. 84, % gain = 20%, then have got a gain of 10%. The CP of the
CP = ? article is
(a) Rs. 60 (b) Rs. 65 (a) Rs .56 (b) Rs. 84
(c) Rs. 70 (d) Rs. 75 (c) Rs. 60 (d) Rs. 92
2. If CP = Rs. 20, % loss = 25%, then 10. Find the single discount equivalent to two
SP = ? successive discounts 5% and 20%.
(a) Rs. 10 (b) Rs. 15 (a) 28% (b) 24%
(c) Rs. 20 (d) Rs. 25 (c) 25% (d) 20%
3. A chair costing Rs. 400 has been sold for 11. A dishonest dealer professes to sell his
Rs. 300. The percentage loss was goods at cost price, but he uses a false
(a) 20% (b) 25% weight and he gained 25%. Find the false
(c) 15% (d) 10% weight.
4. A watch costing Rs. 250 has been sold for (a) 900 g (b) 800 g
Rs.300. The percentage profit was (c) 850 g (d) 925 g
(a) 20% (b) 15% 12. A man sold two books at Rs. 24 each, on
(c) 16 % (d) 25% one he got a profit of 20% and on other, he
5. A shopkeeper uses a weight of 960 g instead lost 20% on the whole, he
of 1000 g. What is his gain%? (a) lost Rs 1 (b) gained Rs 1
(a) 4% (b) 6% (c) lost Rs 2 (d) gained Rs 2
(c) 4 % (d) 6 % 13. Ravi purchased a scooter at th of its
  
6. By selling 18 chocolates, a vendor losses marked price and sold it at 10% more than its
the selling price of 2 chocolates. Find his marked price. His gain per cent is
loss percent. (a) 24% (b) 27%
(a) 10% (b) 15% (c) 30% (d) 32%
(c) l0 % (d) 15 % 14. A man buys an article with 20% discount on
 
its marked price. He makes a profit of 10%
7. By selling an article for Rs.2250, a person
by selling it at Rs. 825. Find its marked
losses 10%. Find his gain or loss per cent, if
price.
he sells it for Rs.3000.
(a) Rs 985.25 (b) Rs 937.50
(a) 20% gain (b) 20% loss
(c) Rs 925.50 (d) Rs 945.25
(c) 18% gain (d) 18% loss
15. A man sells 16 mangoes at a cost price of
8. A man buys 25 oranges for Rs. 100. At
20 mangoes, then percent age of his gain is
what price did he sell each orange to get a
(a) 18% (b) 20%
gain of 30%?
(c) 25% (d) 16%
(a) Rs 5.40 (b) Rs 5.20
16. An article sold for Rs. b yields a% profit.
(c) Rs 5.25 (d) Rs 5.45
Find the cost price of the article.
9. A man sold an article at a loss of 25%. If he  
sells the articles for Rs. 21 more, he would (a) Rs. (b) Rs.
 
 
(c) Rs. (d) Rs.
 

www.sakshieducation.com
www.sakshieducation.com Quantitative Aptitude

17. A trader allows a discount of 10% on the (c) 10 (d) 9


marked price of an article . How much 26. By selling 45 lemons for Rs.40, a man loses
percentage above the cost price the article 20%. How many should he sell for Rs.24 to
be marked to make a profit of 17%? gain 20% in the transaction?
(a) 25% (b) 30% (a) 19 (b) 18
(c) 28% (d) 24% (c) 24 (d) 22
18. A person sells two articles at the same price. 27. A man bought a number of clips at 3 for a
On one, he gets a profit of 25% and on the rupee and an equal number at 2 for a rupee.
other, he losses 20%. Find his profit or loss At what price per dozen should he sell them
per cent in the whole transaction. to make a profit of 20%?
(a) 12 % loss (b) 12 % profit (a) Rs.9 (b) Rs.10
 
  (c) Rs.6 (d) Rs. 7
(c) 2 % profit (d) 2 % Loss 28. A man purchased a box full of pencils at the
 
19. A cloth merchant professes to sell his rate of 7 for Rs.9 and sold all of them at the
material at a loss of 8%. But he measures 84 rate of 8 for Rs. 11. In this transaction, he
cm for a metre. Find his actual gain or loss gained Rs. 10. How many pencils did the
per cent. box contain?
 (a) 111 (b) 112
(a) 9 % gain (b) 9 % loss

  (c) 114 (d) 116
(c) 8 % gain (d) 8 % loss 29. A fruit seller has 24 kg of apples. He sells a
 
20. An article is sold at a certain price. By part of these at a gain of 20% and the
selling it 3/4 of that price, one losses 10%. balance at a loss of 5%. If on the whole he
Find the gain percent at the original price. earns a profit of 10%, the amount of apples
(a) 18% (b) 20% sold at a loss is:
(c) 14% (d) 16% (a) 9.8 kg (b) 8 kg
21. A watch is sold for Rs.144. If percentage (c) 9.6 kg (d) 12.4 kg
profit is equal to its CP numerically, then 30. Padma purchased 30 kg of rice at the rate of
what is CP? Rs. 17.50 per kg and another 30 kg rice at a
( a ) Rs 72 (b) Rs 80 certain rate . He mixed the two and sold the
(c) Rs 90 (d) Rs 100 entire quantity at the rate of Rs.18.60 per kg
22. A person bought an article on 40% discount and made 20% overall profit. At what price
and sold it at 50% more than the marked per kg did he purchase the lot of another 30
price. What profit did he get? kg rice?
(a) 250 (b) 150 (c) 350 (d) None of these (a) Rs.14.50 (b) Rs.13.50
23. A reduction of 20% in the price of sugar (c) Rs.17.50 (d) Rs.16.50
enables a person to buy 2 kg more for Rs. 31. A dairyman pays Rs 6.40 per liter of milk.
30. Find the reduced and the original price He adds water and sells ,the mixture at Rs 8
per kg of sugar. per liter, thereby making 37.5% profit. The
(a) Rs. 4 per kg and Rs. $ per kg proportion of water to milk received by the

 customers is:
(b) Rs. 3 per kg and Rs. per kg (a) 1 : 10 (b) 1 : 12

(c) Rs. 2 per kg and Rs. 2 ½ per kg (c) 2 : 7 (d) 3 : 20
(d) None of these 32. A trader mixes 26 kg of rice at Rs.20 per kg
24. A vendor bought toffees at 6 for a rupee. with 30 kg of rice of other variety at Rs.36
How many for a rupee must he sell to gain per kg and sells the mixture at Rs. 30 per
20%? kg. His profit percent is:
(a) 8 (b) 9 (a) 6% (b) 5%
(c) 5 (d) 2 (c) 11% (d) 10%
25. By selling 12 toffees for a rupee, a man 33. Albert buys 4 horses and 9 cows for
loses 20%. How many for a rupee should Rs.13,400. If he sells the horses at 10%
he sell to get a gain of 20%? profit and the cows at 20% profit, then he
(a) 6 (b) 8

www.sakshieducation.com
www.sakshieducation.com Quantitative Aptitude

earns a total profit of Rs .1880. The cost of gain, then he gains Rs .400. The actual price
a horse is: of the table is:
(a) Rs. 2200 (b) Rs. 2000 (a) Rs. 2100 (b) Rs .1900
(c) Rs. 2700 (d) Rs. 3200 (c) Rs. 2200 (d) Rs. 2400
34. A man purchases two clocks A and B at a 36. A shopkeeper offers 2.5% discount on cash
total cost of Rs .650. He sells A with 20% purchases. What cash amount would Rohan
profit and B at a loss of 25% and gets the pay for a cycle, the marked price of which is
same selling price for both the clocks. What Rs. 650?
are the purchasing prices of A and B (a) Rs. 633 (b) Rs. 633.75
respectively? (c) Rs. 635 (d) Rs. 750
(a) Rs 550, Rs 660 (b) Rs 250, Rs 400 37. A sold an article to B at a profit of 10% and
(c) Rs 378, Rs 375 (d) Rs 300, Rs 350 B sells it to C at a loss of 10% and C paid
35. On selling a chair at 7% loss and a table at 2079. How much money was paid by A?
17% gain, a man gains Rs .296. If he sells (a) Rs 2100 (b) Rs 2400
the chair at 7% gain and the table at 12% (c) Rs 2160 (d) Rs 2480

ANSWER KEY
1 c 10 b 19 b 28 b 37 a
2 b 11 b 20 b 29 c
3 b 12 c 21 b 30
4 a 13 c 22 b 31 a
5 c 14 b 23 b 32 b
6 a 15 c 24 c 33 b
7 a 16 b 25 b 34 b
8 b 17 b 26 b 35 d
9 c 18 d 27 c 36 b

SOLUTIONS
:R 
1. CP = " = Rs. 70
J/ % 
K++% 
2. SP= G H CP = 
 
= Rs. 15
3. Loss = CP - SP
= 400 - 300 = 100

Loss % = G H % = 25%

4. Profit = SP – CP
= 300 – 250 =50

Profit = G H%= 20%

 
5. Gain% = = =4 %
  
6. Let SP of 1 chocolate = Rs. 1
SP of 18 chocolates = Rs. 18
Loss = Rs. 2
; CP = S P + Loss = 18 + 2 = Rs. 20

b% " %" %

www.sakshieducation.com
www.sakshieducation.com Quantitative Aptitude

7. SP =Rs. 2250, Loss= 10%


 
CP = Rs. G H = Rs. 2500
 
CP = Rs. 2500, SP = 3000,
Profit = Rs. (3000 - 2500) = Rs. 500

Gain % = G H% = 20%


8. CP of 1 orange = Rs. = Rs. 4

Gain = 30%
 
SP of 1 orange = Rs. { |

=Rs. 5.20
9. Let the CP = Rs. x
Then,
#
P 9Q  P 9Q " 
O 9 " 
O x = Rs. 60
10. Let MP = Rs. 100
 
After 5% discount, SP= Rs. G H

=Rs. 95
After 20% discount ,

SP = Rs. G H=Rs. 76

; Total discount (100 - 76)% = 24%
11. By short cut method .
Let the error be x g.

Then, = 25

O $9 "  9
O 9 "
O 9 "  o
; Weight used = (1000 - 200) g = 800 g
12. SP of first book = Rs. 24, Gain = 20%

CP of first book = Rs. G $H

= Rs. 20
SP of second book = Rs. 24,
Loss = 20%

CP of second book = Rs. G $H

= Rs. 30
Total CP = Rs. (20 + 30) =Rs. 50
Total SP = (24 × 2)= Rs. 48
Loss = Rs. (58-48)= Rs. 2
13. Let the MP of a scooter = Rs. x

; CP of a scooter = Rs. , gain = 10%

 
SP = Rs. G 9H " Rs.
 
 
Gain = SP – CP = Rs. G  H
 
  
"G H= Rs.
 
 
Gain % = G H%
 

www.sakshieducation.com
www.sakshieducation.com Quantitative Aptitude

" %
14. Let the marked price be Rs. x.
 
CP = 80% of Rs. x = Rs. G 9H= Rs.
 
  
SP=110% of Rs. = Rs. G H
  

= Rs.

9 ' 
; " ' O 9 " P Q" #. 
 
; MP = Rs. 937.50
15. Let the cost price of 20 mangoes be Rs. x

The cost price of 1 mango = Rs.


SP of 1 mango = Rs.

  
Gain = SP- CP = Rs. G  H = Rs.
  

Gain" Œ ૡ૙
࢞ %
૛૙
9 
"P Q % " %
' 9
16. SP = Rs. b and Profit = a%
&)
*"P Q `}

O `}= Rs. G H

17. Let the CP be Rs. 100 and MP be Rs. x above Rs. 100.
Then, MP = Rs. (100 + x) and discount =10%

; SP = Rs. { / & 90| = Rs. 117

O 9 " 30
; Cost should be labelled at 30% above CP.
18. Let SP of each article = Rs. 100
On first 25% profit on second 20% loss
 
CP of first = Rs. G H = Rs. 80

 
CP of second = Rs. G H = Rs. 125

Total CP = Rs. (80 + 125) = Rs. 205
Total SP = Rs. 200;
Loss on the whole = Rs. (205 - 200)
= Rs. 5
 
Loss% = G H% =  %
 
19. Let CP of 1 m cloth be Rs. 100
SP of 84 cm cloth = Rs. 92
CP of 84 cm cloth = Rs. 84
Gain - (SP - CP) = Rs. (92 - 84) =8

Gain% = G H=9 %
 
20. Let the CP be Rs. 100.
SP at 10% loss = Rs. 90

of actual SP = Rs. 90


O ‚+qi)j Ž} "Rs. /  0= Rs. 120

; )3. "  %

www.sakshieducation.com
www.sakshieducation.com Quantitative Aptitude

21. M-I:Let CP " 9


Then, profit = x%
 
;SP = Rs.

/ & 9 09
; " $$

O9 & 9  $$ "
O /9  ' 0/9 & ' 0 "
O 9 " ' /‘ 9 T ' 0
; `} KR €)q+g = Rs. 80
M-I:OTP
22. Let the marked price be Rs. 100. Then cost price = 100 – 40 = Rs. 60
Selling price = 100+ 50 = Rs. 150
Profit = 150 – 60 = 90

 }6KR3q% " "  %


23. The reduced price of sugar " =Rs. 3 per kg and

The original price of sugar
 
" " " hF. D76 no
/   0 $ $
24. C.P. of 6 toffees = Re. 1
S.P. of 6 toffees = 120% of Rs. 1

= Rs. .


For Rs. G H, toffees sold = 6.

For Rs. 1, toffees sold

= G% H=5.

25. Let S.P. of 12 toffees be Rs. x.
Then, 80 : 1 = 120 : x or
 
x=G H"
 

For Rs. , toffees sold = 12.


For Rs. 1, toffees sold = G  H = 8.

26. Let S.P. of 45 lemons be Rs. x.
Then, 80 : 40 = 120 : x

or 9 " G H"%


For Rs. 60, lemons sold = 45. For Rs. 24, lemons sold= G $H = 18.

27. Suppose he bought 1 dozen clips of each kind.
C.P. of 2 dozens
=Rs. G & H=Rs. 10
 
S.P. of 2 dozen = 120% of Rs. 10

= Rs. G H = Rs. 12

Hence, S.P. per dozen = Rs. 6.
28. Suppose, number of pencils bought = L.C.M. of 7 and 8 = 56.

C.P. of 56 pencils = Rs. G %H

= Rs. 72.
S.P. of 56 pencils = Rs. G %H = Rs. 77.

Now, Rs. 5 are gained on 56 pencils.
www.sakshieducation.com
www.sakshieducation.com Quantitative Aptitude


So, Rs. 10 are gained on, G H

=112 pencils.
29. Let the quantity sold at a loss be x kg and let C.P per kg be Re.1
Total C.P = Rs. 24
Total S.P=
Rs.  % KR /$  90 & % KR 9!
  
= Rs. { /$  90 & |= Rs. G H
  
#%  9 %$
; " k #%  9 " '

k 9 " $' k 9 " . %no
30. Let the required price per kg be Rs. x. Then, C.P. of 60 kg rice = Rs. (30 × 17.50 + 30 × x )
"Rs. / & 90
S.P. of 60 kg rice = Rs. (60 × 18.60)
= Rs. 1116
%  / & 90
; "
 & 9
  9
O "
 & 9 
k     9 "  & 9
k ' 9 " $
$ #
k9"P Q " P Q " .
' 
So, the C.P of second lot is Rs. 13.50 per kg

31. Mean cost price = Rs. G 'H
.

= Rs.
By the rule of allegation:
C.P of 1 liter
C.P of 1 liter Milk
water 6.40
0 Mean
Price
૟૝
૚૚

૟૝ ૟૝
૚૚૙ ૚૚
 
; Required ratio = : = 1 : 10

32. C.P. of 56 kg rice
= Rs. (26 × 20 + 30 × 36)
= Rs. (520 + 1080)
= Rs. 1600.
S.P. of 56 kg rice
= Rs. (56 × 30) = Rs. 1680.

; )3. " G H % " 5%

33. Let C.P. of each horse be Rs. x and C.P of each cow be Rs. y. Then,
$9 & = " $ … … . . . . . /30
And, % KR $9 &  % KR =
" ''

www.sakshieducation.com
www.sakshieducation.com Quantitative Aptitude


O 9 & = " ''
 
O 9 & = " $ ……...(ii)
Solving (i) and (ii), we get :
9 " 
; Cost price of each horse = Rs. 2000.
34. Let C.P. of clock A be Rs. x and that of clock B be Rs. /%  90. Then,
 % KR 9 " #% KR /%  90
 '
O %  9 " 9 " 9
# 
% 
O 9 " % O 9 " P Q

= 250.
; C.P. of A = Rs. 250,
C.P. of B = Rs. 400.
35. Let C.P. of the chair be Rs. x and that of the table be Rs. y.
’, #% KR =  #% KR 9 "  %
O #= – #9 "  % … … … /30
And, 12% of y + 7% of x = 400
O = & #9 " $ … … … . /330.
Solving (i) and (ii), we get :
= " $
C.P. of table = Rs. 2400.
36. S.P " # % KR Rs. 650


= Rs. G % H=Rs. 633.75
 
37. Amount paid by B = Rs. 110

Amount paid by C = Rs. " Rs. 99

Then, if C paid Rs. 99 then, A paid
Rs. 100.
If C paid Rs. 2079, then A paid

= Rs. G H= Rs. 2100


www.sakshieducation.com
www.sakshieducation.com Quantitative Aptitude

11. RATIO & PROPORTION

Ratio
The ratio of two quantities a and b is the fraction and is expressed as a : b. Here a is the first
term or antecedent and b is the second term or consequent. Since the ratio expresses the number of
times one quantity contains the other, it is an abstract (without units) quantity.
A ratio remains unaltered if its numerator and denominator are multiplied or divided by the
same number, e.g. 4 : 3 is the same as (4 × 10): (3 × 10) i.e. 40 : 30.
 
20 : 15 is the same as G H : G H i.e. 4:3
 
Kinds of Ratios
Duplicate Ratio : a2 : b2 is called duplicate ratio of a : b.
Triplicate Ratio : a3 : b3 is called triplicate ratio of a : b.
Sub -Duplicate Ratio : √) : √* is called sub-duplicate ratio of a : b.
૜ ૜
Sub - triplicate Ratio: √) : √* is called sub-triplicate ratio of a : b.
Compound Ratio: ab : cd is the compound ratio of a : c and b : d . It is the ratio of the product of
the antecedents to that of the consequents of two or more given ratios.
Inverse Ratio : : is the inverse ratio of a : b.
 
 M  M.
Componendo and Divedendo: If " , then "
 .  M.
Proportion:
 M
When two ratios are equal, they make a proportion, i.e. if " , then a, b, c and d are in
 .
proportion.
This is represented as a : b : : c : d and is read as “a is to b as c is to d”
When a, b, c and d are in proportion, then a and d are called the Extremes and b and c are called the
Means, also Product of the Means = Product of the Extremes i.e. bc = ad.
Continued Proportion
If three quantities a, b and c are such that a : b : : b : c, then b2 = ac and a, b and c are in continued
proportion. Also, the quantity c is called the third proportion of a and b.
Fourth Proportion
If four quantities a, b, c and x are such that a : b : : c : x, then ax = bc and x is called the fourth
proportion of a, b, and c.
Mean or Second Proportion
If three quantities a, b and x are such that a : x :: x : b, then x2 = ab and x is called the mean of a and
b. Also, If a : b =c : d, then the following properties hold good.
(i) b : a = d : c (Invertendo)
(ii) a : c = b : d (Alter nendo)
(iii) (a + b) : b = (c + d) : d (Componendo)
(iv) (a - b) : b = (c - d) : d (Dividendo)
 M.
(v) " (Componendo - Dividendo)
 M.
Variation
If two quantities x and y are related in such a way that as the quantity x changes it also brings a
change in the second quantity y, then the two quantities are in variation.
Direct Variation
The quantity x is in direct variation to y if an increase in x makes y to increase proportionally. Also a
decrease in x makes y to decrease proportionally it can be expressed as x = ky, where, k is called the
constant of proportionality.
e.g. Cost is directly proportional to the number of articles bought.
Inverse Variation

www.sakshieducation.com
www.sakshieducation.com Quantitative Aptitude

The quantity x is in inverse variation to y if an increase in x makes y to decrease proportionally. Also,


X
a decrease in x makes y to increase proportionally. It can be expressed as x= where, k is called the

constant of proportionality, e.g. The time taken by a vehicle in covering a certain distance is
inversely proportional to the speed of the vehicle.
Joint Variation
If there are more than two quantities x, y and z and x varies with both y and z, then x is in joint
variation to y and z. It can be expressed as x = kyz, where, k is the constant of proportionality.
e.g. Men doing a work in some number of days working certain hours a day!
Distribution of an Amount
If an amount A is distributed in the ratio a : b, then

First part = ‚


Second part = ‚

Example 1: Divide 60 in the ratio of 1 : 3
Solution . We have 1 + 3 = 4
first part = 60 = l5


second part = 60 = 45

Thus, the required parts are 15 and 45.
Example 2: The ratio of boys to girls in a science class of 28 is 16 : 12. Express it in simplest ratio.
Solution: 16 : 12 = 4 : 3 Which is the ratio in its simplest form.
Example 3: Compare the ratio of the third proportion of 6 and 5 with the fourth proportion of 4, 3
and 10.
Solution: Third proportion of 6 and 5 is 6 : 5 :: 5 : x

O9"
%
Fourth proportion of 4, 3 and 10 is 4 : 3 :: 10 : y

O=" "
$ 
 
; Required ratio = : = 5 : 9
 
Example 4: Two numbers are in the ratio 2 : 3. If 10 is added to each, they are in the ratio 4 : 5.
Find the two numbers?
Solution: Let the two numbers be x and y.
 
" ……(i)
 
  
" ………(ii)
  
Solving (i) and (ii) we get x = 10 and y = 15
; The two numbers are 10 and 15.
Example 5: The ratio between two numbers is 3 : 7. If their LCM is 210, find the numbers.
Solution: Let the numbers be 3x and 7x
LCM is 3 × 7 × x = 21x
; 21x = 210
; Numbers are 30 and 70.

www.sakshieducation.com
www.sakshieducation.com Quantitative Aptitude

EXERCISE
1. The mean proportional between 234 and (c) 4 :1 5 (d) 9 : 4
104 is: 13. A bag contains 25 paise, 10 paise and 5
(a) 16 (b) 40 paise coins in the ratio 1: 2 : 3. If their
(c) 54 (d) None of these total value is 60, the number of 5 paise
2. The fourth proportional to 5, 8, 15 is : coins is
(a) 22 (b) 24 (a) 100 (b) 500
(c) 23 (d) 20 (c) 300 (d) 400
3. The third proportional to 0.36 and 0.48, 14. The monthly salary of A, B and C is in
is: the ratio of 4 : 5 : 7. If C’s monthly
(a) 0.64 (b) 0.1728 salary is Rs. 300 more than that of A.
(c) 0.44 (d) 0.82 Then, B ’s annual salary is
4. In a ratio , which is equal to 3 : 4, if the (a) Rs. 6000 (b) Rs. 8500
antecedent is 12, then the consequent is: (c) Rs. 4000 (d) Rs. 6500
(a) 10 (b) 16 15. Ratio between two numbers is 5 : 6 and
(c) 20 (d) 22 sum of their squares is 549. The numbers
5. Ratio of the earnings of A and B is 4:7. If are
the earnings of A increase by 50% and (a) 10, 12 (b) 15, 18
those of B decrease by 25%, the new (c) 20,24 (d) 30, 36
ratio of their earnings become 8: 7. What
are A’s earnings? 16. A club consists of 24 members. The ratio
(a) Rs. 25,000 (b) Rs. 26,000 of men to women can be
(c) Rs. 29,000 (d) Data incorrect (a) 2:3 (b) 3:4
6. What least number must be subtracted (c) 1:3 (d)2:5
from each of the numbers 14, 17, 34 and 17. Some money is divided among three
42 so that the remainders may be workers A, B and C such that 5 times
proportional? A’s share is equal to 12 times B’s share
(a) 4 (b) 3 (c) 2 (d) 9 which is equal to 6 times C share. The
7. The compounded ratio of (2 : 3), ratio between the shares of A, B, C is
(6 : 11) and (11 : 2) is : (a) 5 :1 0 :1 2 (b) 12 : 5 :10
(a) 7 : 2 (b) 2 : 1 (c) 10: 1 2 :5 (d) 5 :12 : 10
(c) 11: 2 4 (d) 58 : 121 18. A man is 20 yr older to his son. The
8. 7 is what part of 8. present age of the son is 30 yr. How
(a) (b) many years ago was the ratio of their
 
  ages 1 : 2 ?
(c) (d) (a) 18 yr (b) 20 yr
 

9. If 9 " '= and = " ] , then is (c) 10 yr (d) 15 yr
7
equal to 19. The ratio of Anita's age to the age of her
  mother is 4:9. The difference between
(a) (b) their ages is 25 yr. The ratio of their ages
 

(c) (d) after 10 yr will be
 
10. If a : b = 5 : 9, b : c = 6 : 11, find a : b : (a) 10:6 (b) 6 : 10
c (c) 6 : 11 (d) 11 : 6
(a) 11 :18 :33 (b) 10 :19 :34 20. The electricity bill of a certain
(c) 11: 19 : 34 (d) 10 : 18 : 33 establishment is partly fixed and partly
11. Duplicate ratio of x : 2y varies as the number of units of
(a) 9 : $= (b) x 2 + 2y2 electricity consumed. When in a certain
month 540 units are consumed, the bill is
(c) √9: V= (d) 2y : x
Rs. 1800. In another month 620 units are
12. If A : B = 1 : 3 an d B: C : 4 : 5 , Find A : consumed and the bill is Rs. 2040. In yet
C. another month 500 units are consumed.
(a) 4 : 9 (b) 15:4 The bill for that month would be:
www.sakshieducation.com
www.sakshieducation.com Quantitative Aptitude

(a) Rs. 1605 (b) Rs. 1680 22. An amount of Rs. 735 was divided
(c) Rs. 1840 (d) Rs. 2050 between A, B and C. If each of them had
21. An amount of Rs. 2430 is divided received Rs. 25 less, their shares would
among A, B and C such that if their have been in the ratio of 1:3:2 . The
shares be reduced by 5, 10 and 15 money received by C was:
respectively, the remainders shall be in (a) Rs. 198 (b) Rs. 228
the ratio of 3 : 4 : 5. Then, B’s share was (c) Rs. 225 (d) Rs. 245
(a) Rs. 609 (b) Rs. 798
(c) Rs. 845 (d) Rs. 810

ANSWER KEY
1 d 6 c 11 a 16 c 21 d
2 b 7 b 12 c 17 b 22 d
3 a 8 c 13 c 18 c
4 b 9 a 14 a 19 c
5 d 10 d 15 b 20 b

SOLUTIONS
1. Required mean proportional k 9  %9 & '' " 9   9 & #'
= √ $ $ k 9 " k9"
"√  ' ; Required number = 2.
 
"/ $0 = 156. 7. Required ratio = G H
 
2. Let the fourth proportional to 5, 8, 15 be 
x, Then, 5 : 8 :: 15 : x " " :
  
k 5x = (8 × 15) k 9 " = 24. 8. Required part =
 
3. Let the third proportional to 0.36 and 
9. 3x = 8y O 9 "
0.48 be x. 
Z
Then, 0.36 : 0.48 :: 0.48 : x 9z = 5y O ” "

. $' . $' 9 '= #
k9"P Q " . %$ ; " "
. % ] = 
4. (b)let 3:4 = 12: x 10. a : b = 5 : 9
: 12 × 4 " 9   
Y b : c = 6 : 11= 6 × : 11 × = 9 :
:x = = 16. 

 

5. Let the original earnings of A and B be ; a : b : c = 5 : 9 : = 10 :18 :33

Rs. $9 and Rs. 7x. 11. Duplicate ratio of.
New earnings of A = 150% of Rs. 4x x: 2y = x2 : (2y)2 = x2 : 4y2
"Rs. 6x 12. A : B = 1: 3 = (1 × 4): (3 × 4) = 4 :12
New earnings of B = 75% of Rs. #9 B : C = 4 : 5 = (4 × 3): (5× 3)
    = 12 : 15
= Rs. G #9H " "Rs.
   ; A :B : C = 4 : 12 : 15 OA: C = 4 : 15
This does not give x. So, the given data is 13. Let the number of 25-p, 10-p and 5-p coins
inadequate. be x, 2x and 3x respectively. Then, 0.25
6. Let the required number be x. x+0.2 x+0.05 x=60
Then, 0.6 x=60 => x=100 
(14 - x): (17 - x):: (34 - x) : (42 - x)  number of 5-p coins = 3 × 100 = 300.
$9 $9 14. Let the monthly salaries of A, B and C be
; " k / $  90/$  90
#  9 $  9 4x, 5x and 7x respectively.
" / #  90/ $  90 4x + 300 = 7x O x = 100
www.sakshieducation.com
www.sakshieducation.com Quantitative Aptitude

The monthly salary of B = Rs. 500 and 21. Remainder = Rs. [2430 - (5 +10 + 15)]
his annual salary = Rs. 2400.
= 12 × 500 = Rs. 6000. ; B ’s share = Rs. {G$

H& |
15. Let the two numbers be 5x and 6x 
= Rs. 810.
respectively.
/90 & /%90 " $ 22. Remainder = Rs. [735- (25 × 3)]
= Rs. 660.
O 9 & %9 " $
; Money received by C
O % 9 " $ 
O 9 " = Rs. {G%% H & | = Rs. 245.

O9"
So, the two numbers are 15 and 18
respectively.
16. On dividing 24 into two whole numbers,
the sum of the terms of the ratio must be
a factor of 24. So, 1 : 3 is the required
ratio.
17. Let 5A = 12B = 6C = k
X X X
Then , A = , B = ,C =
  
X X X
; A : B : C = : : = 12 : 5 : 10
  
18. Present age of son = 30 years
Present age of man = (30 + 20)year s
= 50 years
Then,
9
" O  /  90 "   9
 9 
O9"
So, 10 years ago the ratio of their ages
was 1: 2
19. Let Anita’s present age be 4 x and her
mother’s present age be 9 x years.
Then, / 9  $90 " 
O 9 "  O 9 " 
Ratio of their ages after 10 years
$9 & $ &
" "
9& &
 & %
" " " " %:
$ & 
20. Let the fixed amount be Rs. x and the
cost of each unit be Rs. y. Then,
$ = & 9 " ' … … … . . /3 0
And % = & 9 "  $ … … /330
On subtracting (i) from (ii), we get
80y = 240
k y=3.
Putting y = 3 in (i), we get:
$ & 9 " '
k 9 " / '  % 0 " ' .
; Fixed charges = Rs. 180, Charge per unit
= Rs. 3.
Total charges for consuming 500 units
= Rs. (180 + 500 × 3) = Rs.1680.
www.sakshieducation.com
www.sakshieducation.com Quantitative Aptitude

12. PARTNERSHIP
Partnership
When two or more than two persons run a business jointly, they are called partners in the business,
and the deal between them is known as partnership.
Partnership is of two types:
1. Simple Partnership
2. Compound Partnership
1. Simple Partnership: When investments of all the partners are for the same period of time, the
profit or loss is distributed among the partners in the ratio of their original investments.
Suppose A and B invest Rs. p and Rs. q respectively for a year in a business, then at the end of the
year. Share of A’s profit (loss): Share of B’s profit (loss) = p: q.
2. Compound Partnership: When investments of all the partners are for different period of time,
then equivalent capitals are calculated for a unit of time and the profit or loss is divided in the ratio of
the product of time and investment.
Suppose A and B invest Rs. p and Rs. q for x months and y months respectively, then Share of A’s
profit (loss): Share of B’s profit.(loss) = p x : qy.
Partners are of two types
(i) Working Partner, and
(ii) Sleeping Partner . .
(i) Working Partner : A partner who manages the business is called a working partner.
(ii) Sleeping Partner : A partner who only invests the money is called a sleeping partner.
Example 1: A and B started a business with capitals of Rs. 25000 and Rs. 40000 respectively. Find
the share of A and B out of an annual profit of Rs. 6500.
Solution . Ratio of shares of A and B = Ratio of their investments = 25000 : 40000 = 5 : 8

A’s share = Rs. G % H = Rs. 2500


and B’s share = Rs. 6500 = Rs. 4000

Example 2: A, B and C start a business each investing Rs. 16000. After 3 months A withdrew Rs.
2000, B withdrew Rs. 4000 and C invests Rs. 8000 more. At the end of the year a total profit of Rs.
41580 made. Find the share of A, B. a n d C
Solution . Ratio of capitals of A, B and C
= (16000 × 3 + 14000 × 9): (16000 × 3 + 12000 × 9) : (16000 × 3 + 24000 × 9)
= 174000 : 156000 : 264000 = 29 : 26 : 44

A’s share = Rs. ( 41580) = Rs. 12180


B’s share = Rs. 41580 = Rs. 10920


C’s share = Rs. × 41580 = Rs. 18480

Example 3 : A, B and C enter into a partnership with a total of Rs. 8200. A’s capital is Rs. 1000
more than B’s and Rs. 2000 less than C’s. What is B’s share of the year’s profit of Rs. 2,460?
Solution Given, A = B + 1000 = C - 2000
C = B + 3000
A + B + C= (B + 1000) + (B) + (B + 3000)
8200 = 3B + 4000 O 3B = 8200 - 4000 OB = Rs. 1400

Share of profit of B =Rs. $% = Rs. 420


EXERCISE

www.sakshieducation.com
www.sakshieducation.com Quantitative Aptitude

1. A, B and C started a business by 30000. At the end of 10 months C


investing Rs. 28000, Rs. 35000 and Rs. received Rs. 1850 as his share. Find the
14000 respectively. At the end of a year total profit.
they got a total profit of Rs. 5225. Find (a) Rs. 7955 (b) Rs. 7030
A’s share (c) Rs. 8510 (d) Rs. 6845
(a) Rs. 1740 (b) Rs. 1850 9. A, B and C enter into a partnership . A
(c) Rs. 1900 (d) Rs. 1650 contributes 320 for 4 months, B
2. A, B and C started a business by contributes Rs. 510 for 3 months, and C
investing Rs. 45000, Rs. 55000 and Rs. contributes Rs. 270 for 5 months. If the
60000 respectively. At the end of a year total profit is Rs. 208, find the profit
they got a total profit of Rs. 11200. share of the partner A.
Find how much B gets more than A in (a) Rs. 76.50 (b) Rs. 64
the profit (c) Rs. 67.50 (d) Rs. 46
(a) Rs. 700 (b) Rs. 750 10. Three hikers A, B and C start on a trip
(c) Rs. 710 (d) Rs. 780 with Rs. 50 each and agree to share the
3. A and B started a business with expenses equally. If at the end of the trip,
investments Rs. 42000 and Rs. 63000 A has Rs. 20 left with him, B Rs. 30
respectively. After 4 months B withdraws and C Rs. 40, how must they settle their
from the business. At the end of a year accounts?
they got Rs. 9600 as total profit. Find (a) A will pay Rs. 10 to C
the share of B. (b) C will pay Rs. 10 to B
(a) Rs. 5600 (b) Rs. 2800 (c) B will pay Rs. 10 to C
(c) Rs. 3200 (d) Rs. 6400 (d) C will pay Rs. 10 to A
4. Ajay and Abhay started a business with 11. Rs. 1290 is divided between A, B and C
investments of Rs. 13000 and Rs. 39000 so that A’s share is times B’s and B’s
respectively. After 5 months Arun joins 

with a capital of Rs. 52000. At the end share is

times C’s. What is C’s share?
of a year they got a profit of Rs. 14250. (a) Rs. 200 (b) Rs. 400
Find the share of C. (c) Rs. 240 (d) Rs. 420
(a) Rs. 3650 (b) Rs. 5250 12. What amount of money is divided
(c) Rs. 6750 (d) Rs. 2250 between A, B and C if B and C together
5. P and Q started a business with capitals get Rs. 100 and A gets twice as much as
of Rs.25000 and Rs.40000 respectively. B while C with A gets Rs. 150?
Find the ratio of investments? (a) Rs. 200 (b) Rs. 250
(a) 5:2 (b) 5:3 (c) Rs. 300 (d) Rs. 350
(c) 5:8 (d) 5:10 13. A and B entered into partnership with
6. Three partners started a business with capitals in the ratio of 4 : 5. After 3
Rs. 80000. At the end of the year they months, A withdrew 1/4 of his capital
receive Rs. 1800, Rs. 3000 and Rs. and B withdrew 1/5 of his capital. The
4800 as profit. Find the investment of the gain at the end of 10 months was Rs.
second person. 760. Find their shares of profit,
(a) Rs. 25000 (b) Rs. 40000 (a) Rs. 330, Rs. 440 (b) Rs. 330,
(c) Rs. 15000 (d) Rs. 32000 Rs. 430
7. A and B together invested Rs. 12000 in a (c) Rs. 340, Rs. 440 (d) Rs. 340,
business. At the end of the year, out of a Rs. 430
total profit Rs. 1800. A’s share was Rs. 14. Radhika and Renuka enter into a
750. What was the investment of A? partnership with investment of Rs.
(a) Rs. 5000 (b) Rs. 10000 50000 and Rs. 70000 respectively.
(c) Rs. 12000 (d) Rs. 15000 Renuka gets 10% of the total profit for
8. A started a business with Rs. 18000. maintaining the business and the
After 4 months B joins with Rs. 24000. remaining profit is distributed between
After 2 more months C joins with Rs. them in the ratio of their investments. If
www.sakshieducation.com
www.sakshieducation.com Quantitative Aptitude

the total profit at the end of the years is and Rs. 45000 respectively. A and B
Rs. 30000, find the total share of leave the business after a few months at
Renuka. the same time. At the end of the year,
(a) Rs. 16575 (b) Rs. 15750 they share the profits in the ratio of 6 : 4 :
(c) Rs. 18750 (d) Rs. 11250 9. After how many months did A and B
15. A, B and C invest Rs. 4000, 5000 and leave the business ?
6000 respectively in a business and A (a) 6 months (b) 2 months
gets 25% of profit for managing the (c) 3 months (d) 4 months
business, the rest of the profit is divided 21. Four transport companies A, B, C and D
by A, B and C in proportion to their rented a parking place. A kept 12 cars for
investment. If in a year, A gets Rs. 200 5 months, B kept 20 cars for 6 months, C
less than B and C together, what was the kept 15 cars for 5 months and D kept 30
total profit for that year? cars for 6 months in the parking place. If
(a) Rs. 1000 (b) Rs. 1500 A’s share of rent is Rs. 2400 the total rent
(c) Rs. 1800 (d) Rs. 2000 of the parking place is
16. P and Q invested Rs. 8000 and Rs. 4000 (a) Rs. 17400 (b) Rs. 18600
in a partnership business. Each partner (c) Rs. 16500 (d) Rs. 19200
received 5% interest on the capital 22. A and B entered in to a partnership with
invested. At the end of year, there was a investments of Rs. 15000 and Rs. 40000
profit of Rs.10000. What was the share of respectively. After 3 months A left from
each partner (excluding interest)? the business , at the same time C joins
(a) Rs. 6000, Rs. 3000 with Rs. 30000, At the end of 9 months
(b) Rs. 6100, Rs. 3900 they got Rs. 7800 as profit. Find the
(c) Rs. 6267, Rs. 3133 share of B.
(d) Rs. 6348, Rs. 3200 (a) Rs. 4800 (b) Rs. 600
17. A, B and C enter in to a partnership with (c) Rs.2400 (d) Rs. 1200
investment in the ratio 4: 3: 2. After 4
months A and B withdraw half of their
capital and after 7 months C added 2/5 of
his capital. Find the share of B in the
total profit of Rs. 12600 at the end of the
year.
(a) Rs. 3600 (b) Rs. 4800
(c) Rs. 4200 (d) Rs. 3900
18. Two partners invested Rs. 1250 and Rs.
850 respectively in a business. Both the
partners distribute 60% of the profit
equally and distribute the rest 40% as the
interest on their capitals. If one partner
received Rs. 30 more than the other,
find the total profit.
(a) Rs. 300 (b) Rs. 393.75
(c) Rs. 384.50 (d) Rs. 400
19. A and B enter into a partnership with
capitals in the ratio 2 : 3. At the end of 9
months A withdraws from the business.
If their profits are in the ratio 1: 2, how
long did B invest his capital ?
(a) 12 months (b) 8 months
(c) 10 months (d) 11 months
20. A, B and C start a business with
investments of Rs. 90000, Rs. 60000
www.sakshieducation.com
www.sakshieducation.com Quantitative Aptitude

ANSWER KEY
1 c 5 c 9 b 13 b 17 a 21 a
2 a 6 a 10 d 14 c 18 b 22 a
3 c 7 a 11 c 15 d 19 a
4 b 8 d 12 a 16 c 20 d

SOLUTIONS
Let the total profit be Rs. x
1. Ratio of shares of A, B and C = Ratio of 
C’s share = Rs.
their investments = 28000: 35000: 14000 
9
=4:5:2 ; " '
 #
;A’s share = Rs. G H=Rs. 1900 ' #
O9" " %'$
2. Ratio of shares of A, B and C = Ratio of
their investments = 45000: 55000: 60000 Hence, the total profit is Rs. 6845.
= 9:11:12 9. A’s profit : B’s profit; C’s profit
 = MEI of A : MEI of B : MEI of C
A’s share = Rs. /  0 = Rs. 3150
 = 320 × 4 : 510 × 3 : 270 × 5
B’s share = Rs. G  H = Rs. 3850 = 1280 : 1530 : 1350

;B’s share more than A = 128 :153 : 135
= Rs. (3850 - 3150) = Rs. 700 '
"  '
3. Ratio of capitals of A and B ' &  & 

= (42000 × 12) : (63000 × 4) = 2 : 1 ; Profit of A =  '= Rs. 64
 
B’s share = Rs. / % 0 = Rs. 3200 10. They start with total of Rs. (50 × 3) =

4. Ratio of capitals of Ajay, Abhay and Rs. 150 and they return after the trip
Arun = (13000 × 12): (39000 × 12) : with (20 + 30 + 40) = Rs. 90
(52000 × 7) = 3 : 9 : 7 So, to settle their accounts, each person
 
C’s share = Rs. G $ H= Rs. 5250 must have Rs. = Rs. 30 with them.
 
5. Ratio of their investments = 25000:40000 Hence, C must pay Rs. 10 to A.
  11. A: B = 1 :1= 3 : 2 = 3 × 7 : 2 × 7
= = 
 
= 5:8 = 21 : 14

6. The ratio of profit of the three persons B :C = 1 : 1 = 7 : 4 = 7 × 2 : 4 × 2

= 1800:3000:4800 = 3 : 5 : 8 = 14 : 8
; Investment of the second person ; A : B : C = 2 1 : 14 : 8
 
"Rs. G ' H = Rs. 25000 ; Cs share =  = Rs. 240
   
7. Since profits are shared in the ratio of 12. Given, B + C = 100 and A + C= 150
their investments ; A = 2 B,
ᇱ         ; 2B + C = 150
 ᇱ        O B + (B + C) = 150 (Since B + C =100)
Money invested by A and B for the same ; B = 150 - 100 = 50
Period ; A + B+ C = (A +C) + B
# #  = 150 + 50 = Rs. 200.
" " "
'  #  # 13. Ratio of capitals of A and B is 4 :5 . Let,

;Investment of A =  the capitals of A and B be Rs. 4x and 5x

= Rs. 5000 respectively. Hence, monthly equivalent
8. Ratio of capitals of A, B and C of investment of A.

= (18000 × 10): (24000 × 6) : (30000 × =/ $90 & {# $9 |" 9

4) = 15 : 12 : 10
www.sakshieducation.com
www.sakshieducation.com Quantitative Aptitude

 17. Let their initial investments be 4x, 3x and


(Since A invested 4x for 7 months)

2x
Similarly, monthly equivalent of
Ratio of the capitals of A, B and C is
investment of B,
$   

  : 
"/ 90 & Œ# 9 "$ 9 
     :  
 
(since B invested 5x for 7  

months)   

}6KR3q Fg)67 KR ‚ 9 = (16 x + 16 x) : (12 x + 12 x) :
; "
D6KR3q Fg)67 KR † $ 9 (14 x + $90 "  9 < $9 < ' 9
; }6KR3q KR ‚ "

#% =Rs. 330 " '< %< #
 
 B’s share = Rs. G % H "Rs. 3600
; Profit of B " #% "Rs.430 
 18. Since 60% of the profit is distributed
14. Ratio of capital of Radhika and Renuka = equally. So, one partner receives Rs. 30
50000 : 70000 = 5 : 7 more than the other only due to
Share of Renuka for maintaining the distribution of rest 40% of the basis of

business = Rs. G H " Rs. their invested capitals.

3000 ‚[ F $ % D6KR3q  
; [ " "
Remaining profit = Rs. (30000 - 3000) † F$ % D6KR3q ' #
= Rs. 27000 ‚[ F$ % D6KR3q & †[ F$ % D6KR3q
O [
Renuka’s share in the remaining ‚ F $ % D6KR3q – †[ F $ % D6KR3q
profit  & #
 "
= Rs. G # H = Rs. 15750   #
 (By componendo and dividend)
Hence, Renuka’s total share = Rs. $ % KR D6KR3q $
(15750 + 3000) = Rs. 18750 O "
'
15. After giving 25% of the total profit  
amount to A for managing the business, O }6KR3q " "Rs. 393.75
 
the rest 75% of total profit is divided ;Total profit is Rs. 393.75.
among A, B and C in proportion to their 19. Let the initial investments of A and B be
investments. In 75% of total profit Rs. 2x and Rs. 3x respectively. Let B
A’s share : B’s share : C share invest for a period of y months. Then,
= 4000 :5000 :6000 ratio of capitals of A and B = (2x × 9):
; 75% of total profit = 4x + 5x + 6x (3x× y) = 1 : 2
 '9
; Total profit " = 20x O "
%
9 = 
; Share of A = 4x + 25% of 20x = 9x %
Share of B = 5x = 5x O " O=" 
Share of C = 6x = 
Given, 20. Let A and B leave the business after x
/9 & %90  9 "  months Ratio
of capitals of A, B and C is
O x = 100
= 90000 × x : 60000 × x : 450000 × 12
; Total profit = 20x = 20 × 100 = Rs. 2000
 = 3x : 2x : 18
16. Interest paid to P= ' = 400 But, 3x: 2x : 18 = 6 : 4 : 9

Interest paid to Q =

54000 = 2 :1 9 $
 O "
Net profit for distribution '
' $
= Rs. (10000 - 600) = Rs. 9400 O9" "$
Ratio of profit = 8000 : 4000 = 2 :1 

Hence, A and B left the business after 4
; Share of P = 9400 = Rs. 6267 months.

and Share of Q = 9400 = Rs. 3133 21. Ratio of shares of A, B, C and D


www.sakshieducation.com
www.sakshieducation.com Quantitative Aptitude

= (12 × 5): (20 × 6): (15 × 5): (30 × 6)


= 60 :120 :75 :180
=4:8:5:12
Let the total rent for the parking place be Rs.
x

Then, A’s share = Rs. G H

$9
; " $

$ 
O9" " #$
$
22. Ratio of capitals of A, B and C
= (15000 × 3): (40000 × 9) : (30000 × 6)
=1:8:4

B’s share = Rs.G #' H = Rs. 4800


www.sakshieducation.com
www.sakshieducation.com Quantitative Aptitude

13. ALLEGATION OR MIXTURE


Introduction
It is the rule that is used to determine the mean value of the mixture when the prices of the
individual items being mixed together and the proportion in which they are being mixed are given.
Here, the value of the mixture is always higher than the lowest value and lower than the highest
value of the items being mixed.
According to Rule of Allegation
•i).q3q= KR +g7)D76 }63+7 KR 27)676  c7). D63+7
"
•i).q3q= KR 27)676 c7). }63+7  }63+7 KR `g7)D76
It can be also expressed as,
Cost price of 1 unit
Cost price of 1 unit quantity of dearer (y)
quantity of cheaper (x)
Mean price (m)

(m-x)
(y-m)

(Cheaper quantity : Dearer quantity) = (y - m) : (m - x)


Where, mean price (m) is the cost price of a unit quantity of the mixture.
Also, if a container contains x units of liquid from which y units are taken out and replaced by water.

After n operations, the quantity of pure liquid is {9 G & H | unit.

Example 1 : How many kilograms of rice costing Rs. 18 per kg must be mixed with 30 kg of rice
costing Rs. 14 per kg, so that the resultant mixture cost Rs. 15 per kg.
Solution: Applying the rule of allegation, we have
(` 14) (` 18)

Mean price ( ` 15)

18-15=3
15-14=1

•i).q3q= KR +g7)D76 63+7


"
•i).q3q= KR 27)676 63+7
; If cheaper rice is 3 kg, dearer rice is 1 kg.

If cheaper rice is 30 kg, dearer rice = / 0 kg = 10 kg

Example 2: In what proportion must a person mix rice Rs.12.00 per kg and Rs. 14.40 per kg so as
to make a mixture worth Rs. 12.60 per kg?
Solution: CP of 1 kg dearer
1200 paise rice 1440 paise

Mean price
1260 paise
180
60

By the allegation rule,


•i).q3q= KR +g7)D76 63+7 '
" "
•i).q3q= KR 27)676 63+7 %
; He must mix rice in the ratio 3 : 1.
Example 3: In what proportion must water be mixed with milk to gain 20% by selling it at cost
price?
www.sakshieducation.com
www.sakshieducation.com Quantitative Aptitude

CP of 1 L of water CP of 1 L of milk
`0 `1

Mean price

`

 
 

Solution: Let CP of milk = Rs. 1 per litre


; SP of 1 L of mixture = Rs. 1,
Profit = 20%

; CP of 1 L mixture = = Rs.
 

; Required ratio = : = 1 : 5
 

Example 4: Two vessels contain mixture of milk and water in the ratio of 3 : 5 in the first vessel and
in the ratio of 2 : 7 in the second. In what ratio should the contents of these two vessels be mixed
such that the resultant mixture has milk and water in the ratio 1 : 3?
Solution . Here, we can apply the allegation rule taking the concentration of the mixtures. The

concentration of milk in the first vessel is 3/8 and that in the second is

Concentration of milk in Concentration of milk in
૜ ૛
first vessel   second vessel  
ૡ ૢ

Mean concentration of milk in the



new mixture  

     
   
   

The ratio in which the two mixture should be mixed is — : " ': % " 2 : 9
 

EXERCISE
1. In what ratio, tea at Rs. 90 per kg should first group to the number of students in
be mixed with another tea at Rs. 120 per the second group.
kg to get a tea of Rs.100 per kg (a) 2 : 1 (b) 3 : 2
(a) 2 : 1 (b) 3 :1 (c) 3 : 1 (d) 4 :3
(c) 3 : 2 (d) 4 :3 4. On mixing two classes of students having
2. Two varieties of rice at Rs. 10 per kg average marks 25 and 40 respectively,
and Rs.12 per kg are mixed together in the overall average obtained is 30 marks.
the ratio 1 : 2. Find the average price of Find the number of students in first class
the resulting mixture if the second class has 30 students.
(a) Rs.11 per kg (b) Rs. 11.22 per kg (a) 45 (b) 60
(c) Rs. 11.33 per kg (d) Rs. 11.44 (c) 70 (d) 80
per kg 5. 4 kg of rice at Rs. 5 per kg is mixed with
3. On combining two groups of students 8 kg of rice at Rs. 6 per kg. Find the
having 30 and 40 average marks average price of the mixture
respectively in an exam, the resultant (a) Rs. 5 per kg (b) Rs. 5.55 per kg
group has an average score of 34. Find (c) Rs. 5.66 per kg (d) Rs. 6 per
the ratio of the number of students in the kg
www.sakshieducation.com
www.sakshieducation.com Quantitative Aptitude

6. 5 kg of rice at 6 per kg is mixed with 4 14. One type of liquid contains 25% of milk,
kg of rice to get a mixture costing Rs. 7 the other contains 30% of milk. A
per kg. Find the price of costlier rice. container is filled with 6 parts of the first
(a) Rs. 7.00 (b) Rs. 7.50 liquid and 4 parts of the second liquid.
(c) Rs. 8.00 (d) Rs. 8.25 The percentage of milk in the mixture is
7. In what proportion must water be mixed (a) 27% (b) 31%
with spirit to gain  % by selling it at (c) 29 % (d) 33%
 15. A mixture of 45 L of spirit and water
CP?
contain 20% of water in it. How much
(a) 2 :7 (b) 1: 8
water must be added to it make the water
(c) 1: 9 (d) 2: 9
25% in' the new mixture?
8. In what ratio a grocer mix tea at Rs. 22
(a) 5 L (b) 3 L
per kg and Rs. 32 per kg, so that by
(c) 4 L (d) 6 L
selling that mixture at Rs. 28 he may 16. A mixture of 20 kg of spirit and water
gain 12%? contains 10% water. How much water (in
(a) 3 : 5 (b) 7 : 3 kg) must be added to this mixture to raise
(c) 8 : 3 (d) 3 : 11 the percentage of water to 25%?
9. A mixture of 70 L of milk and water (a) 30 (b) 8
contains 10% water. How many litres of (c) 5 (d) 4
water must be added, so that water may 17. A person travels 340 km in 8 h. The first
be 30% of the mixture? part of the journey, he travels by car at
(a) 35 L (b) 33 L the speed of 50 km/h and in the second
(c) 20 L (d) 25 L part of the journey he travels by train at
10. Gold is 20 times heavier than water and the speed of 38 km/h. How many km did
copper 9 times. In what ratio should he travel by train?
these metals be mixed so that the mixture (a) 120 km (b) 190 km
may be 15 times as heavy as water? (c) 150 km (d) 210 km
(a) 6 : 5 (b) 3 : 2 18. A merchant lent out Rs. 6440 in two
(c) 5 : 1 (d) 4 : 3 parts, one at 8% and the other at 12%
11. A mixture of 150 L of wine and water interest. If the yearly average interest
contains 20% water. How many litres of comes out to be 9%. Find the amount lent
water must be added so that water may at 12% interest?
be 25% of the mixture? (a) Rs. 1610 (b) Rs. 4830
(a) 12 L (b) 8 L
(c) Rs.2640 (d) Rs. 3610
(c) 10 L (d) 6 L
19. A sum of Rs. 7.50 is made up of 21
12. Pure milk costs Rs. 21.50 per litre. A
coins which are either 25 paise or 50
milk man adds water to 60 L of pure milk
paise coins. How many coins are there of
and the resultant mixture costs Rs. 20
50 paise?
per litre. How many litres of water does
(a) 9 (b) 12
he add?
 (c) 7 (d) 10
(a) L (b)  L 20. In a zoo, there are some pigeons and
 
 some rabbits. If their heads are counted
(C) $ L (d) L
  these are 100 and if their lE.g.s are
13. A dealer mixes tea costing Rs. 50 per kg
counted these are 320. How many
with a high quality tea and sells the
pigeons are there?
mixture at Rs. 54 per kg. If the ratio in
(a) 66 (b) 60
which the two quality tea were mixed is (c) 40 (d) 45
2 : 1, what is the cost of higher quality
tea? Answer Key
(a) Rs. 62 per kg (b) Rs. 58 per kg 1 a 5 c 9 c 13 a 17 b
(c) Rs. 66 per kg (d) Rs. 72 per kg 2 c 6 d 10 a 14 a 18 a
3 b 7 b 11 c 15 b 19 a
4 b 8 b 12 c 16 d 20 c
www.sakshieducation.com
www.sakshieducation.com Quantitative Aptitude

SOLUTIONS
1.
` 120
` 90
Mean price
` 100
10
20    
  
Required ratio = 20 : 10 = 2 : 1 # # # #
! $
2.
` 12
Ratio of water and spirit = : = 1 : 8
% %
` 10 8. SP of 1 kg mixture = Rs. 28 Gain = 12%
Mean price !
`
CP of 1 kg mixture = Rs. %   &
!!
x-10
12-x =Rs. 25
By the rule of allegation, CP of 1 kg
       dearer tea
 CP of 1 kg cheaper
       ` 32
    (` 22)
 
    Mean price
  Rs. 11.33 per kg ` 
25-22=3
 32-25=7
3. Required ratio =
 Ratio of cheaper tea and dearer tea =7 : 3
 
  9. The mixture contains 10% water and 90%
  milk
     
4. By the rule of allegation
   
    
   Milk of 90% Milk of 0%(water
    
70% milk
 Students in first class =  30 = 60
!
5. Using the cross method if average price is 70-0=70 90-70=20
Rs. x per kg.
Ratio of milk and water = 70 : 20 = 7 :
Then
  2
 For 7 L milk, 2 L water is to be added.
  &'
        ' For 70 L milk, % &L = 20 L water is
&
    to added.
 x= Rs. 5.66 per kg 10. By the rule of allegation,
6.
`x Gold 20 Copper 9
`6
Mean price Mean 15
`7
4 15-9=6 20-15=5
5
By the cross method, Ratio of gold and copper = 6 : 5
  11. The mixture contains 20% water and 80%
      
  of wine.
      Rs. 8.25 By the rule of allegation,
7. Let the CP of the spirit be Rs. 1 per litre Wine of 80% Wine of0%(water

Then SP of 1 L of mixture = Rs. 1 per litre 75% Wine


! "
Gain   %  %
75-0=75 80-75=5
! $
CP of 1 L of mixture = Rs. ! ૚ " Rs. Ratio of wine : water = 75 : 5 = 15 : 1
!#! %
૛ For 15 L wine, 1 L water is to be added.
By the rule of allegation. !"'!
' For 150 L wine,% &L=10 L to be added.
!"

12. By the rule of allegation


`1
(` 0)
www.sakshieducation.com
Mean price

`

www.sakshieducation.com Quantitative Aptitude

Cost per litre of pure milk # 


:   : 
2150 pasie Water 0 pasie  
"
Time spent in train =  8 = 5 h
Mean price $
2000 paise Distance travelled by train =38×5= 190 km
2000-0=2000 2150-2000=150
18. By the rule of allegation
()*+,-,. /0 1)23 4-56  
  Part I 8% Part II 12%
()*+,-,. /0 7*,32  
For every 40 L of pure milk he adds 3 L of Mean
interest 9%
water. 12-9=3 9-8=1
(' %
For 60 L of milk he adds =  Ratio of Part I : Part II = 3 : 1

! Amount lent on 12% interest
= 4 L of water !
13. Let the rate of second quality be Rs. x per =  6440 = Rs. 1610

&" "
kg. 19. Average price = paise = paise
! &
CP of 1 kg tea of 1st Kind CP of 1 kg tea of 2nd Kind By the rule of allegation
` 50 `x
25 pasie coins 50 pasie coins

Mean price Mean price


` 54 ૛૞૙
paise

(x-54) 54-50=4  
 
 
)"
Now  Ratio of 25 paise coins to 50 paise coins

!
! &"
           : =4:3
& &
    8    
∴ no. of 50 paise coins =    #
' CP of second quality tea is Rs. 62 per kg &
"  20. Since there are 100 heads, it means that the
14. Milk in mixture =   8 
! ! number of pigeons and rabbits are 100. A
&
=27% pigeon has 2 legs and a rabbit has 4 legs. If
!
15. 45 L of mixture has 20% water so 45 L of all are pigeons, number of legs = 2 × 100 =
mixture has 9 L of water. suppose x litres 200
must be added. If all are rabbits, number of legs
Then = 4 × 100 = 400
8# Actual number of legs = 320
    By the rule of allegation, Rabbits
 8 
 8    8     9 Pigeons
400
16. Water in mixture = 2 kg
200
Let x kg of water should be mixed. Mean price
8 8  320
'      
 8   8   320-200=120
  8    8  400-320=80

       
17. Average speed of the journey Ratio of pigeons and rabbit
  = 80 : 120 = 2 : 3
  64/; Number of pigeons =  100 = 40
  "
By the rule of allegation
Speed of car (50 km/h) Speed of train (38 km/h)

Average
Speed
 
ૡ૞
km/h  
 ૛ 
   

Time spent in car : Time spent in train

www.sakshieducation.com
www.sakshieducation.com Quantitative Aptitude

14.TIME AND WORK


Work
Work to be done is generally considered as one unit. It may be digging a trench, constructing
or Painting a wall, filling up or emptying a tank, reservoir or a cistern.
General rules to be followed in the problems on Time and Work
1. If A can do a piece of work in n days, then work done by A in 1 day is 1/n.
i.e. if a person can do some work in 12 days, he does 1/ 12th of the work in one day.
2. If A’s 1 day’s work =1/n , then A can finish the whole work in n days.
i.e. if a person’s one day’s work is 1/10, then he can finish the whole work in 10 days.
3. If A is thrice as good a workman as B, then ratio of work done by A and B = 3 : 1. i.e. if a man
works three times as fast as a woman does, then when the work is complete, 3 parts of
the work has been done by the man and 1 part by the woman.
4. If A is thrice as good a workman as B, then ratio of time taken by A and B = 1 : 3. i.e. if the
woman takes 15 days to complete the work, then the man takes 5 days to complete the same work.
5. If two persons A and B can individually do some work in a and b days respectively, then A and B
together can complete the same work in ab/(a + b) days.
6. The fundamental rules on variation also apply in Time and Work.
(i) Work and men are directly proportional to each other i.e. if the work increases, the no. of men
required to do it, also increases, if the work is to be completed in the same number of days.
(ii) Men and days are inversely proportional, i.e. if the number of men increases, the number of days
required to complete the same work decreases and vice versa.
(iii) Work and days are directly proportional, i.e. if the work increases, the number of days required
also increases, if the work is to be completed with the same number of men and vice versa.
Example 1: Ravi can do a job 10 days. Determine his one day job.
Solution: Ravi’s 10 days work = 1

Ravi’s 1 day work =

Example 2: Tuktuki and Rasmani can do a job alone in 20 days and 30 days respectively. In how
many days the job will be finished, if they work together.

Solution: Tuktuki’s 1 day work =


Rasmani’s 1 day work =

   
(Tuktuki + Rasmam)’s 1 day work =   
   
 (Tuktuki + Rasmani) will complete the job in 12 days.

Alternate: Required no. of days = =12
 
Example 3: Mary and Maurice can do a piece of work in 10 days and 15 days respectively. They
work together for 3 days and then Maurice leaves. Mary finishes the remaining work alone. In how
many days is the total work finished?
   
Solution . (Mary + Maurice)’s 1 day work =   
   
 
(Mary + Maurice)’s 3 day work = =
 
 
Remaining work =   =
 
Mary does this piece of work in = 10 days

Mary does the 1/2 piece of work in =  10 = 5 days

Hence, total number of days to finish the work = 3 + 5 = 8 days
Example 4: Ravi and Rishi can do a piece of work in 24 days, Rishi and Ronit in 30 days, Ronit and
Ravi in 40 days. In how many days will they finish it together and separately?
Solution: (Ravi and Rishi)’s 1 day’s work =1/24
(Rishi and Ronit)’s 1 day’s work = 1/30

www.sakshieducation.com
www.sakshieducation.com Quantitative Aptitude

(Ronit and Ravi)’s 1 day’s work = 1/40


On adding, we get
2 (Ravi + Rishi + Ronit)’s 1 day’s work = (1/24 + 1/30 + 1/40) = 1/10
So,(Ravi + Rishi + Ronit)’s 1 day’s work = 1/20
So, they all together can finish the work in 20 days.
Ravi’s 1 day’s work = [(Ravi + Rishi + Ronit)’s 1 day’s work] - [(Rishi + Ronit)’s 1 day’s work] =
(1/20 -1/30 ) = 1/60
So, Ravi alone can finish the work in 60 days.
Similarly, Rishi’s 1 day work = (1/20 - 1/40) = 1/40
So, Rishi alone can finish the work in 40 days
And Ronit’s 1 day work = (1/20- 1/24) = 1/120
So, Rishi alone can finish the work in 120 days.
Example 5: 4 men and 5 women do a piece of work in 8 days while 5 men and 5 women finish it in
6 days, 3 men and 5 women will finish it in how many days?
Solution: (4 men + 5 women)’s 1 day’s work = 1 / 8
(5 men + 5 women)’s 1 day’s work 1/6
Subtracting, 1 man’s 1 day’s work = (1/6 - 1/8) = 1/24
(3 men + 5 women)’s 1 day’s works = (1/8 - 1/24) = 1/12
Thus, 3 men and 5 women will finish it in 12 days.

EXERCISE
Directions (1-5): A can do a piece of work how many days, B alone will be able to
in 8 days and B in 12 days. Find how much complete the same work?
time they will take to complete the work (a) 80 days (b) 84 days
under the following condition. (c) 88 days (d) 92 days
1. Working together 7. A and B can do a piece of work in 12
(a) 4 days (b) 4 days days, B and C can do it in 15 days and C

  and A can do the same work in 20 days.
(c) 4 days (d) 4 days Find the number of days in which A
2. Working alternately starting with A alone can do the same job.
 (a) 20 days (b) 30 days
(a) 8 days (b)8 days

 (c) 45 days (d) 60 days
(c) 9 days (d) 9 days 8. A can do a piece of work in 15 days and

3. Working alternately starting with B B alone can do it in 10 days. B works at it
 for 5 days and then leaves. In how many
(a) 8 days (b) 8 days

  days, A alone can finish the remaining
(c) 9 days (d) 9 days work?

4. If B leaves 3 days before the actual 
(a) 7 days (b) 7 days
completion of the work 

(a) 2 days (b) 3 days (c) 8 days (d) 8 days

(c) 6 days (d) 9 days 9. A and B can do a piece of work
separately in 8 and 12 h. If they work
alternate hours, A starting when will the
5. If B leaves 3 days before the scheduled work be finished?
completion of the work 
(a) 9 h (b) 9 h
(a) 3 days (b) 6 days 

(c) 8 h (d) 8 h
(c) 3 days (d) 6 days 
 
10. Two friends A and B working together
6. A and B together can complete a piece of
completed a work in 26 days. Their skills
work in 35 days while A alone can
of doing the work is in the ratio 8: 5.
complete the same work in 60 days. In
How many days will B take, if engaged
alone?
www.sakshieducation.com
www.sakshieducation.com Quantitative Aptitude

 17. A alone can do a piece of work in 6 days


(a) 67 days (b) 62 days

 and B alone in 8 days. A and B undertook
(c) 67 days (d) 65 days to do it for Rs. 3200, with the help of C,

11. A and B together can do a piece of work they completed the work in 3 days. How
in 12 days, which B and C together can much is to be paid to C?
do in 16 days. After A has been working (a) Rs.450 (b) Rs.400
at it for 5 days and B for 7 days, C (c) Rs.600 (d) Rs.900
finishes it in 13 days. In how many days 18. A sum of money is sufficient to pay A’s
C alone will do the work? wages for 21 days and B s wages for 28
(a) 16 (b) 24 days. The same money is sufficient to
(c) 37 (d) 48 pay the wages of both for:
12. A and B can do a piece of work in 45 
(a) 12 days (b) 12 days
days and 40 days respectively. They 
bE.g.an to do the work together but A (c) 16 days (d) 24 days

leaves after some days and then B 19. 12 men can complete a piece of work in
completed the remaining work in 23 4 days, while 15 women can complete
days. The number of days after which A the same work in 4 days. 6 men start
left the work was: working on the job and after working for
(a) 11 (b) 7 2 days, all of them stopped working.
(c) 9 (d) 12 How many women should be put on the
13. A can do a piece of work in 14 days job to complete the remaining work, if it
which B can do in 21 days. They bE.g.in is to be completed in 3 days?
together but 3days before the completion
of the work, A leaves off. The total (a) 15 (b) 19
number of days to complete the work is: (c) 23 (d) data inadequate
 
(a) (b) 8 20. Sixteen men can complete a work in
 
  twelve days. Twenty-four children can
(c)  (d) 13 complete the same work in eighteen
 
days. Twelve men and eight children
14. A, B and C can complete a work started working and after eight days three
separately in 24, 36 and 48 days more children joined them. How many
respectively. They started together but C days will they now take to complete the
left after 4 days of start and A left 3 days remaining work?
before the completion of the work. In (a) 5 days (b) 4 days
how many days will the work be (c) 10 days (d) 9 days
completed? 21. 5 men and 2 boys working together can
(a) 15 days (b) 24 days do four times as much work as a man and
(c) 25 days (d) 38 days a boy. Working capacities of a man and a
15. A, B and C together earn Rs. 300 per day, boy are in the ratio:
while A and C together earn Rs.188 and (a) 1 :2 (b) 2 : 1
B and C together earn Rs.152. The daily (c) 1: 3 (d) 6 : 7
earning of C is: 22. A and B can complete a piece of work in
(a) Rs.40 (b) Rs. 70 80 and 120 days respectively. They
(c) Rs. 112 (d) Rs. 160 together start the work but A left after 20
16. A, B and C are employed to do a piece of days. After another 12 days C joined B
work for Rs. 529. A and B together are and now they complete the work in 28
 more days. In how many days C can
supposed to do of the work and B and

complete the work, working alone?
C together of the work. What amount (a) 100 days (b) 112 days

should A be paid? (c) 120 days (d) 126 days
(a) Rs. 320 (b) Rs. 345 23. 8 men and 10 women are working
(c) Rs. 355 (d) Rs.380 together on a job and can complete it in

www.sakshieducation.com
www.sakshieducation.com Quantitative Aptitude

10 days. The work done by each woman and one model C tractor take 3 days to do
is four-fifth of the work by each man. In this job. Three tractors on each of models
how many days 10 men alone complete A, B and C take 4 days to do the same
the same work? task. How long will it take to do the job
(a) 20 days (b) 12 days if a team is made up of four tractors of
(c) 15 days (d) 16 days different models?
 
(a) days (b) days
24. There are 2 machine tools. The capacity  
of the 1st machine tool is 20% less than (c) days (d) days
the 2nd one. The 1st machine tool 26. The efficiency of Ronil is twice that of
operated for 5 h whereas the 2nd one Sanjay and he can finish a work in 4 h
operated for 4 h and together they less than the time taken by Sanjay. If
machined 3000 work pieces. The number both of them work together they can
of work pieces machined by the 1st finish the same work in
machine is (a) 8 h (b) 3 h
(a) 950 (b) 1000 (c) 8/3 h (d) 4/3 h
(c) 1032 (d) 1050
25. A firm has tractors of four models A, B,
C and D. Four tractors (two of model B
and one each of models C and D) plough
a field in 2 days. Two model A tractors

Answer Key
1 b 7 b 13 c 19 a 25 d
2 d 8 b 14 a 20 b 26 c
3 c 9 a 15 a 21 b
4 c 10 a 16 b 22 b
5 d 11 b 17 b 23 d
6 b 12 c 18 a 24 b

SOLUTIONS
! ! !
1. A’s one day work = =    day
$ 
! !
B’s one day work = So, total required time = 
!
! ! " !
(A + B)’s one day work =  =  days
$ ! 
 3. Here, there will be no difference in work
 Required number of days =
"
 completed by the 8th day.
  days On the 9th day B works alone and does
"
2. Work done in first 2 days !
 of the work
! ! " !
=   So, after 9 days remaining work to be done
$ ! 
 Work done in first 8 days " ! !! !
   
"  ( ! ! !
=  Time taken by A to do the remaining work
 
  ! !
Remaining work =       days
  ( ! 
!
Work done by A in 9th day = So, total required time =8+1+ =9
$  
After 9 days, the remaining work has 4. If B leaves 3 days before the actual
! ! !
to be done   completion of the work, then A will work
( $ 
alone for the last 3 days.
In the 10th day B will come to do the 
work and he will complete all the So, in last 3 days work done by A =
$
remaining work. Time taken by B for it
www.sakshieducation.com
www.sakshieducation.com Quantitative Aptitude

 " "
 A and B have done    of the work  Work done by B is of the whole which
$ $ !
 Time taken by A and B together to do 5/8 of he does in 26 days
the work = 
" ! '$
= 3 days  Number of days taken by B to do the
$ ! #$ ('! 
 Total required time = 3 + 3 = 6 days whole work  = 67 days
" 
5. Time taken by A and B working together to
 11. A’s 5 days’ work + B’s 7 days’ work + C’s
finish the work =  days
"
 ! 13 days work = 1
B’s work of 3 days =   (A + B)’s 5 days’ work + (B + C)’s
! 
Since, B leaves 3 days before the 2 days’ work + C’s 11 days’ work = 1
scheduled completion of the work. So, this "
   C’s 11 days’ work = 1
! ! !(
work     be done by  C’s 11 day’s work

A. So, time taken to finish 1/4 of the work " !!
=  
by ! !( 
!! ! !
!
A =  = 2 days  C’s 1 day’s work =  
 !! 

 Total required time C alone can finish the work in 24 days.
  12. (A + B)’s 1 day’s work
      =
! !
 
!&
  "  (
! ! ! & " !
6. B’s one day work = - = = = Work done by B in 23 days
" ( (   $
! 
 Required number of days =84 =  ! 
 
!
7. Work done by A and B in one day=  !&
! Remaining work =  
!  
Work done by B and C in one day = Time taken by A and B to finish the
!"
! ( !&
Work done by C and A in one day = remaining work=    
 !& 
Adding all these, 2 (A + B + C)’s work of  A left after 9 days.
! ! ! ! ! !
one day =    13. B’s 3 day’s work   ! 
! !"  " ! &
! ! (
(4 + B + C)’s one day work =
!
Remaining work =  
& &
∴ A’s one day work = (A + B + C)’s one (A + B)’s 1 day’s work
day work - (B + C)’s one day work   
 A’s one day work #  $
  
   (
 work is done by A and B in
  &
  !   !
Hence, A can do this work in 30 days. #  $ 
! !  " 
8. Work done by B in 5 days =5* = (
! !
! Hence, total time taken = !   days
"
Remaining work=1-  !
= 10 days.
Time taken by A to finish the remaining "
! ! 14. (A+ B + C)’ s 1 day’s work
work=   "     !
! ! " #   $
9. (A + B)’s 2 h work =  =  !  
$ ! 
"' " Work done by (A + B + C) in 4 days
(A + B)’s 8 h work =  ! !

!
( =   
! (
Work done by A in 9th h =
$ Work done by B in 3 days
" !  ! !
Total work done up to 9 h =  =
th
=  ! 
( $  ( !
!
Remaining work = ! ! "
 Remaining work = %  & 
! ( ! %
B’s 1 h work = (A + B)’s 1 day’s work
!
! ! !   
B can do of the work in    #  $
 
!  ! "
∴ Both can finish the job in 
10. Skill ratio of A and B = 8 : 5

www.sakshieducation.com
www.sakshieducation.com Quantitative Aptitude

Time taken by A and B to finish 5/9th of !%


 work will be done by them in
" & "
the work= ' =8 days !( !%
% "   = 4 days.
Hence, total time taken = (4+ 3+8) days = !%
"
21. Let 1 man’s 1 day’s work = x and 1 boy’s 1
15days.
day’s work = y.
15. B’s daily earning = Rs. (300 - 188)
Then, 5x + 2y = 4 (x + y)
= Rs.112.
= Rs. [152 - 11)]= Rs.40. x =2y
$ !" )
16. Work done by A =    
   
!" $
A : (B +C) = : = 15 : 8 22. A did the work for 20 days, B did the work
 
!" for (20 + 12 + 38) = 60 days, C did the
So, A’s share =Rs.   = Rs. 345. work for 28 days.

! ! !
17. C’s l day’s work =   Let C alone can complete the work in x
 ( $
 "  days.
  Fraction of work did by A + Fraction of
!   work did by B + Fraction of work did by C
A’s wages : B’s wages : C’s wages
   =1
 : :  : !:  Or


(
 
$
  $ !  )
!
 C’s share = Rs.  !  = Rs. 400. Or )  
$
18. Let total money be Rs. x. = 112 days
) C alone can complete the work in 112 days
A’s 1 day’s wages = Rs. 
! 23. 1 Woman = men
) "
B s 1 day’s wages = Rs.
$ 8 Men + 10 Women
 (A + B)’s 1 day’s wages 
) ) ) = 8 Men + 10 Men = 16 Men
"
= Rs.   = Rs.
! $ ! 16 Men can complete the work in 10 days.
∴Money is sufficient to pay the wages of 10 Men can complete the work in

both for 12 days. = 16 days
! 
19. 1 man 1 day’s work  ; 24. Let the capacity of the second machine be
$
! x work pieces per hour.
1 woman 1 day’s work=
( Capacity of the 1st machine tool
( !
6 men’s 2 day’s work =   = 0.8 x.
$ 
!   * . ),  ) 
Remaining work =  
   ) 
!
Now,
(
work is done in 1 day by 1  )  
woman. Number of work machined by the first
 Tool = 5 × 0.8 x = 1000
So, work will be done in 3 days by ! ! !

 ! 25.    …………….(i)
* + ,
    = 15 women. ! !
    ………………(ii)
! - + 
20. 1 man’s 1 day’s work = ; ! ! ! !
!% and    ………...(iii)
! - * + 
1 child’s 1 day’s Work=
 Subtracting Eq. (i) from Eq. (ii), we
! $ ! !
Work done in 8 days =   get  ...(iv)
!%  - * , (
! ! "
   Subtracting Eq. (iii) from Eq. (ii), we
!( " "
" !% get
Remaining work =     
" "

(12 men + 11 children)’s 1 day’s work - . 
   
!
 …………..(v)
#  $ - * (
 !  Subtracting Eq. (v) from Eq. (iv), we
!%
Now, work is done by them in 1 get
!(
! !
day.  ……….............(vi)
, 

www.sakshieducation.com
www.sakshieducation.com Quantitative Aptitude

Adding Eqs. (iii) and (iv), we get


      "
     
- . / 0  ! 
!
Hence, A, B, C and D will take days to
&
do the required job.
26. As the efficiency of Ronil is twice that of
Sanjay, Ronil will take hours for
completion of the work, when Sanjay is
taking 2x h.
Given, 2x – x= 4=> x= 4
! $
Hence, they take ૚ ૚  h
# 
૝ ૡ

www.sakshieducation.com
www.sakshieducation.com Quantitative Aptitude

15. PIPES AND CISTERNS


Pipes and Cisterns
A pipe or a tap connected with a tank (or a cistern or a reservoir) is called an inlet, if it fills it.
A pipe or a tap connected with a tank (or a cistern or a reservoir) is called an outlet, if it empties it.
General rules to be followed in the problems on Pipes and Cisterns
1. If a pipe can fill a tank in a hours, then the part filled in 1 h =1/a
2. If a pipe can empty a tank in b hours, then the part of the full tank emptied in 1 h = 1 / b
3. If a pipe can fill a tank in a hours and another pipe can empty the full tank in b hours, then the net
part filled in 1 h, when both the pipes are opened = (1/a -1/b)
Thus, in Pipes and Cisterns a filling pipe or tap does positive work and an emptying pipe or a leak
does negative work.
 
Example 1: A pipe can fill of cistern in 12 min. In how many minutes it fill of cistern.


Solution: Time taken to fill of cistern = 12 min
Time taken to fill full cistern = 12  = 16min


Time taken to fill 1/2 of cistern =  16 =8 min

Example 2: A tap can fill a cistern in 20 min and another tap can fill in 30 min. If both are opened
simultaneously, find the time when the cistern will be full.
    
Solution . Total part that can be filled in 1 min =   
   
Hence Tank will be filled in 12 min.
Example 3: Pipe A can fill a tan k in 15 min, pipe B in 18 min and a leak can empty the tank in 90
min. If all of them work together, find the time taken to fill the empty tank?
Solution: Net part filled in 1 min = (1/15 + 1/18 - 1/90) = 1/9
So, the empty tank will be filled in 9 min.
Example 4: Two taps A and B can fill a tank in 20 and 40 min respectively. Both the taps are opened
and after 10 min tap A was shut. In how much more time will the tank be full?
Solution: Part filled by (A+ B) in 10 min = 10 (1/20 + 1/40) = 10-3/40 = 3/4
Part of the tank to be filled = (1- 3/4) = 1/4
Now, l/4th of the tank can be filled by B in (l/4)*40 = 10 min
So, the tank would be full in 10 more min.

EXERCISE
1. Two pipes A and B can fill a tank in 6 3. A pump can fill a tank with water in 2
hours and 4 hours respectively. If they are 
hours. Because of a leak, it took 

opened on alternate hours and if pipe A is
hours to fill the tank. The leak can drain
opened first, in how many hours, the tank
all the water of the tank in:
shall be full?

(a) 43 hrs (b) 9 hrs
(a) 8 (b)  (c) 10 hrs (d) 14 hrs

 4. Two taps A and B can fill a tank in 5
(c) 5 (d) 

hours and 20 hours respectively. If both
2. Two pipes A and B fill a tank in 15 h and
the taps are open then due to a leakage, it
20 h respectively while a third pipe C can
took 30 minutes more to fill the tank. If
empty the full tank in 25 h. All the three
the tank is full, how long will it take for
pipes are opened in the beginning. After
the leakage alone to empty the tank?
10 h, C is closed. In how much time will
(a) 44 hrs (b) 12 hrs
the tank be full?
(c) 18 hrs (d) 36 hrs
(a) 10 h (b) 11 h
(c) 11.5 h (d) 12h

www.sakshieducation.com
www.sakshieducation.com Quantitative Aptitude

5. Two pipes A and B together can fill a pump needs 8 minutes lesser to empty
cistern in 4 hours. Had they been opened the tank than it needs to fill it. What is
separately, then B would have taken 6 the filling capacity of the pump?
hours more than A to fill the cistern. How (a) 50 m3/min (b) 80 m3/min
much time will be taken by A to fill the (c) 36m3/min (d) 48m3/min
cistern separately? 11. A leak in the bottom of a tank can empty
(a) 10 hr (b) 4 hrs the full tank in 6 h. An inlet pipe fills
(c) 6 hrs (d) 8 hrs. water at the rate of 4 L a minute. When
6. One pipe can fill a tank three times as the tank is full, the inlet is opened and
fast as another pipe. If together the two due to the leak the tank is empty in 8 h.
pipes can fill the tank in 36 minutes, then Find capacity of the tank (in litres).
the slower pipe alone will be able to fill (a) 5000 L (b) 5670 L
the tank in: (c) 5700 L (d) 5760 L
(a) 92 min (b) 112 min 12. A tap can fill a cistern in 12 h. After half
(c) 144 min (d) 192 min the tank is filled, 2 more similar taps are
7. A tank is filled in 5 hours by three pipes opened. What is the total time taken to
A, B and C. The pipe C is twice as fast as fill the tank complete by?
B and B is twice as fast as A. How much (a) 9 h (b) 6 h
time will pipe A alone take to fill the (c) 8 h (d) 4 h
tank? 13. A tank 9 ft by 5 ft by 2 ft has a supply
(a) 22 hrs (b) 27 hrs pipe pouring in 576 of water in a minute
(c) 35 hrs (d) cannot be determined and an exhaust pipe emptying it in 3 h. If
8. A tank is filled by three pipes with the tank is full and both pipes are open,
uniform flow. The first two pipes how many hour will it take to empty it?
operating simultaneously fill the tank in 
(a) 9h (b) 9 h

the same time during which the tank is  
filled by the third pipe alone. The second (c) 9 h (d) 9 h
pipe fills the tank 5 hours faster than the
first pipe and 4 hours slower than the 14. A cistern has three pipes A, B and C. The
third pipe. The time required by the first pipes A and B can fill it in 5 h and 6 h
pipe is: respectively and C can empty it in 3 h. If
(a) 8 hrs (b) 13 hrs the pipes are opened in order at 3 pm, 4
(c) 15 hrs (d) 32 hrs pm an d 5 Pm respectively at what time
will the tank be filled?
9. 12 buckets of water fill a tank when the (a) 6 am (b) 5 am
capacity of each bucket is 13.5 litres. (c) 4 am (d) 7 am
How many buckets will be needed to fill
the same tank, if the capacity of each
bucket is 9 litres?
(a) 9 (b) 112 (c) 24 (d) 18
10. A booster pump can be used for filling as
well as for emptying a tank. The capacity
of the tank is 2400 m3. The emptying
capacity of the tank is 10 m3 per minute
higher than its filling capacity and the

www.sakshieducation.com
www.sakshieducation.com Quantitative Aptitude

Answer Key
1 c 6 c 11 d
2 d 7 c 12 c
3 d 8 c 13 a
4 d 9 d 14 a
5 c 10 a

SOLUTIONS


1. A’s work in 1 hour = ,


B's work in 1 hour =
  
(A + B)’s 2 hour’s work when opened alternately =    
 
 
(A + B)’s 4 hour’s work when opened alternately  
 
 
Remaining part     
 

Now, it is A’s turn and part is filled by A in 1 hour.

Total time taken to fill the tank
= (4 + 1) hrs. = 5 hrs.
  
2. Work done =    
  
Or x = 12 h
  
3. Work done by the leak in 1 hour =    
 
Leak will empty the tank in 14 hrs.
4. Part filled by (A + B) in 1 hour
  
=  
 
So, A and B together can fill the tank in 4 hours.
  
Work done by the leak in 1 hour   

Leak will empty the tank in 36 hrs,
5. Let the cistern be filled by pipe A alone in x hours.
Then, pipe B will fill it in (x + 6) hours.
  
+ =
  
       
     

[neglecting the -ve value of x]
6. Let the slower pipe alone fill the tank in x minutes.

Then, faster pipe will fill it in minutes.

    
   
    
    .
7. Suppose pipe A alone takes x hours to fill the tank.
 
Then, pipes B and C will take and hours respectively to fill the tank.

     
    
     
   
8. Suppose, first pipe alone takes x hours to fill the tank.
Then, second and third pipes will take    and    hours respectively to fill the tank.

www.sakshieducation.com
www.sakshieducation.com Quantitative Aptitude

  
+ =
  
=>    =   
    !   
   " . If   ,    becomes negative. Hence   
9. Capacity of the tank= (12 × 13.5) litres = 162 litres.
Capacity of each bucket = 9 litres.
Number of buckets needed =

    !.
10. Let the filling capacity of the pump be x m3/min.
Then, emptying capacity of the pump
=     ³/.
   
So,   !
 
       
      

[neglecting the -ve value of x]

11. Efficiency of leak =


Combined efficiency of leak and inlet pipe =
   
So, efficiency of inlet pipe    
  
Inlet pipe can fill the tank in 24 h,
So, capacity = 24 × 60 × 4 = 5760 L

12. of the tank can be filled by the tap in 6 h.


Part filled by one tap in 1 h =


Part filled by three taps in 1 h =

tank can be filled by the three pipes


In   = 2 h
Total time taken to fill the tan k completely = 6 + 2 = 8 h
13. Volume of the tank = 9 × 5 × 2 × 12 × 12 × 12 m3
Volume of water exhausted in 1 min

 = 864 m3

Hence, combined effect of the two pipes in 1 min is (864 - 576), ie 288 m3 of water is removed in 1
min.
Time required to empty the tank

 =9h
 
14. By 5 pm, the water filled by two pipes = 2 h work done by pipe A + 1 h work done by pipe B
    
    
  
 
After 5 pm all the three pipes are working and the work done by all the three pipes in 1h   
 
 

 
 
After 5 pm only of the tank is to be filled. time taken in filling of the tank
 

=   = 13 h

13 h after 5 pm= 6 am

www.sakshieducation.com
www.sakshieducation.com Quantitative Aptitude

16. TIME AND DISTANCE


Relation between Time, Speed and Distance
Distance covered, time and speed are related by

'(  ……………..(i)


)*((+  …….………(ii)

, -./(  )*((+  '(………….(iii)
• Distance is measured in meters, kilo meters and miles.
• Time in hours, minutes and seconds.
• Speed in km/h, miles/h and m/s.

1. To convert speed of an object from km/h to m/s multiply the speed by .


0 1*  / 2
!

2. To convert speed of an object from m/s to km/h, multiply the speed by .

!
0/  1*2

Average Speed
It is the ratio of total distance covered to total time of journey.
  !"
Average Speed =
#$%&' %()* $+ ,$-./*0
General Rules for Solving Time & Distance Problems
Rule 1
If a certain distance is covered with a speed of  1/ and another equal distance with a speed of y
km/h, then the average speed for the whole journey is the harmonic mean of the two speeds.
 1
Average speed = 3 4km/h =  km/h
 1
 
Rule 2
If three equal distances are covered by three different speeds x, y and z km/h, then average speed for
the whole journey is given by
 12
Average speed = 3   4km/h =  km/h
1 12 2
  
Rule 3
If a certain distance is covered with a speed of  1/ and another distance with a speed of y km/h
but time interval for both journeys being same, then average speed for the whole journey is given by
 1
Average Speed =  km / h

Rule 4
If a certain distance is covered with a speed of x, y and z km/h, but time interval for the three
journeys being equal, then average speed is given by
 1 2
Average speed =  km/h

Rule 5
If the ratio of speeds A and B is x: y, then the ratio of times taken by them to cover the same distance
 
is :
 1
Relative Speed
(i) If two bodies are moving in the same direction at x km/h and y km/h, where (x > y), then their
relative speed is given by (x - y) km/h.
(ii) If two bodies are moving in the opposite direction at x km/h and y km/h, then their relative speed
is given by (x + y) km/h.

www.sakshieducation.com
www.sakshieducation.com Quantitative Aptitude

General Rules for Solving Circular Tracks


Rule 1
When two people are running around a Circular Track starting at the same point and at the same
time, then whenever the two people meet, the person moving with a greater speed covers one round
more than the person moving with lesser speed.

Rule 2
When two people with speeds of x km/h and y km/h start at the same time and from the same point in
the same direction around a circular track of circumference ‘c’ km, then,

The time taken to meet for the first time anywhere on the track = 
1
 
The time taken to meet for the first time at the starting point = LCM of  ,  
 1

Rule 3
When two people with speeds of x km/h and y km/h respectively start at the same time and from the
same point but in opposite direction around a circular track of circumference ‘c’ km, then

The time taken to meet for the first time anywhere on the track = 
 1
 
The time taken to meet for the first time at the starting point = LCM of  ,  
 1

Example 1: Convert 90 km/h into m/s.



Solution: 90 km/h =   m/s= 25 m/s


Example 2: Convert 10 m/s into km/h.



Solution: 10 m/s =   m/s = 36km/h


Example 3: A man can cover a certain distance in 1 h 30 min by covering one-third of the distance at
6 km/h and the rest at 15 km/h. Find the total distance.
 
       
Solution: Let the total distance be x km. Then,           
         
   
   1

Example 4: An aero plane started one hour later than the scheduled departure from a place 1200 km
away from its destination. To reach the destinations on time, the pilot had to increase its speed by
200 km/h. What was the normal speed of the aero plane?
Solution: Let the time taken by the aero plane in second case be x hour. Then,
 
    
   
              
               "- *" 67(
Time taken in second case = 2 h

So, Speed = =600 km/h

Hence, normal speed = 600 - 200 = 400 km/h

Example 5: Speed of three cars are in the ratio 2: 3: 4. What is the ratio of time taken by them in
covering the same distance
Solution: Let the speeds of three cars be 2x, 3x and 4x km/h, covered distance be d, then ratio of
time taken by them
+ + +   
 : :  : :  : : 
     

www.sakshieducation.com
www.sakshieducation.com Quantitative Aptitude

Example 6: Two men A and B start together from the same point to walk around a circular path 8 km
long. A walks 2 km and B walks 4 km an hour. When will they next meet at the starting point, if they
walk in the same direction?
Solution: Time to complete one revolution by A and B is    and    or 4h and 2h

The required time is the LCM of 4 and 2 which is 4 h.
Thus, they will meet next time at the starting point after 4 h.

EXERCISE
1. Which of the following speed is the reaches his school 6 minutes late. Next
fastest? day he increases his speed by 1 kmph
(a) 40 m/s (b) 144 km/h and reaches the school 6 minutes early.
(c) 2400 m/min (d) All are equal How far is the school from his house?
2. Mac travels from A to B a distance of 250 (a) 1 km (b) 15 km
 
miles in  hours. He returns to A in 4 (c) 14 km (d) 1 km

hours 30 minutes. His average speed is: 8. If a train runs at 40 kmph, it reaches its
(a) 42 mph (b) 49 mph destination late by 11 minutes but if it
(c) 48 mph (d) 50 mph runs at 50 kmph, it is late by 5 minutes
3. A, B and C are on a trip by a car. A only. The correct time for the train to
drives during the first hour at an average complete its journey is:
speed of 50 km/hr. B drives during the (a) 13 min. (b) 17 min.
next 2 hours at an average speed of 48 (c) 19 min. (d) 22 min
km/hr. C drives for the next 3 hours at an 9. Walking 5/7 of his usual rate, a boy
average speed of 52 km/hr. They reached reaches his school 6 min late. Find his
their destination after exactly 6 hours. usual time to reach school.
Their mean speed was: (a) 10 min (b) 12 min
 (c) 15 min (d) 18 min
(a) 50 km/hr (b) 50 km/hr

10. If I walk at 4 km/h, I miss the bus by 10
(c) 51 km/hr (d) 52 km/hr
min. If I walk at 5 km/h, I reach 5 min
4. A car travels the first one-third of a
before the arrival of the bus. How far I
certain distance with a speed of 10
walk to reach the bus stand?
km/hr, the next one-third distance with a
(a) 5 km (b) 5.5 km
speed of 20 km/hr, and the last one-third
(c) 6 km (d) 7.5 km
distance with a speed, of 60 km/hr. The
11. A man travels on a scooter from A to B at
average speed of the car for the whole
a speed of 30 km/h and returns back from
journey is:
B to A at 20 km/h. The total journey was
(a) 18 km/hr (b) 34 km/hr
performed by him in 10 h. Find the
(c) 35 km/hr (d) 39 km/hr
distance from A to B.
5. Mary jogs 9 km at a speed of 6 km per
(a) 100 km (b) 110 km
hour. At what speed would she need to
(c) 120 km (d) 125 km
jog during the next 1.5 hours to have an
12. A man walks 7.5 km at a speed of 3
average of 9 km per hour for the entire
km/h. At what speed would the man need
jogging session?
to walk during the next 2 h to have an
(a) 9 kmph (b) 13 kmph
average of 4 km/h for the entire session.
(c) 12 kmph (d) 15 kmph
(a) 3.65 km/h (b) 4.75 km/h
6. A car travelling with 5/7 of its actual
(c) 5.25 km/h (d) 6.50 km/h
speed covers 42 km in 1 hr 40 min 48
13. An express train travelled at an average
sec. find the actual speed of the car.
speed of 75 km/h stopping for 5 min
(a) 17 km/hr (b) 32 km/hr
every 125 km. How long did it take to
(c) 31 km/hr (d) 35 km/hr
reach its destination 375 km from the
7. Starting from his house one day, a
 starting point?
student walks at a speed of 2 kmph and (a) 6 h 30 min (b) 4 h 45 min


www.sakshieducation.com
www.sakshieducation.com Quantitative Aptitude

(c) 3 h 15 min (d) 5 h 10 min 19. A walks at 2 km/h and 5 h after his start,
14. A man performs 2/25 of his total journey B cycles after him at 4 km/h. How far
by bus, 21/50 by car and the remaining 2 from the start does B catch up with A?
km on foot. Find the total journey. (a) 20 km (b) 18 km
(a) 4 km (b) 2.7 km (c) 16 km (d) 14 km
(c) 3.4 km (d) 3.8 km 20. Three persons A, B and C run around a
15. A long distance runner runs 9 laps of a circular track of length 1 km, with
400 m track every day. His timings (in respective speeds of 10, 20 and 25 km/h.
min) for four consecutive days are 88, If they start at the same point and at the
96, 89 and 87 respectively. On an same time in the same direction, when
average, how many m/min does the will they will meet again at the starting
runner cover? point.
(a) 39 m/min (b) 40 m/min (a) after 12 min (b) after 14 min
(c) 41 m/min (d) 43 m/min (c) after 16 min (d) after 18 min
16. I started on my bicycle at 7 a.m. to reach 21. A can give B a 40 m start and C 70 m
a certain place. After going a certain start in a km race. How many metres start
distance, my bicycle went out of order. can B give C in a km race?
Consequently, I rested for 35 minutes 
(a) 31 m start (b) 31 m start

and came back to my house walking all  
the way. I reached my house at 1 p.m. If (c) 31 m start (d) 31 m start
my cycling speed is 10 kmph and my 22. Two trains 121 m long and 99 m long are
walking speed is 1 kmph, then on my running in opposite directions, first at 40
bicycle I covered a distance of: km/h and the second at 32 km/h. In what
 time will they completely clear of each
(a) 4 km (b) 13 km
  other from the moment they meet?
 (a) 10 s (b) 11 s
(c) 5 km (d) 15 km (c) 13 s (d) 14 s

17. The ratio between the speed of Meena 23. A train running at 8/11 of its own speed
and Teena is 2 : 3. Meena takes 20 min 
reached a place in 5 h. How much time

more than Teena to walk from A to B. If
could be saved, if the train would have
Meena had walked at double the speed,
run at its own speed.
find the time she would take to walk 
from A to B. (a) 2 h (b) 2 h


(a) 30 min (b) 60 min (c)  h (d) 1 h

(c) 45 min (d) 110 min
18. Two men starting from the same place
walk at the rate of 4 km/h and 4.6 km/h
respectively. What time will they take to
be 3 km apart, if they walk in the same
direction?
(a) 8 h (b) 4 h
(c) 5 h (d) 6 h

Answer Key
1 d 7 d 13 d 19 a
2 d 8 c 14 a 20 a
3 b 9 c 15 b 21 c
4 a 10 a 16 a 22 b
5 c 11 c 17 a 23 c
6 d 12 c 18 c

www.sakshieducation.com
www.sakshieducation.com Quantitative Aptitude

SOLUTIONS
1. 40 m/s =  

km /h =144 km/h 
       
  
2400 m/min = = 144 km/h 
     km.
So, all speeds are equals. 8. Let the correct time to complete the

2. Speed from A to B =    mph journey be x min.

 Distance covered in (x + 11) min. at 40
  mph.
 kmph = Distance covered in (x + 5) min.

Speed from B to A =   mph at 50 kmph
      
= mph   
 
         
Average speed = 3 

 4mph 

 Or,         
 mph =50mph
  Or, 10x=190
3. Total distance traveled  
Or,     
= (50 × 1 + 48 × 2 + 52 × 3) km 

= (50 + 96 + 156) km = 302 km. 9. Since, the boy now walks at of usual
Total time taken = 6 hrs.
 speed, he will take of his usual time

Mean speed    km/hr
  Extra time =    of usual time
 
= 50 km/hr.
 = 6 min (known)
4. Let the whole distance traveled be x km 
  usual time = 6
and the average speed of the car for the 
whole journey be y km/hr.  Usual time = 15 min
/ / /  10. Suppose the required distance be d km/h
Then      
   1     
    Then,

     
 ! 8 +   +  1
 1  
 8    8  ! 11. Let the distance be d km
!   
5. Let speed of jogging be x km/hr.   -  -   (given)
 

Total time taken =  hrs+ 1.5hrs + = 120 km
  
= 3 hrs. 12. Let speed of walking be x km/h.
Total distance covered = (9 + 1.5x) km. .
 .  Total time taken =    h = 4.5 h

   .    Total distance covered = (7.5 + 2x) km

  .   
   !    0!  2  1*    .     !
  . 
6. Time taken = 1hr 40 min 48 sec = 1 hr 40 Speed of walking = 5.25 km/h
   
min   hrs = hrs 13. Time taken to cover 375 km =  h
   
Let the actual speed be x km/hr, = 5h
   
Then,   =42 Number of stoppages =   = 2
 
   Total time to stoppages = (5 × 2 ) min
Or     = 35 km/hr.
 = 10 min
7. Let the distance be x km. Hence, total time taken = 5 h 10 min
Difference in timings = 12 min 14. Let the total journey be x km
 
= hr = hr.  
   
    
             1
  

www.sakshieducation.com
www.sakshieducation.com Quantitative Aptitude

  ie, 720 s (because the LCM of fractions


15. Average speed (m/min) = 
 
  = LCM of numerators/HFC of
40 m/min denominators)
  
 Hence, they will meet for the first time at
16. Time taken = 5 hrs 25 min = hrs.
 the starting point 12 min from the time
Let the required distance be x km. they start.
  
Then   21. A runs 1000 m while B runs
  
   (1000 -40) or 960 m
     1 runs 1000 m while C runs (1000 - 70)

17. Ratio of speed of Meena and Teena is 2 : or 930 m
3. B runs 960 m while C runs 930 m

Ratio of time taken = 3 : 2 B can give ?   ! 
If Teena takes x minute to walk from A 
to B, or 31 m start
then Meena takes x + 20 min 22. Total distance to be travelled
  = 121 + 99 = 220 m

  Relative speed = Sum of speeds
      
= 72 km/h =   = 20 m/s

     
Hence, Meena takes 60 min walking Time required = = 11 s

at her usual speed. 23. New speed = of usual speed

Hence, at double the speed, she 
would take 30 min.  New time = of usual time
 
18. Let the required time = x hour So, of usual time = h

Relative speed= (4.6 - 4) km/h 
= 0.6 km/h  Usual time =    h

  Hence, time saved =      h
 
.      
 .
19. Distance covered by A when B starts
= (2 × 5) km = 10 km
Relative speed = (4 - 2) km/h = 2 km/h

Then, the time of travel for B = = 5h

Distance travelled by B = (5× 4) km
=20 km
20. Length of the track = 1000 m
 
Speed of A =   = m/s
 
 
Speed of B =    m /s
 
 
Speed of C =   m/s
 
They will meet for the first time at the
starting point at a time which is the
5 5 5
LCM of 9 , , : where x, y, z are the
 1 2
speeds of the three persons.
  
ie, the LCM of ;  ,  ,  <
  
i.e. the LCM of
  
= , , >
  

www.sakshieducation.com
www.sakshieducation.com Quantitative Aptitude

17. PROBLEMS ON TRAINS

General Rules for Solving Train Problems:


Rule 1 Train Vs Stationary Object of no Length
Time taken by a train of length T meter to pass a stationary object such as a pole, standing man or a
building is equal to the time taken by the train to cover L meter.
6*/7%8 $+ %8* %.&(/
Speed of the train 
#()* %&9*/ %$ :.$;; %8* ;%&%($/&.0 $<,*:%
Rule 2 Train Vs Stationary Object of Certain Length
Time taken by a train of length T meter to pass a stationary object of length ‘a’ meter such as another
standing train, bridge or railway platform is equal to the time taken by the train to cover (L + a)
meter.
6*/7%8 $+ %8* %.&(/ 6*/7%8 $+ %8* ;%&%($/&.0 $<,*:%
Speed of the train
#()* %&9*/ %$ :.$;; %8* ;%&%($/&.0 $<,*:%
Rule 3 Train Vs Moving Object of no Length
Time taken by the train of length T meter to pass a man moving is equal to the time taken by the train
to cover L meter
(i) When the train and man move in the same direction with speeds of x m/s and y m/s. Then,
@(A- "B -( -.
  8 
'C( -.1( -" /" (./ "-(
(ii) When the train and man move in opposite directions with speeds of x m/s and y m/s then,
@(A- "B -( -.
  8 
'C( -.1( -" /" (./ "-(
Rule 4 Train Vs Moving Object of Certain Length
Time taken by the train of length T meter to pass a moving object of length ‘a’ meter such as another
moving train is equal to the time taken by the train to cover (L + a) meter.
(i) When the two trains move in the same direction with speeds of x m/s and y m/s, (x > y), then
)D "B -( 7(A- "B -( -E" "6F(/-
  8 
'C( -.1( -" /" (./ "-(

(ii) When the two trains move in opposite directions with speeds of x m/s and y m/s. Then,
)D "B -( 7(A- "B -( -E" "6F(/-
  8 
'C( -.1( -" /" (./ "-(
Rule 5 Two Moving Trains
If two trains start at the same time from points A and B towards each other and after crossing they
take a and b seconds in reaching B and A respectively.
Then, (A’s speed): (B’s speed) = √6: √.
Example 1: A train of length 100 m crosses a man who is coming to the train from opposite
direction in 6 s. What is the speed of train?
Solution: Let speed of train = x km/h
Then, speed of train relative to man = (x + 5) km/h

     m/s

 !
  
     
   
!
   55 km/h

EXERCISE - 1

www.sakshieducation.com
www.sakshieducation.com Quantitative Aptitude

1. A train 600 m long passes a pole in 9 direction. The speed of the second train
seconds. What is the speed of the train in is?
km/hr? a) 72 km/hr b) 56 km/hr c) 85
(a) 120 km/hr (b) 180 km/hr km/hr
(c) 240 km/hr (d) 60 km/hr d) 65 km/hr e) None of these
(e) None of these 10. Excluding stoppages speed of the train is
2. A train 100 m long crosses a standing 40 km/hr and including stoppages speed
man in 10 sec. What is the speed of the of the train is 29 km/hr. for how many
train in km/hr? minutes does the train stops per hour?
a) 36 km/hr b) 24 km/hr a) 16.5 min b) 17 min
c) 12 km/hr d) 48 km/hr c) 16 min d) 15.5 min
e) None of these e) none of these
3. Find the speed of train in km/hr whose 11. Two trains one from A to B another from
length is 200 m and crosses a platform of B to A, started simultaneously. After
length 240m in 22 seconds? they meet the trains reach their
a) 47 km/hr b) 39 km/hr destinations after 9 hours and 16 hours
c) 87 km/hr d) 72 km/hr respectively. The ratio of the speed is?
e) none of these a) 4 :3 b) 2:4
4. A train 150 m long passes an electric c) 3:4 d) 2: 5
pole in 5 sec, How long will it take to e) none of these
cross a bridge of 180 m length? 12. Two trains A and B start from Delhi and
a) 11 sec b) 15 sec c) 24 sec Patna and simultaneously B starts from
d) 29 sec e) none of these Patna to Delhi. After passing each other
5. A train 800 m long running at the speed they take 4 hours 48 minutes and 3 hours
of 78 km/hr will cross a tunnel in 1 20 minutes respectively. What is B’s
minute. The length of the tunnel is? speed if A’s speed is 45 km/hr?
a) 700 m b) 500 m c) 1300 m a) 28 km/hr b) 54 km/hr
d) 800 m e) none of these c) 78 km/hr d) 85 km/hr
6. A train 200 m long running with the e) none of these
speed of 60 km/hr passes a man who is 13. Two trains start at the same time from
running at 12 km/hr in the same direction two stations 240 km apart and going in
of the train. How much time will it take opposite direction and cross each other
to cross him? 150 km apart from first stations. What is
a) 14 sec b) 15 sec c) 19 sec the ratio of their speeds?
d) 23 sec e) Non e of these a) 5: 3 b) 2: 4
7. Two trains one 260m and other 140m c) 3: 4 d) 2: 5
long are running in opposite direction on e) none of these
parallel lines. Speed of 1st is 77 km/hr 14. Two trains of length 200m and 175m run
and other is 67 km/hr, how long will it on parallel tracks. When running in the
take to cross each other? same direction faster train cross slower
(a) 10 sec (b) 7sec (c) 2 mins 
one in 37 sec. When running in

(d) 70 sec (e) None of these
opposite direction at their earlier speeds
8. Two trains are moving in the same 
direction at the rate of 80 km/hr and 50 they pass each other completely in 7

km/hr. The fast train crosses a man in the sec. Find the speed of each train?
slower train in 27 sec. Find the length of (a) 30 m/sec, 20 m/sec
the faster train? (b) 20 m/sec, 15 m/sec
(a) 225 m (b) 275 m (c) 325 m (c) 45 m/sec, 27 m/sec
(d) 256 m (e) None of these (d) 40 m/sec, 16 m/sec
9. A train 200 m long passes a stone in 15 (e) None of these
seconds and passes another train of same 15. Two trains, each of length 90m run on
length by 12 sec coming from opposite parallel tracks. When running in the same

www.sakshieducation.com
www.sakshieducation.com Quantitative Aptitude

direction faster train crosses slower one 18. A jogger running at 9 kmph alongside a
in 18 sec completely. When running in railway track is 240 metres ahead of the
opposite direction, at their earlier speeds engine of a 120 metre long train running
they pass each other completely in 9 sec. at 45 kmph in the same direction. In how
Find the speed of each train? much time will the train pass the jogger?
(a) 48 m/sec, 38 m/sec (a) 38 sec (b) 20 sec
(b) 15 m/sec, 5 m/sec (c) 36 sec (d) 72 sec
(c) 45 m/sec, 27 m/sec 19. Two trains 200 m and 150 m long are
(d) 30 m/sec, 20 m/sec running on parallel rails at the rate of 40
(e) none of these kmph and 45 kmph respectively. In how
16. A train moves past a telegraph' post and a much time will they cross each other, if
bridge 264 m long in 8 seconds and 20 they are running in the same direction?
seconds respectively. What is the speed (a) 80 sec (b) 136 sec
of the train? (c) 192 sec (d) 252 sec
(a) 75 km/hr (b) 82 km/hr 20. Two trains are moving in opposite
(c) 79 km/hr (d) 79.2 km/hr directions @ 60 km/hr and 90 km/hr.
17. How many seconds will a 500 metre long Their lengths are 1.10 km and 0.9 km
train take to cross a man walking with a respectively. The time taken by the
speed of 3 km/hr in the direction of the slower train to cross the faster train in
moving train if the speed of the train is seconds is:
63 km/hr? (a) 39 (b) 47
(a) 32 (b) 30 (c) 48 (d) 49
(c) 40 (d) 48

Answer Key
1 c 6 b 11 a 16 d
2 a 7 a 12 b 17 b
3 d 8 a 13 a 18 c
4 a 9 a 14 a 19 d
5 b 10 a 15 b 20 c

SOLUTIONS
  = 11 sec
1. Speed =  m/sec
 
  5. Distance = Speed × Time = 78 × × 60
=  km/hr = 240 km/hr 

 = 1300 m
[∵ 1 m/sec = km/hr]
 ∴ Length of the tunnel = 1300 – 800 =
 
2. Speed =  500 m
 
= 10 m/sec 6. Speed of the train relative to man = (60 –
 12) km/hr
= 10 × = 36 km/hr  

     = 48 × m/sec = m/sec
 
3. Speed = = =
   Time taken by the train to cross the man
= 20 m/sec = Time taken by it to cover 200 m at

= 20 × = 72 km/hr  
   m/sec = 200 × sec = 15 sec
 
 
4. Speed = =   m/sec 7. Relative speed of the trains
 
    = (77 + 67) km/hr = 144 km/hr
∴ Required Time = 
 

www.sakshieducation.com
www.sakshieducation.com Quantitative Aptitude

 13. In the same time, they cover 150 km and


= 144 × m/sec = 40 m/sec

90 km respectively.
Time taken by the trains to cross each
∴ Ratio of their speeds = 150: 90
other = Time taken to cover (260 + 140)
 = 5: 3
m at 40 m/sec =   sec 14. length: L1= 200m;

= 10 sec L2 = 175m
8. Relative speed of the trains = (80 − 50)  
T1 =  (/; T2 =  (/
 
km/hr = 30 km/hr 5 5  
  )D    ;
= 30× m/sec =   m/sec   
 

@  @ '  '
∴ Length of the train = × 27 = 225 m )5  0 20 2
  '  '

9. Speed of the first train =   m/sec = Speed of each train:

   
 T1 =   (/; T2 =   (/
  m/sec    
  
  
Let the speed of the second train be x )D   3  
  4;
 
m/sec  
  
Relative speed = (   m/sec    
 )5  0 2J  K


= 12 ⟺ 400 = 12 (

  ⟺   


   
400 = 160 + 12 x SF = 30 m/s SL = 20 m/s
    
     
  15. )D    
  
  / (/ 
! 
)5  0 2 20
So, speed of second train =   
! 
 SF = 15 m/s SL = 5 m/s

 km/hr = 72 km/hr
16. Let the length of the train be x metres and
10. Due to stoppages, it covers (40 – 29) = its speed be y m/sec.
11 km less Then,
   
Time taken to cover 11 km =  = = 8  x = 8y Now, =y
 1 
16.5 min  8y + 264 = 20y  8  .
11. (A’s speed) : (B’s speed) = √ H √ = ∴ Speed = 22 m/sec
4:3 
    km/hr = 79.2 km/hr.
12. '  "D !D-( 
 17. Speed of train relative to man
  ;
  = (63 - 3) km/hr = 60 km/hr
'  "D  D-(  
=   m/sec = m/sec.
   
  Time taken to pass the man


= > ?@ 
B
     sec = 30 sec.
 
 0 2 => 
= > A@  √ 18. Speed of train relative to jogger

 √ √ = (45 - 9) km/hr = 36 km/hr
=>    
A √ √ =   m/sec = 10 m/sec.
√ √ 
I     Distance to be covered
√ √
 1/ = (240+ 120) m = 360 m.
B’s speed = 54 km/hr
www.sakshieducation.com
www.sakshieducation.com Quantitative Aptitude


∴ Time taken    sec = 36 sec.

19. Relative speed = (45 - 40) kmph
= 5 kmph
 
   m/sec =   m/sec
 

Time taken =    sec

=252 sec
20. Relative speed = (60 + 90) km/hr
 
=    m/sec = m/sec.
 
Distance covered = (1.10 + 0.9) km
= 2km = 2000m.

Required time =    sec

= 48 sec

www.sakshieducation.com
www.sakshieducation.com Quantitative Aptitude

18.BOATS AND STREAMS


General Rules for Solving Boats and Streams Problems
Downstream Motion
When an object is moving in their direction in which the water in the stream is flowing, then the
object is said to be downstream.
Upstream Motion
When an object is moving against (opposite) direction in which the water in the stream is flowing,
then the object is said to be moving upstream.
Motion in Still Water
When an object is moving in water where there is no motion in the water, then the object is said to be
moving in still water.
Rule 1 Downstream and Upstream Speed
Let the speed of the boat in still water be x km/h and speed of the stream be y km/h, then
Speed of the boat with stream= downstream speed = (x + y) km/h
Speed of the boat against stream = upstream speed = ( x - y ) km/h
As, when the boat is moving downstream, the speed of the water aids the speed of the boat and when
the boat is moving upstream, the speed of the water reduces the speed of the boat.
Rule 2 Speed of Boat in Still Water & Speed of Stream
If the downstream speed of boat is a km/h and the upstream speed of boat is b km/h, then

Speed of boat in still water = (a+ b)km/h


Speed of stream = (a - b)km/h

Example 1: The current of a stream runs at 1 km/h. A motor boat goes 35 km upstream and back
again at the starting point in 12 h. What is the speed of motor boat in still water?
Solution: Let the speed of the boat in still water be x km/h. Then,
 
                    
 
           ( x = -1/6 is not possible)
So, speed of boat in still water = 6 km/h
Example 2: A man can row 6 km/h in still water. When the river is running at 4 km/h, it takes him 2
h 15 min to row to a place and back. How far is the place?
Solution: Speed downstream = (6 + 4) km/h = 10 km/h
Speed upstream = (6 - 4) km/h = 2 km/h
Let the required distance be x km.
  
Then,    = 3.75km
  

EXERCISE

1. The stream runs at 1 km/hr, a motor boat 3. Speed of a boat in standing water is 9
goes 35km upstream and back again to kmph and the speed of the stream is 1.5
the starting point after 12hours. What’s kmph. A man rows to a place at a
the speed of the motor boat in still water? distance of 105 km and comes back to
(a) 6km/hr (b) 7km/hr the starting point. The total time taken by
(c) 8km/hr (d) 10km/hr him is:
2. If the man’s rate of rowing in still water (a) 22 hours (b) 27 hours
is 5km/hr and the speed of stream is (c) 20 hours (d) 24 hours
10km/hr. The distance given is 15km/hr
.Find the time taken to row downstream? 4. A man can row 9km/hr in still water, if
(a) 1hr (b) 2hr the river running at 4 km/hr. It takes 6
(c) 3hr (d) 4hre hours more to upstream than to go

www.sakshieducation.com
www.sakshieducation.com Quantitative Aptitude

downstream for the same distance. How minutes. The speed of the stream (in
far is the place? km/hr) is:
(a) 48.75 km (b) 47.85km (a) 9 (b) 5
(c) 75.48km (d) 45.78km (c) 6 (d) 11
5. The ratio of speed of boat in still water to 
9. A man can row 9 kmph in still water

the speed of current is 6:1. If the
and finds that it takes him thrice as much
upstream speed is 2 km/hr. find the speed
time to row up than as to row down the
of the boat in still water?
same distance in the river. The speed of
(a) 2.4km/hr (b) 3.6km/hr
the current is:
(c) 4.4km/hr (d) 5.2km/hr  
6. The speed of a boat in still water is 15 (a) 2 km/hr (b) 4 km/hr

km/hr and the rate of current is 3 km/hr.  
(c) 4 km/hr (d) 4 km/hr
 
The distance travelled down steam in 12
10. In a stream running at 2 km/h, a motor
minutes is:
boat goes 5 km upstream and back again
(a) 3.3 km (b) 2.9 km
to the starting point in 1 h 20 min. Find
(c) 2.4 km (d) 3.6 km
 the speed of the motor boat in still water.
7. A person can row  km/h in still water. (a) 4 km/h (b) 8 km/h

It takes him twice as long to row up a (c) 10 km/h (d) 6 km/h
distance as to row down the same
distance. Find the speed of the stream.
(a) 2 km/h (b) 2.2 km/h
(c) 2.5 km/h (d) 2.7 km/h
8. A motorboat, whose speed is 15 km/hr in
still water goes 30 km downstream and
comes back in a total of 4 hours 30

ANSWER KEY
1 a 3 d 5 a 7 c 9 c
2 a 4 a 6 d 8 b 10 b

SOLUTIONS
1. Let the speed of the motor boat be  Hence, 13*   L   
kmph 
=>   
 
+ =12 =>  
   distance=13* =48.75 km

2. Tdown=
 
=1 Hr 5. Let the speed of boat in still water be
3. Speed upstream= 7.5 kmph. 6 1* and speed of stream be 
Speed downstream = 10.5 kmph. kmph
Total time taken Upstream speed=6   ==2
=



 hour = 24 hours. =>  =0.4
. .
=>6   L .   . 1*
4. Downstream speed=9+4=13 kmph
6. Speed downstream = (15 + 3) kmph
Upstream speed=9-4=5 kmph
= 18 kmph.
Let the time taken to row downstream be 
  Distance traveled  !   1

Then time taken to row upstream will be = 3.6 km.
   hrs 7. Speed upstream + speed downstream

Distance is same in both downstream and =    = 15 km/h

upstream.
www.sakshieducation.com
www.sakshieducation.com Quantitative Aptitude

Since, the times taken are in the ratio 


So, Speed upstream = km/hr;

2:1,the speeds will be in the ratio 1:2.

Speed downstream = 14 km/hr.
Speed up stream =   = 5 km/h Hence, speed of the current

  
Speed downstream =   =   km/hr
  
= 10 km/h 
=  km/hr.
 
Speed of the stream =    10. Let the speed of the motor boat in still

= 2.5 km/h water be x km/h. Then,
8. Let the speed of the stream be x km/hr. Downstream speed = (x + 2) km/h,
Then, Upstream speed = (x - 2) km/h
Speed downstream = (15+x) km/hr, Speed   
 
upstream =(15 - x ) km/hr.   
     
    
             
 
           
       
           
         ! 
    km/hr.         ! 
9. Let speed up stream be x kmph. Then, !
speed downstream = 3x kmph. [as  cannot be nE.g.ative]

Speed in still water = (3.x + x) kmph

= 2x kmph.
! 
  
 

www.sakshieducation.com
www.sakshieducation.com Quantitative Aptitude

19.SIMPLE INTEREST
Interest
It is the sum which is paid by the borrower to the lender for using the money for a specific time
period.
The money borrowed is called the Principal.
The rate at which the interest is calculated on the principal is called Rate of Interest.
The time for which the money is borrowed is called Time.
The total sum of principal and interest is called Amount.
Simple Interest:
If P = Principal, R = Rate per cent per annum T= Number of years, SI = Simple Interest and
A= Amount.
Then,
EF
(i) SI =

=G
(ii) P=
F
=G
(iii) R=
E
=G
(iv) '
EF
EF F
(v) M  N  )C  N   N   
 
Here, the interest is calculated on the original principal i.e., the principal to calculate the interest
remains constant throughout the time period. The interest earned on the principal is not taken into
account for the purpose of calculating interest for later years.
Example 1: Find the SI on Rs. 7200 at 8% per annum for 10 months.
 
Solution: Here, P = Rs. 7200, R = 8% per annum and T = yr = 8
 
EF  
SI =   = Rs.   !  = Rs. 480
  
Example 2: A sum is lent at 10% per annum Simple interest . In how many years it will get
doubled?
Solution: Sum will be doubled when SI = P
Therefore,
EF
)C  N 

 
O'   '    8(.
O 
Example 3: Three persons separately borrow Rs. 51000 in all from a banker at 10% and returned
with interest after 2, 5 and 6 year respectively. If the returned amounts are equal, what are the sums
borrowed by each of them?
Solution: If N , N , N be the sums borrowed and M , M , M be the amounts. Then,
M  M  M
N     N     N   
N   N   N 
  
N N !N
   P
  
P P P
 N  , N  , N 
 !

www.sakshieducation.com
www.sakshieducation.com Quantitative Aptitude

H H H
But N  N  N       
 
 P   P  P   
   P
 
 P  
H 
Hence, N      Rs. 20000
 
H 
N     = Rs. 16000
 
H 
N     = Rs. 15000
EXERCISE
1. Find SI if P = Rs. 1000, R = 20% per (c) Rs. 3.60 (d) Rs. 4.80
annum 4 yr 8. In how much time would the simple
(a) Rs. 400 (b) Rs. 600 interest on a certain sum be 0.125 times
(c) Rs. 800 (d) Rs. 850 the principal at 10% per annum?
 
2. A sum of money at simple interest (a)  % years (b)  % years
amounts to Rs. 815 in 3 years and to Rs.  
(c)  % years (d)  % years
854 in 4 years. The sum is:
(a) Rs. 850 (b) Rs. 790 9. At what rate percent per annum will the
(c) Rs. 698 (d) Rs. 800 simple interest on a sum of money be
3. If Rs. 64 accounts to Rs. 83.20 in 2 2/5th of the amount in 10 years?
years, what will Rs. 86 amount to in 4 (a) 4% (b) 55%
years at the same rate of S.I? (c) 8% (d) 65 %
(a) Rs. 115.80 (b) Rs. 127.70 10. The simple interest on a certain sum of
(c) Rs. 127.40 (d) Rs. 137.60 money at the rate of 5% p.a. for 8 years
4. If a sum of money at simple interest is Rs. 840. At what rate of interest the
doubles in 6 years, it will become 4 times same amount of interest can be received
in: on the same sum after 5 years?
(a) 17 years (b) 15 years (a) 10% (b) 8%
(c) 16 years (d) 18 years (c) 9% (d) 12%
5. The rate at which a sum becomes four 11. The interest on a certain deposit at 4.5%
times of itself in 15 years at S.I will be: p.a. is Rs. 202.50 in one year. How
(a) 18% (b) 17.2% much will the additional interest in one
(c) 20% (d) 27% year be on the same deposit at 5% p.a.?
6. A sum of money triples itself in 15 years (a) Rs. 30.25 (b) Rs. 22.50
6 months. In how many years would it (c) Rs. 25 (d ) Rs. 52.75
double itself? 12. What will be the ratio of simple
(a) 5 years 3 months interest earned by certain amount at the
(b) 7 years 9 months same rate of interest for 6 years and
(c) 10 years 3 months that for 9 years?
(d) 11 years 6 months (a) 5 :3 (b) 4 : 7
(c) 2 : 3 (d) data inadequate
7. The simple interest on Rs.10 for 4 13. Nitin borrowed some money at the rate
months at the rate of 3 paise per rupee of 6% p.a. for the first three years, 9%
per month is: p.a. for the next five years and 13% p.a.
(a) Rs. 1.20 (b) Rs.1.90 for the period beyond eight years. If the

www.sakshieducation.com
www.sakshieducation.com Quantitative Aptitude

total interest paid for 11 years is 8160 returns the loan with interest at the end of
how much money did he borrow? 4 yr, .how much does the man save?
(a) Rs. 8000 (b) Rs. 13,000 (a) Rs. 625 (b) Rs. 450
(c) 11,000 (d) data inadequate (c) Rs. 575 (d) Rs. 700
14. Consider the following statements: 20. Find the simple interest on Rs. 4500
If a sum of money is lent at simple from Oct. 10, 2009 to Dec. 22, 2009 at
interest, then the 
9 % per annum

(1) Money gets doubled in 5 years if the

(a) Rs. 96 (b) Rs. 84
rate of interest is 16 %. (c) Rs. 86 (d) Rs. 94

(2) Money gets doubled in 5 years if the 21. A person borrowed Rs. 2000 at 5% per
rate of interest is 20%. annum simple interest and immediately
(3) Money becomes four times in 11 lent it at 6% per annum simple interest.
years if it gets doubled in 5 years. 
At the end of  yr he collected the

Of these statements,
amount and settled his loan. What was
(a) 1 and 3 are correct
his profit?
(b) 2 alone is correct
(a) Rs. 45 (b) Rs. 40
(c) 3 alone is correct
(c) Rs. 50 (d) Rs. 56
(d) 2 and 3 are correct
22. What annual payment will discharge a
15. The SI on a sum of money is 25% of the
debt of Rs. 9675 due in 4 installments at
principal, and the rate per annum is equal
5% simple interest?
to the number of years. Find the rate per
(a) Rs. 2240 (b) Rs. 2180
cent,
(c) Rs. 2250 (d) Rs. 2160
(a) 4.5% (b) 6%
23. A person invests money in three different
(c) 5% (d) 8%
schemes for 5 yr, 10 yr and 15 yr at 8%,
16. A man wanted to invest Rs. 20000 for a
 10% and 12% simple interest
period of  8 in order to get an interest respectively. At the completion of each

of Rs. 20000. At what rate of simple scheme, he gets the same interest. The
interest should he invest? ratio of his investments is
 
(a) 13 % (b) 13 % (a) 45: 9: 5 (b) 45: 18: 10
 
 (c) 25: 18: 10 (d) 25: 9: 5
(c) 13 % (d) 13 %
  24. Two equal sums of money were lent at
17. A sum becomes 10/9 times itself in 1 yr. simple interest at 10% per annum for 4 yr
Find the rate of simple interest. and 5 yr respectively. If the difference in
 
(a)  % (b) 11 % interests for two periods was Rs. 220,

  then each sum is
(c) 12 % (d) 12 %
 (a) Rs. 880 (b) Rs. 1100
18. At simple interest of 5%, 6% and 8% for
(c) Rs. 2200 (d) Rs. 1640
three consecutive year, the interest
25. An automobile financier claims to be
earned is Rs. 760. Find the principal.
lending money at simple interest, but he
(a) Rs. 4600 (b) Rs. 3200
includes the interest every six months for
(c) Rs. 4000 (d) Rs. 3600

calculating the principal. If he is charging
19. A man borrowed Rs. 5000 at % per an interest of 10%, the effective rate of
annum simple interest for 6 yr. Instead of interest becomes:
clearing the loan at the end of 6 yr, if he (a) 13% (b) 10.25%

www.sakshieducation.com
www.sakshieducation.com Quantitative Aptitude

(c) 15% (d) 11%

ANSWER KEY
1 c 7 a 13 a 19 a 25 b
2 c 8 d 14 b 20 b
3 d 9 a 15 c 21 c
4 d 10 b 16 a 22 c
5 c 11 b 17 b 23 b
6 b 12 c 18 c 24 c

SOLUTIONS
EF  
1. S.I = =  Rs. 800 S.I. =x, Rate = %
  
2. S.I. for 1 year = Rs. (854 - 815) = 39.  
Time =  8(.

S.I. for 3 years = Rs. (39 × 3) = Rs. 117. 

Principal =Rs. (815 - 117) = Rs. 698. = 7 years 9 months.



3. P = Rs. 64, 7. S.I. = Rs. (10× ×4) = Rs. 1.20

S.I. = Rs. (83.20 - 64) = Rs. 19.20, 8. Let the sum be x.
T = 2 years. 
Then, S.I. = 0.125 x = x, R = 10%.
 .
So, rate R =   % = 15% 
  Time =   years
 
Now, P = Rs. 86, R = 15%, T = 4 years.  
 = years =  years.
S.I. =   = Rs. 51.60.
 
M  !  . 9. Let the sum be x. Then, S.I. = ,

 . Time = 10 years.

4. Let the sum be x. Then, S.I. =x. Rate =   % = 4%

 
Rate =  %  % 10. S.I. = Rs. 840, R = 5%, T = 8 years.
 
  
Now, sum = x, S.I. = 3x, Rate = %. Principal (P) = Rs.  
 

Time=

= 18 years. = Rs. 2100.

 Now, P = Rs. 2100, S.I. = Rs. 840,

5. Let the sum be x. T = 5 years.
Then, S.I. = 3x.   !
  ). C    O.-(  0 2 %  !%
O.-(  0 20 2%   
N'    11. S.I. = Rs. 202.50, R = 4.5%, T = 1 year.
20 %. Principal = Rs. 
.
 = Rs.
6. Let the sum be x. Then, S.I. = 2x, .
 4500.
Time = 15 years
 Now, P = Rs. 4500, R = 5%,
   T = 1 year.
O.-(  J K%  % 
  S.I. = Rs.   = Rs. 225.
 

Difference in interest
Now, sum = x,
= Rs. (225 - 202.50) = Rs. 22.50.
www.sakshieducation.com
www.sakshieducation.com Quantitative Aptitude

12. Let the principal be P and rate of interest =G


R= 
E
be R%.  
I

J
    %per annum
 
Required ratio = R  S  
I

J = % per annum =  % per annum
NO 18. Let the principal be Rs. x Then,
  : 
NO   
      
13. Let the sum be Rs. x. Then,   
  
࢞ൈ૟ൈ૜ ࢞ൈૢൈ૞ ࢞ ൈ ૚૜ ൈ ૜       
൬ ൰൅൬ ൰൅൬ ൰   
૚૙૙ ૚૙૙ ૚૙૙  
= 8160 0 2

֞ ૚ૡ࢞ ൅ ૝૞࢞ ൅ ૜ૢ࢞ ൌ ሺૡ૚૟૙ ൈ ૚૙૙ሻ
Hence, the principal is Rs. 4000.
    ! 
19. P = Rs. 5000, R = % per annum,
  = Rs. 8000.
14. Let the sum be x. Then, S.I. = x T = 6 year
 Simple interest for 6 year
1. Time =  = 6 years (False)
  
 = Rs.      = Rs.
 
2. Time = = 5 years (True) 1875

3. Suppose sum = x. Then, S.I. = x and Simple interest for 4 year
Time = 5 years.  
= Rs.     = Rs.
 
Rate =  % = 20% 1250

Now, sum = x, S.I. = 3x and Rate = 20%. Savings = Rs. (1875 - 1250) = Rs. 625
 
Time =  years=15 years False) 20. P = Rs. 4500, R = %
 
So, 2 alone is correct. T = (21 + 30 + 22) days
15. Let the principal be P, 
= 73 days =   year
E 
Then interest = and EF  
SI =   = Rs.    
  
Rate = Time = T 
EF = Rs. 84
Now SI 

21. For borrowed money
N N. '
 P = Rs. 2000, R = 5% per annum and
  
 '   T=

year
 '   8(.s SI = Rs.   
 
 = Rs.
 
 O  %  '  O
 250
16. P = Rs. 20000, SI = 20000, T  year For lent money

=G
R=   P = Rs. 2000, R = 6% per annum and
E 
   T = year

0 2 % *( .D
   SI= Rs. 

  

 = Rs.
  
  %per annum
 300
17. Let the sum be Rs. x, Amount = Rs. His profit = Rs. (300 - 250) = Rs. 50

, T= 1 year 22. Let the annual installment be Rs. x .
 Then,
S I = (Amount - sum) = Rs.   

= Rs.  

www.sakshieducation.com
www.sakshieducation.com Quantitative Aptitude

  
 

  

       


 

  Hence, each sum is Rs. 2200.
  25. Let the sum be Rs. 100. Then,
   
     = 9675 S.I. for first 6 months = Rs.  
   
!     = Rs. 5.

   = 2250 S.I. for last 6 months = Rs. 


 
Hence, the annual installment is Rs. =Rs. 5.25.
2250. So, amount at the end of 1 year
23. Let the investments be x, y, and z = Rs. (100 + 5 + 5.25) = Rs. 110.25.
respectively Effective rate = (110.25 -100)
! 8  = 10.25%.

 
T    


 8 T
  8  T   
 ! 
: 8: T  : !: 
24. Let each sum be Rs. x. Then,
   
0 20 2  
 

www.sakshieducation.com
www.sakshieducation.com Quantitative Aptitude

20.COMPOUND INTEREST
In compound interest, the interest is added to the principal at the end of each period and the
amount thus obtained becomes the principal for the next period. The process is repeated till the end
of the specified time.
If P = Principal
R = Rate per cent p.a
Time = n years
A = Amount; CI = Compound Interest
When the interest is compounded annually
F 
Amount after n years = (A) =N   

F  F 
Compound Interest = N     N  NU    V
 
Important Formulae
1. If the rate of interest differs from year to year i.e. R1 in the first year, R2 in the second year,O in
F F F
the third year. Then, A = N            
  
2. When the principal changes every year, we say that the interest is compounded annually. Then,
F 
A =N   

3.When the principal changes every six months, we say that the interest is compounded half yearly
or semi-annually. Then,
 
A =N 3  
4


4.When the principal changes every three months, we say that the interest is compounded quarterly
Then,
 
A =N 3  4


5. When the principal changes after every month, we say that the interest is compounded monthly
Then,

 
A =N 3  
4


6. When the interest is compounded annually but time is in fraction say  year Then,
O
O 
M  N 0  2 J   K
 

7. The difference between the simple interest and compound interest for 2 years (or terms) is given
by the formula
O 
,  N0 2

Where D is the difference, P is the principal and R is the rate of interest.
8. Present worth of x % due n years, hence is given by
K
Present worth =  
I J


www.sakshieducation.com
www.sakshieducation.com Quantitative Aptitude

Example1: Find the compound interest on Rs. 5500 at 9% per annum for 2 years, if the interest is
compounded annually?
Solution: P = Rs. 5500, R = 10% per annum and n = 2 years

 
M   0  2   
  
= Rs. 6534.56
?C  M  N  .   
=Rs. 1034.55
Example 2: Find the compound interest on Rs. 12000 for 3 years, if the rate of interest for first year
is 5%, second year is 6% and third year is 7%.
Solution: P= Rs. 12000, O  = 5%, O = 6% and O = 7%
F F F
Amount  N         
  
 
= Rs. W         X
  
  
= Rs. W    X
  
= Rs. 14290.92
Compound interest = Rs. (14290.92 - 12000) = Rs. 2290.92

Example 3: If simple interest on a sum of money at  % per annum for 3 year is Rs. 1800. Find the

compound interest on the same sum for same period at the same rate.

Solution: Here, Rate = % per annum, Time = 3 years, SI = Rs. 1800

 
Principal = Rs. 3  4=Rs. 8000


 
F   
Amount = N     Rs. Y! 3  
4 Z= Rs. [!   \
  

  
= Rs. W!    X= Rs. 1938.38
  
CI = Rs. [9938.38 - 8000] = Rs. 1938.38
Example 4: The difference between the compound interest and simple interest on a certain sum at
10% per annum for 2 year is Rs. 150. Find the sum.
Solution: Here, D = Rs. 150, T = 2 year, R = 10%
NO N
,     N  
   

Example 5: In what time will Rs. 64000 invested at 5% per annum fetch an interest of Rs. 4921, the
interest being compounded half yearly.
F 
Solution : A= P + CI = N   



       [  \
 
!   
 0 2
 

  
0 2 0 2
 

www.sakshieducation.com
www.sakshieducation.com Quantitative Aptitude


 -    -  8(.

Example 6: The value of a TV that was purchased in January 1999, depreciates at 12% per annum. If
its value in January 2001 is Rs. 4840, then what was the purchase price of TV?
Solution: Let P be the price of TV in January 1999, then the value of TV in January 2001,
i.e. after two years.
O 
 N 0  2

 
 !  N 0  2

 
 !  N 0 2

!    
N
  
 N  Rs. 6250

EXERCISE
1. Find the compound interest on Rs. 9375 (c) 6.08% (d) 6.09%
at 8% per annum for 2 yr. 10. A sum of money placed at compound
(a) Rs.1560 (b) Rs.1512 interest doubles itself in 5 years. It will
(c) Rs.1590 (d) Rs.1548 amount to eight times itself at the same
2. The compound interest on Rs.10240 at rate of interest in:
 (a) 7 years (b) 12 years
% per annum for 2 yr 73 days is
(c) 15 years (d) 30 years
(a) Rs. 1464.50 (b) Rs. 1664.50
8. A sum of Rs. 12,000 deposited at
(c) Rs. 1480 (d) Rs.
compound interest becomes double after
1580.50
5 years. After 20 years, it will become:
3. The difference between compound
(a) Rs. 1,10,000 (b) Rs. 1,30,000
Interest and simple interest on an amount
(c) Rs. 1,24,000 (d) Rs. 1,92,000
of Rs. 15,000 for 2 years is Rs. 96.
9. The least number of complete years in
What is the rate of interest per annum?
which a sum of money put out at 20%
(a) 8 (b) 11
compound interest will be more than
(c) 12 (d) None of these
doubled is:
4. The difference between the simple
(a) 7 (b) 4
interest the compound interest on Rs.
(c) 5 (d) 8
8000 at 10% per annum 3 yr is
10. Sum of money becomes Rs.13,380 after
(a) Rs. 260 (b) Rs. 352
3 years and Rs. 20,070 after 6 years on
(c) Rs.248 (d) Rs. 310
compound interest. The sum is:
5. The present worth of Rs. 2809 due 2 yr
(a) Rs. 9200 (b) Rs. 9000
hence at 6% per annum, is
(c) Rs. 8920 (d) Rs. 9040
(a) Rs. 2100 (b) Rs. 2600
11. A sum of money invested at compound
(c) Rs. 2400 (d) Rs. 2500
interest amounts to Rs. 800 in 3 years
9. The effective annual rate of interest
and to Rs. 840 in 4 years. The rate of
corresponding to a nominal rate of 6%
interest per annum is:
per annum payable half-yearly is:
(a) 4% (b) 8%
(a) 6.10% (b) 6.11 %
www.sakshieducation.com
www.sakshieducation.com Quantitative Aptitude

(c) 5% (d) 6% 18. On a sum of money, the simple interest


14. In a factory, the production of cement for 2 years is Rs. 660, while the
rises to 2420 tons from 2000 tons in two compound interest is Rs. 696.30, the rate
years. Find the rate of growth per annum. of interest being the same in both the
cases. The rate of interest is:
(a) 8% (b) 9% (a) 13% (b) 14%
(c) 10% (d) 11% (c) 12% (d) 11%
13. What is the difference between SI and CI
19. A person lent out a certain sum on simple
for 2 yr on Rs.10000, when the rate of interest and the same sum on compound
interest is 11% for the first year and 12% interest at a certain rate of
for second year? interest per annum. He noticed that the
(a) Rs. 120 (b) Rs. 132 ratio between the difference of
(c) Rs. 144 (d) Rs. 128
compound interest and simple interest of
14. A man borrows Rs. 2550 to be paid back 3 years and that of 2 years is 25: 8. The
with compound interest at the rate of 4% rate of interest per annum, is
per annum by the end of 2 years in two (a) 13% (b) 14%
equal yearly installments. How much 
(c) 12% (d)  %
will each installment be? 
(a) Rs. 1275 (b) Rs.1383 20. A man borrows Rs. 12,500 at 20%
(c) Rs.1352 (d) Rs. 1287 compound interest. At the end of every
15. Mr. Dua invested money in two schemes year he pays Rs. 2000 as part repayment.
A and B offering compound interest @ 8 How much does he still owe after three
p.c.p.a. and 9 p.c.p.a. respectively. If the such installments?
total amount of interest accrued through (a) Rs. 14,000 (b) Rs. 13,684
two schemes together in two years was (c) Rs. 15,600 (d) None of these
Rs. 4818.30 and the total amount 21. A man deposited a total sum of Rs.
invested was Rs. 27,000, what was the 88400 in the name of his two sons aged
amount invested in Scheme A? 19 and 17 yr, so that at the age of 21 ,
(a) Rs. 12,000 (b) Rs. 14,500 both will get equal amounts. If the
(c) Rs. 16,000 (d) cannot be determined money is invested at the rate of 10%
16. The compound interest on a sum of compound interest per annum. What are
money for 2 years is Rs. 832 and the the shares of his two sons?
simple interest on the same sum for the (a) Rs. 48800, Rs. 40000
same period is Rs. 800. The difference (b) Rs. 48400, Rs. 48000
between the compound interest and the (c) Rs. 48400, Rs.40000
simple interest for 3 years will be: (d) Rs. 48000, Rs. 40000
(a) Rs. 50 (b) Rs. 6 7 22. If the compound interest, on a certain
(c) Rs. 98.56 (d) Rs. 75.45 sum for 2 yr at 3 percent be Rs. 101.50,
17. The difference between the simple what would be the simple interest?
Interest on a certain sum at the rate of (a) Rs. 100 (b) Rs. 98
10% per annum for 2 years and (c) Rs. 96.50 (d) Rs. 94
compound interest which is 23. The compound interest on a certain sum
compounded every 6 months is Rs. for 2 yr is Rs. 40.80 and the simple
124.05. What is the principal sum? interest is Rs. 40. Find the sum.
(a) Rs. 9000 (b) Rs.8000 (a) Rs.400 (b) Rs. 450
(c) Rs.10,000 (d) Rs. 13,000 (c) Rs.500 (d) Rs. 550

www.sakshieducation.com
www.sakshieducation.com Quantitative Aptitude

ANSWER KEY
1 a 6 d 11 c 16 c 21 c
2 a 7 c 12 c 17 b 22 a
3 a 8 d 13 b 18 d 23 c
4 c 9 b 14 c 19 d
5 d 10 c 15 a 20 d

SOLUTIONS
F  
1. A = N    4. SI = Rs. W X  Rs. 2400
 
  
= Rs. [      \ CI = Rs. [!     ! \
 
  = Rs. [10648 - 8000] = Rs. 2648
= Rs. W    X = Rs. 10935
  Difference = Rs. [2648 - 2400] = Rs. 248
CI = 10935-9375 = Rs. 1560
  
2. P = Rs. 10240, R = %, T = 2 years 73 5. N`  Rs. a   b
I J

days  
   Rs. !   
 
A  U        
 = Rs. 2500
 
 V 6. Amount of Rs.100 for 1 year when
 
   compounded half-yearly
= Rs. W       X 
   
   = Rs. [     \ Rs. 106.09
= Rs.       
  
= Rs. 11704.50 Effective rate = (106.09-100) %
CI = Rs. (11704.50-10240) = 106.09.
F  F 
= Rs. 1464.50 7. N     N     
 
F  
3. [       \ Let  
F
  !N
 
 ] O 
0 2  0  2  !  
 
O  O 
  R0  2  S O  O 
   ;0  2 <  0  2
 
   O     O
  R S O 
  0  2    


∴ Required time = 15 years.

 O    F 
 8.       

 O  !.
Rate = 8% O 
 0  2 
"  
Method-II: N   
 O 
  R0  2 S  
    

O 
^ ²    0  2 
^! 

www.sakshieducation.com
www.sakshieducation.com Quantitative Aptitude

O  i !  l
 N 0  2  N   ;0  2  <
 h  k
O  h  k
  0  2     h   ;0  2  <k
 g  j
 
= 4818.30.
   
9. N     2d ef    2   !!  
   0  2
 
Now,      > 2.
   
    !!
So, n = 4 years. 

10. Rate of increase from 13380 to     !!  
20070=50% = 48183000 (188l x – 1664 x)
Hence sum must be 8920 = (50787000 - 48183000)
11. S.I. on Rs. 800 for 1 year = Rs. (840 - 217 x = 2604000
800)  
= Rs. 40.   

  16. SI of. the first year = Rs.

= Rs. 400
O.-(  0 2 %  % 
! 
So, we can say that Rs. 32 is the simple
12. Production after two years = 2420
F interest on Rs. 400 for one year.
 U  V = 2420 Therefore,

O     R=
=G
=
. 
= 4%
 0  2  0 2 E 
  
O 
  O % !  !  
   m. n. op O . !  O . 0 2
13. For first year SI = CI 
 .
Therefore, the required difference
Total Difference = (32 + 66.56)
= SI at 12% for one year and SI at 11%
 !. 
for one year on Rs. 10000
  17. Let the sum be Rs. P. Then
  = Rs. 132
   N 
14. Let the value of each installment be Rs. N R  0 2  S 
 
x. Then, (P.W. of Rs. x due 1 year  . 
hence) + (P.W. of Rs. x due 2 years
 
hence) = Rs. 2550  N R0 2    S  . 
   
  
= 2550  !  
   
   N[  \
    
      
  N0  2!
       !
   18. Difference in C.I. and S.I. for 2 years
0 2   = Rs. (696.30 - 660 ) =Rs. 36.30.

Value of each installment = Rs. 1352. S.I. for one year = Rs. 330.
15. Let the investment in scheme A be Rs. x. ∴ S.I. on Rs. 330 for 1 year = Rs. 36.30
Then, investment in scheme B    .
O.-(  0 2 %  %
= Rs.   .   
19. Let the principal be Rs. P and rate of
interest be R% per annum.

www.sakshieducation.com
www.sakshieducation.com Quantitative Aptitude

Difference of C.I. and S.I. for 2 years  


⇒101.50 = N [   \
ࡾ  ࡼൈࡾൈ૛ ࡼࡾ 
ൌ ቈࡼ ൈ ൬૚ ൅ ൰ െ ࡼ቉ െ ൬ ൰ൌ 
૚૙૙ ૚૙૙ ૚૙ ⇒101.50 = N 

Difference of C.I. and S.I. for 3 years  .    
O  NO N 
 RN  0  2  NS  0 2
  Now, SI =
EF



 
NO   O 23. Difference between CI and SI
 0 2
  D = Rs. 40.80 - 40.00 = Rs. 0.80
NO   O 
    O SI of. the first year = Rs. = Rs. 20
  
  0 2
NO !  So, we can say that Rs. 0.80 is the interest of
 Rs. 20 for one year. Therefore,
   =G . 
 O   % R= = = 4%
! !  E 
૜ =G 
20. Balance  [q   
૛૙
 r Now, principal = = = Rs. 500
૚૙૙ F 

૛૙ ૛
q       
૚૙૙
૛૙
    >\
૚૙૙
   
=Rs. W        
   
 
   X
 
= Rs. [21600-(2880+2400+2000)]
= Rs. 14320.
21. Let x and y be .the shares of elder and
younger sons respectively. The amounts
invested in their names fetch interest for
21 - 19 = 2 year and 21 - 17 = 4 year
respectively. Since, the two sons are to
receive equal amounts when they attain
21 year. We have,

  ૝
 
 

 
 
 
 s H t  : 

  !!  !


and y = × 88400 = 40000

Hence, required shared of sons are Rs.
48400 and Rs. 40000 respectively.
F 
22. CI = N W    X

 
⇒ 101.50 = N [    \


www.sakshieducation.com
www.sakshieducation.com Quantitative Aptitude

21. PROBABILITY
2. In a simultaneous toss of two coins, find the
The probability is the chance of occurring of a probability of exactly one tail?
certain event when expressed quantitatively, i.e.    
(a) (b) (c) (d)
   
probability is a quantitative measure of the
3. In a simultaneous toss of three coins find the
certainty.
probability of all heads
Event:    
The possible outcomes of a trial are (a) (b) (c) (d)
   
called events. 4. If four coins are tossed, find the probability to
Sample space: get at least 1 head?
   
The set of all possible outcomes of an (a) (b) (c) (d)
   
experiment is called a sample space. We 5. A Dice is thrown. Find the probability that
generally denote it by S. the number showing on the dice is divisible by 2
Algebra of events:     
(a) (b) (c) (d) (e)
If A and B are two events associated     

with sample space S, then 6. In a single throw of two dice, find the
(i) A∪B is the event that either A or B or both probability of getting doublet?
   
occur. (a) (b) (c) (d)
   
(ii) A∩B is the event that A and B both occur. 7. Two dice are tossed. Find the probability that
Mathematical definition of probability: the total is a prime number?
Probability of an event A, denoted as    
(a) (b) (c) (d)
P(A), is defined as    
       8. In simultaneous throw of a pair of a dice, find
P(A)=
     the probability that the sum of numbers shown
The probability of the happening of a on the two faces divisible by 5 or 6
certain event is denoted by p and that of not    
(a) (b) (c) (d)
   
happening by q.
9. What is the probability of one card drawn at
Here, p, q are non-negative and cannot
random from the pack of playing cards may be
exceed unity, i.e. 0≤p≤1 and 0≤q≤1.
either a queen or an ace?
For any two events A and B,        
   . (a) (b) (c) (d)
   
If A and B are two independent events, then 10. One card is drawn from pack of playing
    .  cards. Obtain the probability that it is a letter
card or a heart
  
EXERCISE (a)

(b)

(c)

1. In a simultaneous toss of two coins, find the  
(d) (e)
 
probability of two heads?
   
11. A card is drawn from a pack of 52 cars. Find
(a) (b) (c) (d) the probability that it is a diamond.
   

www.sakshieducation.com
www.sakshieducation.com Quantitative Aptitude

   19. A bag contains 5 red, 8 blue balls and also


(a) (b) (c)
  
  contains 4 green 7 black balls. If a ball is drawn,
(d) (e) find the probability to that is not green?
 
12. The probability of getting a king and a    
(a) (b) (c) (d)
queen when two cards are drawn from a pack of    
20. If events A and B are independent and P(A)
52 cards is:
  = 0.15 P(A   .  then P(B)=?
(a) (b) (c) 63   
  (a) (b) (c)
    
(d) (e)
  (d) None of these
13. If 3 cards are drawn simultaneously form a 21. A family has 2 children. What is the
pack of well shuffled cards, find the probability probability that both the children are girls given
of then being all Queen. that at least one of them is a girl?
     
(a) (b) (c) (a) (b) (c)
     
   
(d) (e) (d) (e)
   
14. A card is drawn from a pack of 100 cards
numbered 1 to 100. Find the probability of
drawing a number which is square? Answer Key
   
(a) (b) (c) (d) 1 a 8 a 15 a
   
2 b 9 a 16 c
15. A box contains 49 tickets numbered 1 to 49.
3 a 10 d 17 a
One ticket drawn at randomly, find the
4 c 11 a 18 b
probability that number on the ticket is either
5 c 12 a 19 c
divisible by 3 or is a perfect square?
6 d 13 e 20 c
   
(a) (b) (c) (d) 7 a 14 d 21 c
   
16. One bag contains 4 white 2 black balls.
Another contains 3 white and 5 black balls. One
ball is drawn from each bag. Find the
probability that both are white?
   
(a) (b) (c) (d)
   
17. A can solve 80% of the problems given in
an exam and B can solve 70%. What is the
probability that exactly one of them will solve a
problem selected at random from the exam?
   
(a) (b) (c) (d)
   
18. In an arrangement of ‘SHIP’. Find the
probability that ‘S’ letter occupies the first
place?
   
(a) (b) (c) (d)
   

www.sakshieducation.com
www.sakshieducation.com Quantitative Aptitude

SOLUTIONS n(S) = 6
Numbers divisible by 2 are 2, 4 and 6
1. Sample space, S = {HH, HT, TH, TT} n(E) =3
 
n(S) = 4 P(E) = 
 
There is only one chance to get two heads. That 
6. Ans: d)
is {HH} 
Sample spaces = {(1, 1), (1, 2) … (1, 6)
So, n(E) = 1
 !  (2, 1), (2, 2) ... (2, 6)
Probability =  (3, 1), (3, 2) ... (3, 6)
"! 

2. Ans: (b) (4, 1), (4, 2) ... (4, 6)

(5, 1), (5, 2) ... (5, 6)
Sample spaces = {HH, HT, TH, TT}
(6, 1), (6, 2) ... (6, 6)}
Number of elements in the sample space n(S) =
n(S) = 36
4
Favorable cases to get doublets are = {(1, 1), (2,
Probability of exactly 1 tail occurs in 2 ways.
2), (3, 3), (4, 4), (5, 5), (6, 6)}
That is {HT, TH}
n(E) = 6
So, n(E) = 2
 !  
 !   Probability =  
Probability =   "!  
"!  
 7. Ans: (a)
3. Ans: (a)
   , , , , … … . , 
Sample spaces = {HHH, HHT, HTH,   
THH, TTH, HTT, TTT, THT} Favorable cases to get prime numbers as sum=
Number of sample spaces = n(S) = 8 {(1,1) (1,2) (1,4) (1,6) (2,1), (2,3) (2,5) (3,2),
Probability of getting all heads occurs in (3,4) (4,1) (4,3) (5,2) (5,6) (6,1), (6,5)}
{HHH} 1 way n(A)= 15
n(E1) = 1 #!  
 !      
Probability =  $!  
"!  
 8. Ans:

4. Ans: (c)
 n(S) =  !   
Sample spaces = {HHHH, HHHT, Event of getting a sum of numbers shown on the
HHTH, HTHH, THHH, TTHH, THHT, HHTT, two faces divisible by 5 or 6
TTTH, TTHT, THTT, HTTT, = [(1,4), (1,5), (2,3), (2,4), (3,2), (3,3), (4,1),
TTTT, HTHT, HTTH, THTH} (4,2),(4,6),(5,1), (5,5),(6,4), (6,6)]
n(S) = 16 (2n possibilities, here n = 4) n(E) =13
Favorable cases to get at least 1 head = {HHHH, %&! 
P(E) = 
HHHT, HHTH, HTHH, THHH, TTHH, THHT, %'! 

HHTT, TTTH, TTHT, THTT, HTTT, HTHT, 9. Ans: (a)

HTTH, THTH} There are 52 playing cards in a pack of
n(E) = 15 cards. So, n(S) = 52
 ! 
Probability =  (a) Getting a queen:
"! 
 There are 4 queen cards, one from each verity of
5. Ans: (c)
 symbols.
Sample spaces = {1, 2, 3, 4, 5, 6} So, n (A) = 4
www.sakshieducation.com
www.sakshieducation.com Quantitative Aptitude

b) Getting an ace: n(A) = 16


There are 4 aces in a pack. So, n (B) = 4 #! 
P(A) = 
#!  (!  "! 
P (A) = = and P(B) =  n(B) = favorable cases, perfect square = {1, 4, 9,
"!  "! 
 16, 25, 36, 49}
P (A∩B) = (There is no cards common

n(B) = 7
between them) (! 
So, P (A∪B) = P (A) + P (B) – P (A∩B) P(B) = 
"! 
  
= n(A∩B) = {9, 36} = 2
  
#+(! 


=
 P(A∩B) = 
"! 
 
 P(A∪B) = P (A)+P (B)- P (A B)
10. Ans: (d)    

= 
Let A=letter card and B=Heart    

Required probability = (A or B) P (A∪B) =

P(A∪B) = P(A)+P(B)-P(A∩B) 16. Ans: (c)
   
=  Probability of drawing white ball from first bag
   
 ,- -. /0123 4566  
11. Ans: (a) =  
 2-256 4566  
Total number of ways drawing one card from 52 Probability of drawing white ball from second
,- -. /0123 4566 
cards=52C1 bag= 
2-256 4566 
One diamond card can be chosen in 13C1 ways. Since, these are independent
)  
P (getting diamond) =   The probability of both the balls is white =
)  
    
12. Ans: (a) !  
    
Probability (a king and a Queen) 17. Ans: (a)
)*) *  A can solve 80% of problems
=  ૞૛ൈ૞૚  
) 
૛ n(A) = 80% =
 
13. Ans: (e)  

)   P( ) = 1-(P(A)) =  
 
P (getting queen) =  
)   B can solve 70% of problems

14. Ans: (d) 
 n (B) = 70% =

Here n(S) = 100 (As mentioned in the  
   
problem)  

Favorable cases to get the square cards = {1, 4, /////


"#$%&"#' (")*+*&,&-.   
9, 16, 25, 36, 49, 64, 81, 100}  
n(E) = 10 0  1 
 ! !
 !      
Probability =     1
"!  
  !
15. Ans: (a)   

 1 2
Sample space = n(S) = 49  
  
n(A) = favorable cases divisible by 3 
18. Ans: (b)
= {3, 6, 9, 12, 15, 18, 21, 24, 27, 30, 33, 36, 39, 
42, 45, and 48} ‘s’ occupies the first place

www.sakshieducation.com
www.sakshieducation.com Quantitative Aptitude

#!
"#$%&"#' (")*+*&,&-.    
$!
n(s) = no of samples = 4
n(A) = no of letters excluding ‘s’
.
= 4-1=3 letters
n(A) = 3!
! 
P(A) = 
! 
19. Ans: (c)
4   0 1   
   5"## *+,,  
   
    
  
 
Required ratio P( ) = 1- P(A)= 
 
20. Ans: (c)
        
.   .     
0.45  .   .  ! 
.   .  . 0
. 0  . 
.   
   
. 0 0 1

21. Ans: (c)

Let b stand for boy and g for girl. The
sample space of the experiment is S= {(b, b), (g,
b), (b, g), (g, g)}
There are two events
A---> both the children are girls
B----> at least one of the child is a girl
{(g, g)} and {(g, b),(b, g),(g, g)}
A∩B = {g, g}
 
P(B)= and P(A∩B)=
 


P( /)=  ∵ conditional probability




www.sakshieducation.com
www.sakshieducation.com Quantitative Aptitude

22.PERMUTATIONS AND COMBINATIONS

4. How many three digit numbers can be


Permutations: If r objects are to be chosen formed by using the digits in 755628. If
from n, where n ≥ r and these r objects are to repetition is not allowed?
be arranged and the order in which these (a) 100 (b) 120 (c) 150 (d) 180
objects are arranged is important, such an 5. There are 10 students in a batch. In how
arrangement is called a permutation of n many ways can the first five ranks be getting
objects taken r a time. Permutation is in?
denoted by n p r or p (n, r). (a) 30000 (b) 30240
! (c) 30500 (d) 30600
n
p r  ! ,   
6. Find the number of different words that
Combinations: If r objects are to be chosen
can be formed from the word ‘SUCCEED’
from n, where n ≥r and the order of choosing
(a) 1390 (b) 1400
the r objects is not important, then such a
(c) 1320 (d) 1260
choices is called a combination of n objects
7. The number of arrangements that can be
taken r a time. Combination is denoted by n C

made with the letters of the word ‘MATHS’.
r or  So, that Letter M will occupy always the first

! place is?
n
C r ,  
!!
(a) 3! (b) 4! (c) 5! (d) 6!
Factorial Notation: Let n be a positive
8. How many 3-letter words with or without
intE.g.er. Then, factorial n, denoted by n! is
meaning, can be formed by using all the
defined as:
letters of the word ‘LOGARITHMS’, if
n! = n (n – 1) (n – 2) …. 3.2.1.
repetition of letters is not allowed?
EXERCISE (a) 700 (b) 720 (c) 750 (d) 760
1. If 6 boys and 6 girls have to sit in a round 9. The number of arrangement of the word
circular music chair. So, that there is a girl ‘CABLE’ So that the vowels always occupy
between every 2 boys. Find the number of odd positions is?
ways they can sit? (a) 12 (b) 18 (c) 24 (d) 36
(a) 6! × 5! (b) 6! × 4! 10. What is the number of words formed
(c) 6! × 3! (d) 6! × 2! from the letters of the word ‘JOKE’ So that
2. Find the number of ways in which 6gents the vowels and consonants alternate?
and 6 ladies be seated in a row. So that all (a) 4 (b) 8 (c) 12 (d) None of these
the ladies sit together and all the gents sit 11. Find the number of different signals that
together? can be transmitted by arranging 3 yellow
(a) 2(5!)2 (b) 2(6!)2 flags, 4 red flags and 2 blue flags on a pole.
(c) 2(7!)2 (d) 2(8!)2 All the flags are used to transmit the signal.
3. What is the number of ways in which an (a) 1200 (b) 1260
ascending A.P, Comprising three numbers (c) 1300 (d) 1350
can be formed from 1, 2, 3, 4, 5, 6, 7? 12. In how many different ways a group of 4
(a) 8 (b) 9 (c) 10 (d) 11 men and 4 women can be formed out of 7
men and 8 women?
(a) 2450 (b) 1050

www.sakshieducation.com
www.sakshieducation.com Quantitative Aptitude

(c) 117 (d) 232 r=3


13. Find the no of parallelograms that can be !  !
So, 5P3 =  = =6×5
 ! !
formed from a set of four parallel lines
× 4 × 3 = 360
intersecting from a set of four parallel lines
360 three digit numbers can be
intersecting another set of three parallel
formed by using the digits in 755628
lines?
5. Ans: (b) 30240
(a) 12 (b) 24 (c) 48 (d) 18
Explanation:
14. How many rectangles can be formed on a
Number of students = 10
chess board?
First 10 ranks have to be get = r = 5
(a) 1248 (b) 1264 !
n
(c) 1296 (d) 1292 Pr =
 –!
15. Find the number of combinations of four 10 !     !
P5 =  =
things selected out of 8 things? ! !

(a) 50 (b) 60 (c) 70 (d) 80 = 10 × 9 × 8 × 7 × 6


= 30240
Value of 10P5 = 30240
Answer Key 6. Ans: (d) 1260
1 a 6 d 11 b Explanation:
2 b 7 b 12 a Number of letters = 7 = n
3 b 8 b 13 d Number of C = 2 = p
4 b 9 d 14 c Number of E = 2 = q
5 b 10 b 15 c ! !   !
= =
! ! !! !  
= 7 × 6 × 5 × 2 × 3 = 1260
7. Ans: (b) 4!
SOLUTIONS
Explanation:
1. Ans: (a) 6! × 5!
M A T H S
Explanation:
Always M is in first place
Circular permutation = n! (n – 1)!
M
∴ Number of ways = 6! (6 – 1)!
∴ Number of ways = 4!
= 6! × 5!
8. Ans: (b) 720
2. Ans: (b) 2 (6!)2
Explanation:
Explanation:
In ‘LOGARTHMS’
The number of ways = 2 (6!)(6!)
Total number of letters = 10 No’s
= 2 (6!)2
Out of 10 letters, 3 letter can be formed =
3. Ans: (b) 9    !
10
Explanation: P3 = =720
!
The possible A.P types, 9. Ans: (d) 36
For d=1: (1, 2, 3), (2, 3, 4), (3, 4, 5), (4, 5, 6), Explanation:
(5, 6, 7) No of letters in ‘CABLE’ = 5 No’s
For d=2: (1, 3, 5), (2, 4, 6), (3, 5, 7) No of vowels = (A, E) = 2 No’s
For d =3: (1, 4, 7) No of arranging vowels at any two odd
The number of required A.P is 9. places out of three = 3P2
4. Ans: (b) 120 Since 5 letters word has 3 odd places.
Explanation: And number of arranging 3 consonants on
Number of digits n = 5 the remaining 3 places =3!

www.sakshieducation.com
www.sakshieducation.com Quantitative Aptitude

∴ required no of arrangements = 3P2×3! =     


That is,    
!    
×3! = 6×6 = 36 arrangements   
!
10. Ans: (b) 8 13. Ans: (d) 18
Explanation: Explanation:
Word name: ‘JOKE’ Required number = 4C2 × 3C2
Vowels: O, E 4 !  
C2 = = =6
! ! ! !
Consonants: J, K !
3
∴ Possible arrangement C2 = =3
!!
4
Beginning with consonant: JOKE, KOJE, C2 ×3C2 = 6×3 = 18
JEKO, KEJO = 4 Numbers
Beginning with vowel: OJEK, OKEJ, EJOK, 14. Ans: (c) 1296
EKOJ = 4 Numbers Explanation:
Required number = 4+4 = 8 numbers. There are 9 horizontal and 9 vertical
11. Ans: (b) 1260 lines on a chess board. Hence,
Explanation: The no of rectangles = 9C2×9C2
Total number of flags = 9 n !
Cr =
!!
Number of different signals which can be
9 ! !   !
! C2 = = = =36
transmitted = !! ! ! ! !
! ! !
    ! ∴ 9C2×9C2 = 36×36 = 362= 1296
=
    ! 15. Ans: (c) 70
= 9 × 4 × 7 × 5 = 1260 Explanation:
12. Ans: (a) 2450 Out of 8 things 4 things selected =
Explanation: 8
C4
4 men out of 7 men and 4 women out 8 !
C4 =
of 8 women can be formed in 7C4×8C4 ways ! !
   !
= = 70
!  

www.sakshieducation.com

You might also like